Pharmacology FINALLLpdf

You might also like

Download as pdf or txt
Download as pdf or txt
You are on page 1of 223

KROK 1 PHARMACOLOGY

1.Methotrexate (structural analogue of the folic acid which is competitive inhibitor


of the dihydrofolatreductase) is prescribed for treatment of the malignant
tumour. On which level does methotrexate hinder synthesis of the nucleic
acids?
A. Replication
B. Transcription
C. *Mononucleotide synthesis
D. Reparation
E. Processing

2.A 60-year-old patient was hospitalised to the surgical department because of


infection caused by blue pus bacillus (Pseudomonas aeruginosa) which is
sensative to penicillin antibiotics. Indicate which of the given penicillins has
marked activity to the Pseudomonas aeruginosa?
A. Benzylpenicillin
B. Phenoxymethylpenicillin
C. *Carbenicillin disodium
D. Oxacillin
E. Methicillin

3.A 45-year-old woman suffers from allergic seasonal coryza caused by the
ambrosia blossoming. What adipose cells group stabilizer medicine can be
used for prevention of this disease?
A. Diazoline
B. Phencarol
C. Tavegyl
D. Dimedrol
E. *Ketotifen

4.The alternate usage of dichlotiazide, etacrin acid and lasex did not cause marked
diuretic effect in the patient with marked peripheral edema. The aldosterone
level in the blood is increased. Indicatewhichmedicine should be prescribed:
A. Mannit
B. *Spironolacton
C. Clopamid
D. Urea
E. Amilorid

5.A 56-year-old patient complaining of thirst and frequent urination was


diagnosed with diabete mellitus. Butamin was prescribed. How does the
medicine act?
A. It helps to absorb the glucose by the cells of the organism tissues
B. *It stimulates beta-cells of Langergans islets
C. It relieves transport of glucose through the cells’ membranes
D. It inhibits alpha-cells of Langergans islets
E. It inhibits absorption of glucose in the intestines

6.A 50-year-old male farm worker has been brought to the emergency room. He
was found confused in the orchard and since then has remained unconscious.
His heart rate is 45 and his blood pressure is 80/40 mm Hg. He is sweating and
salivating profusely. Which of the following should be prescribed?
A. *Atropine
B. Norepinephrine
C. Proserine
D. Physostigmine
E. Pentamine

7.A 58-year-old female has undergone surgery for necrotic bowel. Despite having
been treated with antibiotics, on postoperative day 5, she develops symptoms
(fever, hypotension, tachycardia, declining urine output, and confusion)
consistent with septic shock. What hemodynamic support would be helpful?
A. Dobutamine infusion
B. Antibiotic administration
C. Fluid administration
D. *Fluids and Dobutamine infusion
E. Atropine administration

8.A 42-year-old man who has been injured in a car accident is brought into the
emergency room. His blood alcohol level on admission is 250 mg/dL. Hospital
records show a prior hospitalization for alcohol related seizures. His wife
confirms that he has been drinking heavily for 3 weeks. What treatment should
be provided to the patient if he goes into withdrawal?
A. Phenobarbital
B. Pentobarbital
C. *Diazepam
D. Phenytoin
E. None

9.A 13-year-old girl with history of asthma complained of cough, dyspnea and
wheezing. Her symptoms became so severe that her parents brought her to the
emergency room. Physical examination revealed diaphoresis, dyspnea,
tachycardia and tachypnea. Her respiratory rate was 42/min, pulse rate was 110
beats per minute, and blood pressure was 130/70 mm Hg. Choose from the
following list the most appropriate drug to reverse the bronchoconstriction
rapidly:
A. Cromolyn
B. *Salbutamol
C. Beclomethasone
D. Methylprednidsolone
E. Ipratropium

10.A doctor administered Allopurinol to a 26-year-old young man with the


symptoms of gout. What pharmacological action of Allopurinol ensures
therapeutical effect?
A. By increasing uric acid excretion
B. By inhibiting leucocyte migration into the joint
C. By general anti-inflammatory effect
D. By general analgetic effect
E. *By inhibiting uric acid synthesis

11.A patient had been taking glucocorticoids for a long time. When the
preparation was withdrawn he developed the symptoms of disease aggravation,
decreased blood pressure and weakness. What is the reason of this condition?
A. Hyperproduction of ACTH
B. Sensibilization
C. *Appearance of adrenal insufficiency
D. Habituation
E. Cumulation

12.The patient with pneumonia was treated with antibiotics for a long period.
After treatment patient complains of frequent and watery stool, abdomenal
pain. What is the reason of intestine function disorder?
A. Antibiotics toxic influence on the GIT
B. Autoimmune reaction development
C. Bacteria toxins influence
D. *Intestinal disbacteriosis development
E. Hereditary enzyme defect

13.A 55-year-old patient with continuing ventricular arrhythmias was admitted to


the hospital. The patient is taking timolol drops for glaucoma, daily insulin
injections for diabetes mellitus, and an ACE inhibitor for hypertension. You
have decided to use phenytoin instead of procainamide. What is the reason?
A. The local anesthetic effect of procainamide would potentiate diabetes
B. The hypertensive effects of procainamide would aggravate the hypertension
C. The local anesthetic effect of procai-namide would aggravate the hypertension
D. The cholinergic effects of procainamide would aggravate the diabetes
E. *The anticholinergic effect of procainamide would aggravate glaucoma

14.A 35-year-old man under the treatment for pulmonary tuberculosis has acute-
onset of right big toe pain, swelling, and low-grade fever. The gouty arthritis
was diagnosed and high serum uric acid level was found. Which of the
following antituberculosis drugs are known for causing high uric acid levels?
A. *Pyrazinamide
B. Cycloserine
C. Thiacetazone
D. Rifampicin
E. Aminosalicylic acid

15.A 25-year-old woman with red and itchy eczematoid dermatitis visits your
office. She had a dental procedure one day earlier with administration of a
local anesthetic. There were no other findings, although she indicated that she
had a history of allergic reactions. Which of the following drugs is most likely
involved?
A. Cocaine
B. *Procaine
C. Lidocaine
D. Bupivacaine
E. Etidocaine

16.The CNS stimulation produced by methylxanthines, such as caffeine, is most


likely due to the antagonism of one of the following receptors:
A. Glycine receptors
B. Glutamate receptors
C. *Adenosine receptors
D. GABA receptors
E. Cholinergic muscarinic receptors

17.A 37-year-old man was admitted to the surgical department with the symptoms
of acute pancreatitis: vomiting, diarrhea, bradycardia, hypotention, weakness,
dehydration of the organism. What medicine should be used first of all?
A. *Contrycal
B. No-spa
C. Platyphylline
D. Etaperazine
E. Ephedrine

18.A patient has been taking a mixture prescribed by neuropathologist for


neurasthenia for two weeks. The patient feels better but has developed coryza,
conjunctivitis, rash, inertia, decrease of memory. She is diagnosed with
bromizm. What should be prescribed to decrease the symptoms?
A. Glucose solution 5 %
B. *Natrium chloride
C. Asparcam
D. Polyglucin
E. No-spa
19.Analeptical remedy of reflective type from the H-cholinomimetics group was
given to the patient for restoration of breathing after poisoning with carbon
monoxide. What medicine was prescribed to the patient?
A. Atropine sulphate
B. Adrenalin hydrochloride
C. *Lobeline hydrochloride
D. Mesaton
E. Pentamin

20.A patient with bronchial asthma had been taking tablets which caused
insomnia, headache, increased blood pressure. What medecine can cause such
complications?
A. Adrenaline
B. Chromolin sodium
C. *Ephedrine
D. Euphyline
E. Izadrine

21.To anaesthetize the manipulation related to burn surface treatment, a patient


was intravenously injected a medication for short-acting narcosis. 1 minute
later the patient being under anaesthesia had increased blood pressure,
tachycardia, increased tone of sceletal muscles; reflexes remained. After
awakening the patient had desorientation and visual hallucinations. What
medication was the patient injected?
A. Sombrevin
B. Diethyl ether
C. *Ketamine
D. Thiopental sodium
E. Nitrous oxide

22.Intake of oral contraceptives containing sex hormones inhibits secretion of the


hypophysiae hormones. Secretion of which of the indicated hormones is
inhibited while taking oral contraceptives with sex hormones?
A. Vasopressin
B. Thyrotropic
C. Somatotropic
D. *Follicle-stimulating
E. Oxytocin

23.A 60-year-old patient was admitted to the surgical department because of


infection caused by blue pus bacillus (Pseudomonas aeruginosa) which is
sensative to penicillin antibiotics. Indicate which of the given penicillins has
marked activity to the Pseudomonas aeruginosa?
A. *Carbenicillin disodium
B. Benzylpenicillin
C. Phenoxymethylpenicillin
D. Oxacillin
E. Methicillin

24.A 52-year-old patient has the following diagnosis: systemic amebiasis with
involvment of intestines, liver, lungs. What drug should be prescribed?
A. Quiniofone
B. Tetracycline
C. *Metronidasol
D. Quingamine
E. Enteroseptol

25.A 38-year-old man who poisoned himself with mercury dichloride was taken
to the admission room in grave condition. What antidote should be
immediately introduced?
A. Dipiroxim
B. Atropine
C. Nalorphine
D. Isonitrosine
E. *Unithiol

26.A patient with complaints of dryness in the mouth, photophobia and vision
impairment was admitted to the reception-room. Skin is hyperemic, dry, pupils
are dilated, tachycardia. Poisoning with belladonna alkaloids was diagnosed on
further examination. What medicine should be prescribed?
A. *Prozerin
B. Diazepam
C. Pilocarpine
D. Armine
E. Dipiroxim

27.A patient with rheumatoid arthritis who had been treated with indometacin has
got signs of gastropathy. What activity of the drug can this complication be
connected with?
A. Antiserotonin
B. Antihistamine
C. *Anticyclooxygenase
D. Antikinine
E. Locally irritating

28.A patient who was previously ill with mastectomy as a result of breast cancer
was prescribed radiation therapy. What vitamin preparation has marked
radioprotective action caused by antioxidant activity?
A. Ergocalciferol
B. *Tocopherol acetate
C. Thiamine chloride
D. Riboflavin
E. Folic acid

29.A patient has got a spasm of smooth muscles of bronchi. Activators of what
membrane cytoreceptors are phisiologically reasoned to stop an attack?
A. Alpha-аdrenoreceptors
B. Alpha- and beta-аdrenoreceptors
C. *Beta-adrenoreceptors
D. Н-cholinoreceptors
E. М-cholinoreceptors

30.A 42-year-old man suffering from gout has increased level of urinary acid in
blood. Allopurinol was prescribed to decrease the level of urinary acid.
Competitive inhibitor of what enzyme is allopurinol?
A. Adenosine deaminase
B. Adenine phosphoribosiltransferase
C. *Xanthine oxidase
D. Hypoxanthine phosphoribosiltransferase
E. Guanine deaminase

31.A patient with infectious mononucleosis had been taking glucocorticoids for
two weeks. He was brought into remission, but he fell ill with acute attack of
chronic tonsillitis. What action of glucocorticoids caused this complication?
A. *Immunosuppressive
B. Anti-inflammatory
C. Antishock
D. Antiallergic
E. Antitoxic

32.A patient working at a chemical plant was admitted to the toxicological


department with mercury poisoning. What medicine should be used?
A. Isonitrozin
B. Naloxone
C. *Unithiol
D. Activated carbon
E. Enterosorbent

33.A patient with continious bronchopneumonia was admitted to the therapeutic


department. Antibiotic therapy didn’t give much effect. What medication for
improvement of immune state should be added to the complex treatment of this
patient?
A. Analgin
B. *Timaline
C. Sulfocamphocaine
D. Benadryl
E. Paracetamol

34.Patient with abscess of the cut wound applied to the traumatological


department. In order to clean the wound from the pus doctor washed it with
3 % hydrogen peroxide. Foam was absent. What caused the absence of the
drug activity?
A. Low concentration H2O2
B. Inherited insufficiency of erythrocyte phosphatdehydrogenase
C. Shallow wound
D. Pus in the wound
E. *Inherited insufficiency of catalase

35.A 50-year-old patient with typhoid fever was treated with Levomycetin, next
day his condition became worse, temperature rose to 39,60. What caused the
complication?
A. *The effect of endotoxin agent
B. Allergic reaction
C. Irresponsiveness of an agent to the levomycetin
D. Secondary infection addition
E. Reinfection

36.Tuberculosis can be treated by means of combined chemotherapy that includes


substances with different mechanisms of action.What antituberculous
medication inhibits transcription of RNA into DNA in mycobacteria?
A. Isoniazid
B. *Rifampicin
C. Streptomycin
D. Ethionamide
E. Para-aminosalicylic acid

37.A patient who has been treated with diazepam on account of neurosis
complains of toothache. Doctor administered him an analgetic, but its dose was
lower than average therapeutic dose. What phenomenon did the doctor take
into account while prescribing the patient an underdose?
A. Summation
B. Cumulation
C. *Potentiation
D. Drug dependence
E. Tolerance

38.To anaesthetize the surgical treatment of burn surface, a patient was


intravenously injected a medication for short-acting narcosis. 1 minute later the
patient being under anaesthesia showed increased blood pressure, tachycardia,
increased tone of skeletal muscles; reflexes remained. After recovering from
anaesthesia the patient had disorientation and visual hallucinations. What
medication was the patient injected?
A. Sombrevin
B. *Ketamine
C. Diethyl ether
D. Thiopental sodium
E. Nitrous oxide

39.A patient had to go through an operation. Doctors introduced him dithylinum


(listenone) and performed intubation. After the end of operation and cessation
of anesthesia the independent respiration wasn’t restored. Which enzyme
deficit prolongs the action of muscle relaxant?
A. Succinate dehydrogenase
B. Carbanhydrase
C. N-acetyltransferase
D. *Pseudocholinesterase
E. K-Na-adenosine triphosphatase

40.A patient consulted a doctor about bowels disfunction. The doctor established
symptoms of duodenitis and enteritis. Laboratory examination helped to make
the following diagnosis: lambliosis. What medication should be administered?
A. Erythromycin
B. Monomycin
C. Chingamin
D. *Metronidazole
E. Tetracycline

41.Introduction of a pharmaceutical substance to an experimental animal resulted


in reduction of salivation, pupil mydriasis. Next intravenous introduction of
acetylcholine didn’t lead to any significant changes of heart rate. Name this
substance:
A. *Atropine
B. Adrenaline
C. Propranolol
D. Proserin
E. Salbutamol

42.Continious taking of a drug can result in osteoporosis, erosion of stomach


mucous membrane, hypokaliemia, retention of sodium and water, reduced
content of corticotropin in blood. Name this drug:
A. Hydrochlorothiazide
B. *Prednisolone
C. Digoxin
D. Indometacin
E. Reserpine
43.A 63 y.o. man with collapse symptoms was delivered to the emergency
hospital. A doctor chose noradrenaline in order to prevent hypotension. What is
the action mechanism of this medication?
A. Activation of serotonin receptors
B. Activation of beta-adrenoreceptors
C. *Activation of alpha1-adrenoreceptors
D. Activation of dopamine receptors
E. Block of M –cholinoreceptors

44.A 35 y.o. patient who often consumes alcohol was treated with diuretics. There
appeared serious muscle and heart weakness, vomiting, diarrhea, AP-100/60
mm Hg, depression. This condition is caused by intensified excretion with
urine of:
A. Sodium
B. Chlorine
C. Calcium
D. Phosphates
E. *Potassium

45.A patient was diagnosed with active focal pulmonary tuberculosis. What drug
should be prescribed in the first place?
A. *Isoniazid
B. Sulfalen
C. Cyclocerine
D. Ethionamide
E. Ethoxide

46.Examination of a patient revealed extremely myotic pupils, sleepiness,


infrequent Chain-Stoke’s respiration, urinary retention, slowing-down of heart
rate, enhancement of spinal reflexes. What substance caused the poisoning?
A. Atropine
B. Phosphacole
C. *Morphine
D. Caffeine
E. Barbital

47.A doctor administered a patient with allergic dermatitis a H 1-histamine blocker


as a part of complex treatment. Name this medication:
A. Cromolyn sodium
B. Prednisolone
C. Adrenaline
D. *Loratadine
E. Hydrocortisone
48.A patient with II stage hypertension has been taking one of hypotensive
medications for the purpose of treatment. After a time arterial pressure
decreased, but the patient started complaining of flaccidity, sleepiness,
indifference. A bit later he felt stomach pain. He was diagnosed with ulcer.
What hypotensive medication has the patient been taking?
A. Dibazole
B. *Reserpine
C. Furosemide
D. Verapamil
E. Captopril

49.For the preparation of a patient’s burn skin surface a certain medication was
used. Its antiseptic action is provided by free oxygen that segregates in
presence of organic substances. Choose the right answer:
A. *Potassium permanganate
B. Furacilin
C. Chlorhexidine
D. Boric acid
E. Sodium bicarbonate

50.A 50 y.o. patient with chronic cardiac insufficiency and tachyarrythmia was
prescribed a cardiotonic drug. What drug was prescribed?
A. Dopamine
B. Dobutamine
C. *Digoxin
D. Amyodarone
E. Mildronate

51.A 36 y.o. man has a craniocerebral trauma. Objectively: diminished breath


sounds, thready pulse, no reflexes. What way of pyracetam introduction will be
the most apropriate in this case?
A. Rectal
B. Subcutaneous
C. Peroral
D. *Intravenous
E. Inhalation

52.A patient ill with bronchial asthma didn’t inform his doctor that he had attacks
of stenocardia. Doctor administered him a medication, which taking resulted in
less frequent attacks of bronchial asthma, but stenocardia attacks became more
frequent. What medication was administered?
A. Salbutamol
B. Aminophylline
C. *Isadrin
D. Cromolyn sodium
E. Phenotherol

53.A patient who has been suffering from cardiac insufficiency for several months
has been taking digoxin on an outpatient basis. At a certain stage of treatment
there appeared symptoms of drug overdose. What phenomenon underlies the
development of this complication?
A. Habituation
B. Sensibilization
C. *Material cumulation
D. Functional cumulation
E. Tachyphylaxis

54.A patient suffers from vision impairment – hemeralopy (night blindness). What
vitamin preparation should be administered the patient in order to restore his
vision?
A. *Retinol acetate
B. Vicasol
C. Pyridoxine
D. Thiamine chloride
E. Tocopherol acetate

55.A patient suffers from severe postoperative pseudomonadous infection. What


of the following antibiotics should be administered in this case?
A. Benzylpenicillin
B. Cephazolin
C. *Amicacin sulfate
D. Erythromycin
E. Doxycycline

56.After a tooth extraction a patient felt persistent pain behind his breast bone.
After sublingual intake of an antianginal drug the pain behind the breast bone
disappeared, but the patient complained of headache and dizziness. What drug
are these properties typical for?
A. Propranolol
B. *Nitroglycerin
C. Metoprolol
D. Validol
E. Verapamil

57.A patient with fracture of his lower jaw was admitted to the maxillofacial
department. It was decided to fix his bones surgically under anaesthetic. After
intravenous introduction of muscle relaxant there arose short fibrillar
contractions of the patient’s facial muscles. What muscle relaxant was applied?
A. Tubocurarin chloride
B. *Dithylinum
C. Pipecuronium bromide
D. Diazepam
E. Melictine

58.Examination of a 60 y.o. patient revealed hyperglycemia and glucosuria. A


doctor administered him a medication for internal use. What medication is it?
A. Furosemide
B. Oxytocin
C. Pancreatine
D. Corglycon
E. *Glibenclamid

59.A liquidator of a breakdown at a nuclear power plant who was irradiated


complained about vomiting that occurs all of a sudden. What medication
should be prescribed?
A. Reserpine
B. Atropine
C. *Metoclopramide
D. Aeron
E. De-Nol

60.A patient with chronic cardiac insufficiency has been treated with cardiotonic
drugs and a thiazide diuretic, but in spite of it there are still edemata and risk of
ascites. What medication should be prescribed to amplify diuretic effect of the
applied drugs?
A. Furosemide
B. Amyloride
C. *Spironolactone
D. Clopamide
E. Manitole

61.A patient ill with collagenesis has been taking prednisolone for a long time.
Hypokaliemia development caused spastic pain of skeletal muscles. What
medication should be used in order to correct potassium exchange?
A. Dithylinum
B. *Panangin
C. Diazepam
D. Noshpa
E. Thyrocalcitonin

62.A 37 year old patient suffering from obliterating vascular endarteritis of lower
limbs takes daily 60 microgram/kilogram of phenylin. Because of presentations
of convulsive disorder (craniocerebral trauma in anamnesis) he was prescribed
phenobarbital. With holding this drug caused nasal hemorhage. What is this
complication connected with?
A. Aliphatic hydroxylation of phenobarbital
B. Conjugation of phenylin with glucuronic acid
C. Oxidative deamination of phenylin
D. *Induction of enzymes of microsomal oxidation in liver caused by
phenobarbital
E. Inhibition of microsomal oxidation in liver caused by phenobarbital

63.A patient ill with neurodermatitis has been taking prednisolone for a long time.
Examination revealed high rate of sugar in his blood. This complication is
caused by the drug influence upon the following link of carbohydrate
metabolism:
A. Glycogenogenesis activation
B. *Gluconeogenesis activation
C. Intensification of glucose absorption in the bowels
D. Inhibition of glycogen synthesis
E. Activation of insulin decomposition

64.A patient with frequent attacks of stenocardia was prescribed sustak-forte to be


taken one tablet twice a day. At first the effect was positive but on the second
day stenocardia attacks resumed. What can explain inefficiency of the
prescribed drug?
A. *Tachyphylaxis
B. Cumulation
C. Sensibilization
D. Idiosyncrasy
E. Dependence

65.Examination of a 70 year old patient rrevealed insulin-dependent diabetes.


What drug should be administered?
A. Insulin
B. Mercazolilum
C. *Glibenclamid
D. Parathyroidin
E. Cortisone

66.A patient suffering from chronic cardiac insufficiency was recommended to


undergo a prophylactic course of treatment with a cardiological drug from the
group of cardiac glycosides that is to be taken enterally. What drug was
recommended?
A. Strophanthine
B. *Digoxin
C. Corglycon
D. Cordiamin
E. Cordarone
67.An ophthalmologist used a 1 % mesaton solution for the diagnostic purpose
(pupil dilation for eye-ground examination). What is the cause of mydriasis
induced by the drug?
A. Activation of alph2 adrenoreceptors
B. *Activation of alpha1 adrenoreceptors
C. Block of alph1 adrenoreceptors
D. Activation of beta1 adrenoreceptors
E. Activation of M-cholinoreceptors

68.Mother of a 2 year old child consulted a stomatologist. In the period of


pregnancy she was irregularly taking antibiotics for an infectious disease.
Examination of the child revealed incisor destruction, yellow enamel, brown
rim around the dental cervix. What drug has apparent teratogenic effect?
A. Furosemide
B. *Doxacycline
C. Ampiox
D. Xantinol nicotinate
E. Octadine

69.Examination of a 60 year old patient revealed hyperglycemia and glucosuria. A


doctor administered him a medication for internal use. What medication is it?
A. Furosemide
B. *Glibenclamid
C. Oxytocin
D. Pancreatine
E. Corglycon

70.A patient ill with essential hypertension was recommended a drug that prevents
thrombosis. It is to be taken parenterally. What drug is it?
A. *Heparin
B. Amben
C. Protamine sulfate
D. Neodicumarin
E. Syncumar

71.A 7 year old child is ill with bronchitis. It is necessary to administer him an
antibacterial drug. What drug of fluoroquinolone group is CONTRA-
INDICATED at this age?
A. Ampicillin
B. *Cyprofloxacin
C. Amoxicillin
D. Sulfadimethoxine
E. Ampiox
72.If a man has an attack of bronchiospasm it is necessary to reduce the effect of
vagus on smooth muscles of bronchi. What membrane cytoreceptors should be
blocked for this purpose?
A. N - cholinoreceptors
B. Alpha-adrenoreceptors
C. *M - cholinoreceptors
D. Beta-adrenoreceptors
E. Alpha- and beta-adrenoreceptors

73.A patient takes digoxin for treatment of cardiac insufficiency. What diuretic
may increase digoxin toxicity due to the intensified excretion of K+ ions?
A. Spironolactone
B. Panangine
C. Siliborum
D. Lisinopril
E. *Hydrochlorothiazide

74.A patient ill with amebiasis was prescribed a certain drug. The use of alcohol
together with this drug is contraindicated because the drug inhibits metabolism
of ethyl alcohol. What drug is it?
A. Reserpine
B. Clonidine
C. Diazepam
D. *Metronidazole
E. Aminazine

75.A patient ill with chronic cardiac insufficiency was prescribed an average
therapeutic dose of digoxin. Two weeks after begin of its taking there appeared
symptoms of drug intoxication (bradycardia, extrasystole, nausea). Name the
phenomenon that caused accumulation of the drug in the organism?
A. Functional cumulation
B. *Material cumulation
C. Tolerance
D. Tachyphylaxis
E. Idiosyncrasy

76.On the 2-3rd day after stomach resection intestinal peristalsis wasn’t restored.
What is to be administered for stimulation of gastrointestinal tract?
A. Prasosin
B. *Proserin
C. Cyclodole
D. Atropine sulfate
E. Acetylcholine
77.A stomatologist injected a patient with a certain drug in order to reduce
salivation during tooth filling. What drug is it?
A. Adrenaline hydrochloride
B. Proserin
C. Pilocarpine hydrochloride
D. *Atropine sulfate
E. Mesaton

78.A patient consulted a stomatologist about purulent inflammation of his gums.


What drug will be the most effective if it is suspected that a causative agent is
an anaerobe?
A. Gentamicin
B. Oxacillin sodium
C. *Metronidazole
D. Co-trimoxazole
E. Nitroxoline

79.Examination of a 35 year old patient revealed high acidity of gastric juice.


What receptors should be blocked in order to reduce it?
A. Alpha1-adrenoreceptors
B. Alpha2-adrenoreceptors
C. *Histamine
D. Beta1-adrenoreceptors
E. Beta2-adrenoreceptors

80.A 12 year old child has intolerance to some food stuffs. Their consumption
causes an allergic reaction in form of itching skin eruptions. What
antihistaminic drug should be admistered so that the child could attend school ?
A. Dimedrol
B. Diclofenac
C. *Loratadine
D. Aminophylline
E. Ephedrine

81.A patient that entered the admission office had the following signs of acute
cardiac insufficiency: paleness, acrocyanosis, frequent shallow respiration.
What drug is indicated in this case?
A. Digitoxin
B. Cordiamin
C. *Corglycon
D. Nitroglycerine
E. Adrenaline hydrochloride
82.A patient had hemorrhagic stroke. Blood examination revealed strengthened
kinin concentration.The patient was prescribed contrical. It was administered in
order to inhibit the following proteinase:
A. Pepsin
B. Trypsin
C. *Kallikrein
D. Chemotrypsin
E. Collagenase

83.A patient with bacterial pneumonia was prescribed benzylpenicillin. What is


the mechanism of its antibacterial effect?
A. Inhibition of intracellular protein synthesis
B. Abnormal permeability of cytoplasmic membrane
C. *Inhibition of synthesis of microorganism wall
D. Inhibition of SH-groups of microorganism enzymes
E. Antagonism with p-amino-benzoic acid

84.A patient who has been treated in a neural clinic and has been taking a sedative
for a long time got the following complication: cough, rhinitis, epiphora. What
drug caused these disturbances?
A. Diazepam
B. *Sodium bromide
C. Valerian
D. Phenazepam
E. Reserpine

85.In order to accelerate healing of a radiation ulcer a vitamin drug was


administered. What drug is it?
A. Retabolil
B. Prednisolone
C. *Retinol acetate
D. Levamisole
E. Methyluracil

86.Pharmacological effects of antidepressants are connected with inhibition of an


enzyme catalyzing biogenic amines noradrenaline and serotonine in the
mitochondrions of cerebral neurons. What enzyme participates in this process?
A. Transaminase
B. Decarboxylase
C. Peptidase
D. *Monoamine oxidase
E. Lyase
87.An oncological patient was prescribed methotrexate. With the lapse of time
target cells of the tumour lost susceptibility to this drug. There is change of
gene expression of the folowing enzyme:
A. Thiaminase
B. *Dehydrofolate reductase
C. Deaminase
D. Folate oxidase
E. Folate decarboxylase

88.A patient with hypertensic crisis was admitted to the cardiological department,
he was injected intravenously with an antihypertensive drug – salt of an
alkaline-earth metal. What drug was injected?
A. Potassium chloride
B. Sodium hydrocarbonate
C. *Magnesium sulfate
D. Calcium lactate
E. Benzohexamethonium

89.A patient with acute morphine poisoning was delivered to a hospital. What
specific narcotic antagonist should be chosen in this case?
A. Paracetamol
B. Methacin
C. *Naloxone
D. Digoxin
E. Unithiol

90.A 30-year-old patient complains about having abdominal pain and diarrhea;
body temperature rise up to 37, 5oC along with chills. The day before a patient
had been in a forest and drunk from an open water reservoir. Laboratory
analyses enabled to make the following diagnosis: amebic dysentery. What is
the drug of choice for its treatment?
A. Furazolidonum
B. Levomycetin
C. *Metronidazole
D. Phthalazol
E. Emetine hydrochloride

91.A patient suffering from stomach ulcer has been treated with an antacid drug
almagel. For acute bronchitis treatment he was prescribed the antibiotic
methacycline. However within next 5 days the fever didn’t fall, cough and
sputum nature remained unchanged. A physician came to the conclusion that
the drugs were incompatible. What type of drug incompatibility is the case?
A. Pharmacokinetic, biotransformation stage
B. Pharmaceutic
C. *Pharmacokinetic, absorption stage
D. Pharmacodynamic
E. Direct antagonism

92.Proserin increases skeletal muscle tone when given systematically. Halothane


induces relaxation of skeletal muscles and reduces proserin effects. What is the
nature of proserin and halothane interaction?
A. Direct functional antagonism
B. Competitive antagonism
C. *Indirect functional antagonism
D. Independent antagonism
E. Noncompetitive antagonism

93.A patient suffering from myasthenia has been administered proserin. After its
administration the patient has got nausea, diarrhea, twitch of tongue and
skeletal muscles. What drug would help to eliminate the intoxication?
A. *Atropine sulfate
B. Physostigmine
C. Pyridostigmine bromide
D. Isadrine
E. Mesatonum

94.A patient who had myocardial infarction was administered 75 mg of


acetylsalicinic acid a day. What is the purpose of this administration?
A. Inflammation reduction
B. Pain relief
C. Temperature reduction
D. *Reduction of thrombocyte aggregation
E. Coronary vessel dilatation

95.To prevent the transplant rejection after organ transplantation it is required to


administer hormonotherapy for the purpose of immunosuppression. What
hormones are used for this purpose?
A. Mineralocorticoids
B. Sexual hormones
C. Catecholamines
D. Thyroid
E. *Glucocorticoids

96.A patient suffering from chronic hyperacidic gastritis takes an antacid drug for
heartburn elimination. After its ingestion the patient feels better but at the same
time he has a sensation of stomach swelling. Which of the following drugs
might be the cause of such side effect?
A. Magnesium oxide
B. Magnesium trisilicate
C. Aluminium hydrooxide
D. *Sodium hydrocarbonate
E. Pepsin

97.Continuous use of a certain drug may cause osteoporosis, erosions of stomach


mucosa, hypokaliemia, retention of sodium and water in the organism,
decreased concentration of corticotropin in blood. What drug is it?
A. Hypothiazide
B. *Prednisolone
C. Digoxin
D. Indometacin
E. Reserpine

98.A 63-year-old patient with collapse presentations was delivered to the


emergency hospital. A physician has chosen noradrenalin against hypotension
What is its mechanism of action?
A. Activation of serotonin receptors
B. Activation of beta-adrenoreceptors
C. *Activation of alpha1-adrenoreceptors
D. Activation of dopamine receptors
E. Block of M – cholinoreceptors

99.A patient suffering from syphilis has been treated with bismuth preparations.
As a result of it some grey spots turned up on the mucous membrane of the oral
cavity; nephropathy symptoms were also present. What drug should be used
for treatment of bismuth intoxication?
A. Methylene blue
B. *Unithiol
C. Naloxone
D. Bemegride
E. Nalorphine

100. A patient suffering from coronary artery disease had taken a certain drug
many times a day in order to arrest stenocardia attacks. Overdose of this drug
Þnally caused intoxication. Objectively: cyanotic skin and mucous membranes,
dramatic fall in the arterial pressure, tachycardia, respiration inhibition. Blood
has increased concentration of methemoglobin. The drug the patient had taken
relates to the following group:
A. Alpha-adrenoceptor blockers
B. Calcium channel blockers
C. *Organic nitrates
D. Adenosine drugs
E. Myotropic spamolytics

101. An elderly female patient suffers from the type 2 diabetes mellitus
accompanied by obesity, atherosclerosis, coronary artery disease. Basal
hyperinsulinemia is also present. What treatment would be the most
appropriate?
A. Insulin
B. Retabolil
C. Lovastatin
D. *Glibenclamid
E. Amlodipine

102. In spite of treatment with cardiotonics and thiazide diuretic a patient


suffering from chronic cardiac failure still presents with edemata and faces a
risk of ascites. What medication should be administered in order to increase the
diuretic effect of the above mentioned drugs?
A. *Spironolactone
B. Furosemide
C. Amiloride
D. Clopamide
E. Manithol

103. A patient presents with twilight vision impairment. Which of the


following vitamins should be administered?
A. Cyanocobalamin
B. Pyridoxine hydrochloride
C. *Retinol acetate
D. Ascorbic acid
E. Nicotinic acid

104. A female patient consulted a doctor about pain and limited movements
in the knee joints. Which of the following nonsteroid anti-inflammatory drugs
should be administered taking into consideration that the patient has a history
of chronic gastroduodenitis?
A. Diclofenac sodium
B. Promedol
C. *Celecoxib
D. Acetylsalicilic acid
E. Butadiounum

105. A student came to see a doctor and asked to administer him a drug for
treatment of allergic rhinitis that occurs in the period of linden fowering. What
drug may be used?
A. *Loratadine
B. Noradrenaline hydrotartrate
C. Propanolol
D. Ambroxol
E. Losartan
106. A patient consulted a physician about muscle rigidity, constrained
movements, permanent arm tremor. The patient was diagnosed with
Parkinson’s disease. What preparation should be administered?
A. Phenytoin
B. Phenobarbital
C. *Levodopa
D. Diazepam
E. Ethosuximide

107. A patient with coronary artery disease was admitted to the cardiological
department. For stenocardia prevention a drug from the group of beta-
adrenoceptor blockers was administered. What drug is it?
A. *Metoprolol
B. Atropine sulfate
C. Morphine hydrochloride
D. Oxytocin
E. Furosemide

108. A patient with drug intoxication presented with the dryness of oral
mucous membrane and mydriatic pupils. Such action of this drug is associated
with the following effect:
A. Muscarinic cholinoreceptor stumulation
B. Nicotinic cholinoreceptor stumulation
C. *Muscarinic cholinoreceptor block
D. Adrenoreceptor stimulation
E. Adrenoreceptor block

109. A patient with a limb fracture must be administered a depolarizing drug


from the myorelaxant group for the purpose of a short-time surgery. What drug
is it?
A. *Dithylinum
B. Tubocurarine chloride
C. Cytitonum
D. Atropine sulfate
E. Pentaminum

110. A patient suffering from chronic bronchitis takes a synthetic mucolytic


drug that facilitates the sputum thinning. What drug is it?
A. *Acetylcysteine
B. Diazepam
C. Heparin
D. Furosemide
E. Enalapril
111. Before tooth extraction a patient was advised to take a certain drug for
haemorrhage prevention. What drug was advised?
A. Heparin
B. Asparcam
C. Magnesium sulfate
D. Dimedrol
E. *Vicasolum

112. A 45-year-old patient suffers from neurosis characterized by irritability,


sleeplessness, motiveless anxiety. What drug would eliminate all the
symptoms?
A. *Diazepam
B. Valerian extract
C. Pyracetam
D. Caffeine sodium benzoate
E. Levodopa

113. A patient with myocardial infarction was admitted to the cardiological


department. For pain relief it was decided to potentiate fentanyl action with a
neuroleptic. Which of the following neuroleptics is the most suitable for
neuroleptanalgesia?
A. Aminazine
B. Triftazine
C. *Droperidol
D. Haloperidol
E. Sulpiride

114. Treatment course of bacterial pneumonia included benzylpenicillin


sodium salt. What is the mechanism of its antimicrobial action?
A. Inhibition of the intracellular protein synthesis
B. *Inhibition of cell wall synthesis of the microorganism
C. Inhibition of the cholinesterase activity
D. Inhibition of the SH enzyme groups of the microorganisms
E. Antagonism with the paraaminobenzoic acid

115. A patient with a hypertensive crisis was admitted to the cardiological


department. He was given an intravenous injection of an antihypertensive drug
- alkali-earth metal salt. What drug was injected?
A. Potassium chloride
B. Sodium hydrocarbonate
C. *Magnesium sulfate
D. Calcium lactate
E. Benzohexonium
116. An unconscious patient was delivered to the admission ward.
Objectively: the patient’s skin is cold, pupils are myotic, he has laboured
Cheyne-Stokes respiration, arterial pressure is low, urinary bladder is full.
What substance has caused intoxication?
A. Tranquilizers
B. Non-narcotic analgetics
C. *Narcotic analgetics
D. Muscarinic receptor blockers
E. Loratadine

117. A woman works as railway traffic controller. She suffers from seasonal
vasomotor rhinitis and gets treatment in the outpatient setting. She was
prescribed an antihistamine that has no effect upon central nervous system.
What drug is it?
A. Dimedrol
B. Promethazine
C. Suprastin
D. *Loratadine
E. Tavegil

118. During an operation a patient got injection of muscle relaxant


dithylinum. Relaxation of skeletal muscles and inhibition of respiration lasted
two hours. This condition was caused by absence of the following enzyme in
blood serum:
A. Catalase
B. Acetylcholinesterase
C. *Butyrylcholin esterase
D. Glucose 6-phosphatase
E. Glutathione peroxidase

119. A patient who suffers from insomnia caused by emotional disorder was
prescribed a hypnotic drug with tranquillizing effect. What hypnotic was
prescribed?
A. Phenobarbital
B. Chloral hydrate
C. Sodium ethaminal
D. *Nitrazepam
E. Bromisoval

120. A healthy man is in a region with high risk of catching malaria. What
drug should be administered for individual chemoprophylaxis of malaria?
A. Sulfalen
B. Tetracycline
C. Metronidazole
D. Biseptol
E. *Chingamin

121. A patient suffering from infectious mononucleosis has been taking


glucocorticosteroids for two weeks. This resulted in remission but the patient
got exacerbation of chronic tonsillitis. This complication is induced by the
following effect of glucocorticosteroids:
A. Anti-inflammatory
B. Anti-shock
C. Antiallergenic
D. *Immunosuppressive
E. Antitoxic

122. A 4 year old child was admitted to the orthopaedic department with shin
fracture together with displacement. Bone fragments reposition requires
preliminary analgesia. What preparation should be chosen?
A. Analgin
B. Morphine hydrochloride
C. Panadol
D. *Promedol
E. Loratadine

123. A patient suffers from diabetes melitus. After the regular insulin
injection his condition grew worse: there appeared anxiety, cold sweat, tremor
of limbs, general weakness, dizziness. What preparation can eliminate these
symptoms?
A. Butamide
B. Caffeine
C. *Adrenaline hydrochloride
D. Noradrenaline
E. Glibutide

124. A 19 year old woman suffers from primary syphilis. Doctor administered
her complex therapy that includes benzylpenicillin sodium salt. What is the
mechanism of action of this drug?
A. It blocks synthesis of cytoplasm proteins
B. It blocks thiol enzymes
C. It blocks RNA synthesis
D. It blocks DNA synthesis
E. *It blocks synthesis of peptidoglycan of microbal membrane

125. A 65 year old female patient suffers from chronic renal insufficiency
accompanied by evident edemata caused by chronic glomerulonephritis. What
diuretic should be administered for forced diuresis
A. Hydrochlorothiazide
B. Chlorthalidone
C. Cyclometazide
D. *Furosemide
E. Acetazolamide

126. A patient suffers from stenocardia and takes isosorbide mononitrate. He


was prescribed a complementary drug with disaggregating effect. What drug is
it?
A. Nitroglycerine
B. Propranolol
C. Nifedipine
D. *Acetylsalicinic acid
E. Validol

127. A patient in postoperative period was prescribed an anticholinesterase


drug for stimulation of intestinal peristalsis and tonus of urinary bladder. What
drug is it?
A. *Proserin
B. Dichlothiazide
C. Reserpine
D. Mannitol
E. Propanolol

128. A 66 year old female patient got intravenous injection of magnesium


sulfate solution for the purpose of elimination of hypertensive crisis. But
arterial pressure didn’t go down and after repeated introduction of the same
preparation there appered sluggishness, slow response, inhibition of
consciousness and respiration. What preparation is antagonist of magnesium
sulfate and can eliminate symptoms of its overdose?
A. *Calcium chloride
B. Potassium chloride
C. Sodium chloride
D. Activated carbon
E. Potassium permanganate

129. A 64 year old woman has impairment of twilight vision (hemeralopy).


What vitamin should be recommended in the first place?
A. *Vitamin A
B. Vitamin B2
C. Vitamin E
D. Vitamin C
E. Vitamin B6

130. A patient underwent appendectomy. In the postoperative period he has


been taking an antibiotic. The patient complains about hearing impairment and
vestibular disorders. What group of antibiotics has such by effects?
A. Penicillins
B. Tetracyclines
C. Macrolides
D. Cephalosporins
E. *Aminoglycosides

131. A patient complained about muscle rigidity, constrained motions,


constant tremor of arms. On the grounds of examination his disease was
diagnosed as Parkinson’s disease. What drug should be administered?
A. *Levodopa
B. Diphenylhydantoin
C. Phenobarbital
D. Diazepam
E. Ethosuximide

132. A woman was delivered to a hospital for trachea intubation. What of the
following drugs should be applied in this case?
A. Nitroglycerine
B. Metronidazole
C. *Dithylinum
D. Atropine sulfate
E. Gentamycin sulfate

133. A patient suffers from pulmonary tuberculosis. During treatment neuritis


of visual nerve arose. What drug has caused this by effect?
A. Ethambutol
B. Kanamycin
C. Rifampicin
D. Streptomycin
E. *Isoniazid

134. A patient suffers from chronic left-ventricular insufficiency. What


medication should be administered?
A. *Digoxin
B. Bemegride
C. Etimizole
D. Vinpocetine
E. Pyracetam

135. Burned skin surface was treated with a certain preparation. Its antiseptic
properties are provided by atomic oxygen that is formed in presence of organic
substances. What preparation was applied?
A. Furacillin
B. Chlorhexidine bigluconate
C. Alcoholic iodine solution
D. *Potassium permanganate
E. Sodium hydrocarbonate

136. A patient with chronic cardiac insufficiency has been taking foxglove
(Digitalis) preparations for a long time. Due to the violation of intake schedule
the woman got symptoms of intoxication. These symptoms result from:
A. Tachyphylaxis
B. Idiosyncrasy
C. Antagonism
D. Sensibilization
E. *Material cumulation

137. A patient has herpetic rash. What medication should be administered?


A. Gentamycin
B. Clotrimazole
C. Benzylpenicillin sodium salt
D. *Acyclovir
E. Biseptol

138. A patient with hip fracture was prescribed a narcotic analgetic. Its
anesthetic action is determined by interaction with the following receptors:
A. Adrenoreceptors
B. Cholinoreceptors
C. *Opiate receptors
D. Benzodiazepine receptors
E. GABA-ergic receptors

139. A 5-year-old child has been diagnosed with acute right distal pneumonia.
Sputum inoculation revealed that the causative agent is resistant to penicillin,
but it is senstive to macrolides. What drug should be prescribed?
A. Tetracycline
B. Gentamycin
C. Streptomycin
D. Ampicillin
E. *Azithromycin

140. A patient with essential hypertension has a high rate of blood renin.
Which of antihypertensive drugs should be preferred?
A. Propranolol
B. Prazosinum
C. *Lisinopril
D. Nifedipine
E. Dichlothiazide
141. A patient with cardiogenic shock, hypotension, asphyxia and edemata
was given an injection of non-glycosidic cardiotonic. What drug was injected?
A. Caffeine sodium benzoate
B. Cordiamin
C. Aethimizolum
D. *Dobutamine
E. Bemegride

142. Which of the listed diuretic agents WILL NOT have diuretic effect on a
patient with Addison’s disease?
A. Furosemide
B. Hydrochlorothiazide
C. Triamterene
D. *Spironolactone
E. Ethacrynic acid

143. This drug has a destructive effect on erythrocytic forms of malarial


plasmodia and dysenteric amoebae. It is used for treatment and prevention of
such diseases as malaria, amebiasis and interstitial disease. What drug is it?
A. *Chingamin
B. Emetine hydrochloride
C. Tetracycline
D. Erythromycin
E. Quinine

144. A child suffers from drug idiosyncrasy. What is the cause of such
reaction?
A. Exhaustion of substrate interacting with pharmaceutical substance
B. Accumulation of pharmaceutical substance
C. *Hereditary enzymopathy
D. Inhibition of microsomal liver enzymes
E. Associated disease of target organ

145. Examination of a 70-year-old patient revealed insulin-dependent


diabetes. What drug should be administered?
A. *Glibenclamid
B. Insulin
C. Mercazolilum
D. Parathyroidin
E. Cortisone

146. A patient suffers from chronic left ventricular insufficiency.What


medication should be administered?
A. Bemegride
B. Etimizole
C. Vinpocetine
D. Pyracetam
E. *Digoxin

147. A patient with acute myocardial infarction has been administered


heparin as a part of complex therapy. Some time after heparin injection the
patient developed hematuria. What heparin antagonist should be injected in
order to manage the complication?
A. Vicasol
B. Aminocaproic acid
C. *Protamine sulfate
D. Neodicumarin
E. Fibrinogen

148. A patient with coronary disease and arrhythmia has been administered a
drug that blocks potassium channels and prolongs the action potential. What
drug is it?
A. Corglyconum
B. *Amiodarone
C. Nitroglycerin
D. Dobutamine
E. Lisinopril

149. A patient takes digoxin for cardiac insufficiency. What diuretic may
increase digoxin toxicity due to the intensified excretion of K+ ions?
A. Spironolactone
B. Panangine
C. Siliborum
D. *Hydrochlorothiazide
E. Lisinopril

150. A female patient consulted a doctor about a sense of epigastric


discomfort, nausea and anorexia. A duodenal content analysis revealed
lamblia. What drug should be prescribed?
A. Chingamin
B. Rifampicin
C. Isoniazid
D. Acyclovir
E. *Metronidazole

151. During anesthetization a patient presented with symptoms of tonus


increase of parasympathetic nervous system such as hypersalivation and
laryngospasm. What drug could have prevented these undesirable effects?
A. Adrenaline hydrochloride
B. *Atropine sulphate
C. Neostigmine
D. Analgin
E. Pyracetam

152. A female patient was administered loratadine for allergic cheilitis


treatment. What is the mechanism of the drug’s action?
A. It blocks the adrenergic receptors
B. *It blocks the activity of H1 histamine receptors
C. It stimulates the activity of monoamineoxidase
D. It inhibits the activity of Na, K-ATP
E. It inhibits the activity of choline esterase

153. A doctor prescribed a cephalosporin antibiotic to the patient after


appendectomy for infection prevention. Antimicrobial activity of this group of
antibiotics is based upon the disturbance of the following process:
A. Nucleic acid synthesis
B. Ribosome protein synthesis
C. *Microbial wall formation
D. Energy metabolism
E. Choline esterase block

154. An alcoholic has alcoholic psychosis with evident psychomotor


agitation.What neuroleptic drug should be administered for emergency care?
A. *Aminazine
B. Diazepam
C. Sodium bromide
D. Reserpine
E. Halothane

155. Examination of a child who hasn’t got fresh fruit and vegetables during
winter revealed numerous subcutaneous hemorrhages, gingivitis, carious
cavities in teeth. What vitamin combination should be prescribed in this case?
A. Thiamine and pyridoxine
B. Folic acid and cobalamin
C. *Ascorbic acid and rutin
D. Riboflavin and nicotinamide
E. Calciferol and ascorbic acid

156. Parodontitis is treated with calcium preparations and a hormone that


stimulates tooth mineralization and inhibits tissue resorption. What hormone is
it?
A. Parathormone
B. Adrenalin
C. Aldosterone
D. Thyroxine
E. *Calcitonin

157. A 5 year old child has been diagnosed with acute right distal pneumonia.
Sputum inoculation revealed that the causative agent is resistant to penicillin,
but it is senstive to macrolides.What drug should be prescribed?
A. Tetracycline
B. Gentamycin
C. *Azithromycin
D. Streptomycin
E. Ampicillin

158. A 63 year old patient with collapse presentations was delivered to the
emergency hospital. A physician has chosen noradrenalin against hypotension.
What is its mechanism of action?
A. Activation of serotonin receptors
B. Activation of beta-adrenoreceptors
C. *Activation of alpha1-adrenoreceptors
D. Activation of dopamine receptors
E. Block of M-cholinoreceptors

159. A 65 year old patient with chronic heart failure has been taking digitoxin
in self-administered dosages for a long time. She was admitted to the hospital
for general health aggravation, arrhythmia, nausea, reduced diuresis, insomnia.
What is the primary action to be taken?
A. To reduce digitoxin dosage
B. To administer strophanthine intravenously
C. To administer digoxin
D. *To with hold digitoxin
E. To give an intravenous injection of calcium gluconate solution

160. A 20 year old patient complains of morbid thirst and huperdiuresis (up to
10 l daily). Glucose concentration in blood is normal but it is absent in urine.
The patient has been diagnosed with diabetes insipidus. What hormonal drug is
the most appropriate for management of this disorder?
A. Cortisol
B. Thyroxin
C. *Vasopressin
D. Oxytocin
E. Insulin

161. A patient has myocardial infarction with thrombosis of the left coronary
artery.What pharmocological preparation group should be used to reestablish
blood flow?
A. Narcotic analgesics
B. Beta-adrenergic blockers
C. Angiotensin-converting enzyme inhibitors
D. Glucocorticoids
E. *Fibrinolysis activators

162. A nurse accidentally injected a nearly double dose of insulin to a patient


with diabetes mellitus. The patient lapsed into a hypoglycemic coma. What
drug should be injected in order to help him out of coma?
A. Lidase
B. Insulin
C. *Glucose
D. Somatotropin
E. Noradrenaline

163. A patient has a slowly healing fracture. What medicine can be used to
accelerate formation of connective tissue matrix?
A. Prednisolone
B. Cyclophosphan
C. *Methyluracil
D. Methotrexate
E. Cyclosporine

164. While under barbituric anaesthesia a 65 year old male patient developed
respiratory inhibition. Anesthesiologist made him a 10 ml intravenous injection
of 0,5 % bemegride solution. The patient’s condition got better, the pulmonary
ventilation volume increased. What phenomenon underlies the interaction of
these medivcations?
A. Indirect antagonism
B. *Direct antagonism
C. Unilateral antagonism
D. Direct synergism
E. Indirect synergism

165. After the second abortion a 23 year old woman has been diagnosed with
toxoplasmosis.Which drug should be used for toxoplasmosis treatment?
A. Itraconazole
B. Mebendazole
C. Azidothimidine
D. *Co-trimoxazole
E. Acyclovir

166. After 4 months of treatment for tuberculosis the patient began


complaining of toes and fingers numbness, sensation of creeps. He was
diagnosed with polyneuritis. What antituberculous drug might have caused
these complications?
A. Rifampicin
B. Ciprofloxacin
C. Sodium salt of benzylpenicillin
D. *Isoniazid
E. Alcohol iodine solution

167. An elderly patient has chronic constipations induced by large intestine


hypotonia. What drug should be administered?
A. Sodium sulphate
B. Castor oil
C. *Bisacodyl
D. Atropine
E. Novocaine amide

168. A patient with epilepsy and depressive reaction has been administered a
drug that reduced epilepsy manifestations and improved the patient’s psychic
condition.
A. Ethosuxemide
B. Amitriptyline
C. Phenytoin
D. Phenobarbital
E. *Sodium valproate

169. A patient diagnosed with morphinism has been admitted to the


narcological department. A doctor noted a decrease in pharmacological activity
of morphine. Repetitive use of a drug may result in tolerance to its effect, and
this phenomenon is called:
A. Cumulation
B. Tachyphylaxis
C. *Addiction
D. Antagonism
E. Allergy

170. As a result of durative antibiotic therapy a 37 year old patient developed


intestinal dysbacteriosis. What type of drugs should be used in order to
normalize intestinal microflora?
A. Sulfanilamides
B. Bacteriophages
C. Autovaccines
D. *Eubiotics
E. Vitamins

171. An oncological patient had been administered methotrexate. With time


target cells of the tumour lost sensitivity to this drug. At the same time the
change in gene expression of the following enzyme is observed:
A. Thiaminase
B. Deaminase
C. *Dehydropholate reductase
D. Pholate oxidase
E. Pholate decarboxylase

172. A patient consulted a dentist about itching and burning in the oral cavity;
high temperature. The patient was diagnosed with trichomonal
gingivostomatitis. What drug should be chosen for his treatment?
A. Ampicillin
B. *Metronidazole
C. Doxycycline hydrochloride
D. Gentamicin sulfate
E. Nystatin

173. A patient has been diagnosed with transmural myocardial infarction.


What drug should be given in order to prevent cardiogenic shock?
A. Reserpin
B. Octadine
C. *Promedol
D. Phentolamine
E. Analgin

174. A patient with ischemic heart disease has been administered an anti-
anginal drug that reduces the myocardial oxygen consumption and improves
blood supply of myocardium. What drug is it?
A. Validol
B. Propranolol
C. *Nitroglycerine
D. Promedol
E. Retabolil

175. For relief of hypertensive crisis a doctor administered a patient a drug


that apart from antihypertensive effect has also sedative, spasmolytic and
anticonvulsive effect. The drug was taken parenterally. When it is taken
enterally it acts as a laxative and cholagogue. What drug was administered?
A. Dibasolum
B. Reserpine
C. *Magnesium sulfate
D. No-spa
E. Apressin

176. A patient with diabetes mellitus complicated by angiopathy has been


recommended a drug which is a sulphonyl urease derivate of the second
generation. It improves microcirculation and is known for its relatively good
tolerance. What drug is it?
A. Glibutidum
B. Insulin
C. *Glibenclamide
D. Acarbose
E. Adrenalin

177. Pharmacological effects of antidepressants are based upon blocking


(inhibiting) the enzyme that acts as a catalyst for the breakdown of biogenic
amines noradrenalin and serotonin in the mitochondria of cephalic neurons.
What enzyme takes part in this process?
A. Transaminase
B. *Monoamine oxidase
C. Decarboxylase
D. Peptidase
E. Lyase

178. A patient diagnosed with focal tuberculosis of the upper lobe of the right
lung had been taking isoniazid as a part of combination therapy. After some
time, the patient reported of muscle weakness, decreased skin sensitivity,
blurred vision, impaired motor coordination. Which vitamin preparation should
be used to address these phenomena?
A. Vitamin A
B. Vitamin D
C. *Vitamin B6
D. Vitamin B12
E. Vitamin C

179. A 60-year-old male patient has a 9-year history of diabetes and takes
insulin Semilente for the correction of hyperglycemia. 10 days ago he began
taking anaprilin for hypertension. One hour after administration of the
antihypertensive drug the patient developed hypoglycemic coma. What is the
mechanism of hypoglycemia in case of anaprilin use?
A. Reduction of glucagon half-life
B. Increase of insulin Semilente half-life
C. *Inhibition of glycogenolysis
D. Increase of bioavailability of insulin Semilente
E. Decrease in glucose absorption

180. A 60-year-old patient with a long history of stenocardia takes


coronarodilator agents. He has also been administered acetylsalicylic acid to
reduce platelet aggregation. What is the mechanism of antiplatelet action of
acetylsalicylic acid?
A. *It reduces the activity of cyclooxygenase
B. It reduces the activity of phosphodiesterase
C. It enhances the activity of platelet adenylate cyclase
D. It enhances the synthesis of prostacyclin
E. It has membrane stabilizing effect

181. A girl receives antibiotics of the penicillin group for acute bronchitis. On
the third day of treatment she developed allergic dermatitis. Which drug should
be administered?
A. Cromolyn sodium
B. *Loratadine
C. Beclomethasone
D. Ephedrine hydrochloride
E. Levamisole

182. A 42-year-old male patient with gout has an increased blood uric acid
concentration. In order to reduce the level of uric acid the doctor administered
him allopurinol. Allopurinol is the competitive inhibitor of the following
enzyme:
A. Adenosine deaminase
B. *Xanthine oxidase
C. Adenine phosphoribosyltransferase
D. Hypoxanthinephosphoribosyltransferase
E. Guanine deaminase

183. A 40-year-old female patient diagnosed with acute pancreatitis has been
delivered to the admission department of a regional hospital. What drug should
be administered the patient in the first place?
A. Platyphyllin
B. Atropine
C. *Contrycal
D. Metacin
E. Pirenzepine

184. A female patient with bronchial asthma had taken prednisolone tablets
(1 tablet times a day) for 2 months. Due to a significant improvement of her
condition the patient suddenly stopped taking it. What complication is likely to
develop in this case?
A. Cushing’s syndrome
B. Gastrorrhagia
C. Upper body obesity
D. Hypotension
E. *Withdrawal syndrome

185. During a surgery with the use of hygronium the patient had an abrupt
fall in blood pressure. Blood pressure can be normalized by the representatives
of the following drug group:
A. Alpha-blockers
B. *Alpha-adrenergic agonists
C. Ganglionic blockers
D. M-cholinergic agents
E. N-cholinergic agents

186. A patient with biliary dyskinesia and constipations has been prescribed a
cholagogue having also a laxative effect. What drug has been administered?
A. Allochol
B. Cholosas
C. Cholenzyme
D. *Magnesium sulfate
E. Nicodinum

187. It is known that individuals with genetically caused deficiency of


glucose-6-phosphate dehydrogenase may develop RBC hemolysis in response
to the administration of some antimalarial drugs. Manifestation of adverse
reactions to drugs is called:
A. Allergy
B. *Idiosyncrasy
C. Sensibilization
D. Tachyphylaxis
E. Tolerance

188. Curari form substances introduced into a human body cause the
relaxation of all skeletal muscles. What changes in the neuromuscular synapse
cause this phenomenon?
A. Impaired acetylcholine release
B. *Blockade of N-cholinergic receptors of the synaptic membrane
C. Blockade of Ca2+ channels of the presynaptic membrane
D. Impaired cholinesterase synthesis
E. Depolarization of the postsynaptic membrane

189. A comatose patient was taken to the hospital. He has a history of


diabetes mellitus. Objectively: Kussmaul breathing, low blood pressure,
acetone odor of breath. After the emergency treatment the patient’s condition
improved. What drug had been administered to the patient?
A. Adrenaline
B. *Insulin
C. Isadrinum
D. Glibenclamide
E. Furosemide

190. In order to stimulate breathing in a child born with asphyxia, the doctor
gave him a drug injection into the umbilical vein. What drug might have been
injected?
A. Corazolum
B. *Aethimizolum
C. Cordiaminum
D. Sulfocamphocainum
E. Coffeinum

191. To prevent attacks of acute pancreatitis a doctor prescribed the patient


trasylol (contrycal, gordox), which is an inhibitor of:
A. Elastase
B. *Trypsin
C. Carboxypeptidase
D. Chymotrypsin
E. Gastricsin

192. A patient with urolithiasis has unbearable spasmodic pain. To prevent


pain shock, he has been given an injection of atropine along with a narcotic
analgesic having antispasmodic effect. What drug was it?
A. Nalorphine
B. *Promedol
C. Tramadol
D. Ethylmorphine hydrochloride
E. Morphine hydrochloride

193. Despite the administration of cardiotonics and a thiazide diuretic a


patient with chronic heart failure has persistent edemata, there is a risk of
ascites. What medication should be administered in order to enhance the
diuretic effect of the drugs used?
A. Furosemide
B. *Spironolactone
C. Amiloride
D. Clopamide
E. Manithol

194. A 66-year-old patient with Parkinson’s disease shows an improvement in


locomotor activity after prolonged use of a certain drug which is converted to
dopamine by the decarboxylation. What drug has the patient taken?
A. Naloxone
B. Celecoxib
C. *Levodopa
D. Droperidol
E. Chlorpromazine

195. A 46-year-old female is scheduled for a maxillofacial surgery. It is


known that the patient is prone to high blood coagulation. What natural
anticoagulant can be used to prevent blood clotting?
A. Hirudin
B. *Heparin
C. Sodium citrate
D. Fibrinolysin
E. None of the above-listed substances

196. A patient has been administered an anti-inflammatory drug that blocks


the action of cyclooxygenase. Specify this anti-inflammatory agent:
A. Analgene
B. Allopurinol
C. *Aspirin
D. Thiamin
E. Creatine

197. A 26-year-old female patient with bronchitis has been administered a


broad spectrum antibiotic as a causal treatment drug. Specify this drug:
A. *Doxycycline
B. Interferon
C. BCG vaccine
D. Ambroxol
E. Dexamethasone

198. A 12-year-old child has a viral infection complicated by obstructive


bronchitis. Bronchospasm can be eliminated by inhalations of a drug from the
following pharmacological group:
A. M-anticholinergics
B. N-cholinomimetics
C. *Beta-2-agonists
D. Beta-adrenergic blockers
E. Analeptics

199. A 26-year-old woman at 40 weeks pregnant has been delivered to the


maternity ward. Objectively: the uterine cervix is opened, but the contractions
are absent. The doctor has administered her a hormonal drug to stimulate the
labor. Name this drug:
A. *Oxytocin
B. Hydrocortisone
C. Estrone
D. Testosterone
E. ACTH

200. A patient has recurrent attacks of epileptic seizures and stays


unconscious between them. In order to stop convulsions the drugs of the
following group should be used in the first place:
A. *Tranquilizers
B. Neuroleptics
C. Muscle relaxants
D. Sedatives
E. Analeptics

201. A patient with arthritis and varicose veins has been taking a non-
steroidal anti-inflammatory drug for a long time, which caused thrombosis of
skin veins. Which of the following drugs might have caused this complication?
A. Indomethacin
B. Aspirin
C. Phenylbutazone
D. *Celecoxib
E. Ibuprofen

202. A patient has arterial hypertension. What long-acting drug from the
group of calcium channel blockers should be prescribed?
A. *Amlodipine
B. Octadine
C. Pyrroxanum
D. Atenolol
E. Reserpine

203. A 65-year-old female patient has chronic constipations due to the colon
hypotonia. What drug should be chosen in this case?
A. *Bisacodyl
B. Castor oil
C. Magnesium sulfate
D. Neostigmine methylsulfate
E. Metoclopramide

204. Administration of doxycycline hydrochloride caused an imbalance of the


symbiotic intestinal microflora. Specify the kind of imbalance caused by the
antibiotic therapy:
A. *Dysbacteriosis
B. Sensibilization
C. Idiosyncrasy
D. Superimposed infection
E. Bacteriosis

205. Ascarid eggs have been detected during stool analysis. What drug
should be prescribed?
A. Nystatin
B. Chloramphenicol
C. *Mebendazole
D. Tetracycline
E. Furazolidone

206. A patient with chronic heart failure with edema has increased level of
blood aldosterone. What diuretic would be most effective in this case?
A. Triamterene
B. Acetazolamide
C. Hydrochlorothiazide
D. Furosemide
E. *Spironolactone

207. What drug will be most appropriate for the patient who has chronic
gastritis with increased secretion?
A. Pancreatine
B. *Pirenzepine
C. Pepsin
D. Aprotinin
E. Chlorphentermine

208. A child was born asphyxiated. What drug must be administered to the
newborn to stimulate breathing?
A. Lobeline
B. *Aethimizolum
C. Prazosin
D. Atropine
E. Proserine

209. A patient complaining of rapid pulse, dyspnea and bluish color of


mucosa has been admitted to the cardiological department. The objective
symptoms are as follows: edema of lower extremities, ascites. Which of the
given medicines should be prescribed for intravenous administration to
improve the patient’s general state?
A. Cordiamin
B. Adrenalin hydrochloride
C. Digitoxin
D. *Corglyconum
E. Drotaverine

210. A child patient has dry cough. What non-narcotic antitussive drug will
relieve the patient’s condition?
A. Codeine phosphate
B. Morphine hydrochloride
C. Potassium iodide
D. Althaea officinalis root extract
E. *Glaucine hydrochloride
211. An patient with insomnia induced by allergic rash and itch has been
prescribed the drug that has antihistamine and hypnotic effect. Specify this
drug:
A. Loratadine
B. Prednisolone
C. *Benadryl
D. Acetylsalicylic acid
E. Analgin

212. During local anesthetization the patient has gone into anaphylactic
shock. What drug must be administered to the patient?
A. Diazepam
B. Atropine sulfate
C. *Epinephrine hydrochloride
D. Propranolol
E. Nitroglycerin

213. A patient who had been taking diclofenac sodium for arthritis of
mandibular joint developed an acute condition of gastric ulcer. Such side effect
of this medicine is caused by inhibition of the following enzyme:
A. Cyclooxygenase-2 (COX-2)
B. Lipoxygenase
C. Phosphodiesterase
D. *Cyclooxygenase-1 (COX-1)
E. Monoamine oxidase

214. A 55-year-old male had been delivered to the resuscitation unit


unconscious. Relatives reported him to have mistakenly drunk an alcoholic
solution of unknown origin. On examination the patient was diagnosed with
methanol intoxication. What antidote should be used in this case?
A. Teturamum
B. *Ethanol
C. Naloxone
D. Protamine sulfate
E. Acetylcysteine

215. An 8-week-pregnant woman with acute respiratory disease and


temperature rise up to 39,0oC has called in a doctor. The doctor insisted on her
avoiding taking paracetamol, because in this period of pregnancy there is a risk
of its:
A. *Teratogenicity
B. Embryotoxicity
C. Fetotoxicity
D. Hepatotoxicity
E. Allergenicity
216. A pneumonia patient has been administered acetylcysteine as a part of
complex therapy. What principle of therapy has been taken into consideration
when applying this drug?
A. Symptomatic
B. Etiotropic
C. Antimicrobial
D. Immunomodulatory
E. *Pathogenetic

217. A 68-year-old patient consults a cardiologist, complaining of high


arterial blood pressure, pain in the heart egion, intermittent pulse. Prescribe the
beta-1-adrenoreceptor blocker for the treatment of the described pathology:
A. Morphine hydrochloride
B. Nootropil
C. *Metoprolol
D. Fenoterol
E. Benzylpenicillin

218. A patient is diagnosed with acute morphine hydrochloride intoxication.


Prescribe an oxidizing agent for gastric lavage:
A. *Potassium permanganate
B. Chloramine
C. Sulfocamphocainum (Procaine + Sulfocamphoric acid)
D. Cerigel
E. Chlorhexidine (bi)gluconate

219. A 35-year-old female patient has been hospitalised with acute


intoxication caused by salts of high-density metals (lead,most probably).As a
part of complex therapy the antidote that contains two active sulfhydric groups
has been prescribed. Specify this antidote:
A. Metamizole
B. Mannitol
C. *Dimercaprol
D. Nalorphine hydrochloride
E. Calcium chloride

220. A 60-year-old male patient has type II diabetes. A doctor has prescribed
him synthetic hypoglycemic long-acting drug that is sulfonylurea derivative.
What drug is it?
A. Butamide
B. Metformin
C. Actrapid (soluble insulin)
D. *Glibenclamide
E. Acarbose
221. A patient has been given atropine sulfate for rapid relief of spastic colon
symptoms. The use of this drug is contraindicated during the following
disease:
A. Bronchial asthma
B. Bradycardia
C. Hypotension
D. Gastric ulcer
E. *Glaucoma

222. A 63-year-old male patient with bladder atony had been prescribed a
medication, which he had been arbitrarily taking at a higher dose. The patient
developed hypehydration, salivation, diarrhea, muscle spasms. The prescribed
drug relates to the following group:
A. Cholinesterase reactivators
B. Adrenergc blockers
C. Tocolytics
D. *Cholinomimetics
E. Ganglionic blockers

223. A patient complains of photoreception disorder and frequent acute viral


diseases. He has been prescribed a vitamin that affects photoreception
processes by producing rhodopsin, the photosensitive pigment. What vitamin is
it?
A. Tocopherol acetate
B. Pyridoxine hydrochloride
C. Cyanocobalamin
D. *Retinol acetate
E. Thiamine

224. A patient with acne has been prescribed doxycycline hydrochloride.


What recommendations should be given to the patient, while he is taking this
drug?
A. Take with large quantity of liquid, preferably milk
B. Take before meal
C. *Avoid long stay in the sun
D. The course of treatment should not exceed 1 day
E. Do not take with vitamins

225. A 30-year-old patient with a past history of virus B hepatitis complains


of prolonged nosebleeds. What drug will be most efficient in remedying this
condition?
A. *Vicasolum
B. Fraxiparine
C. Folic acid
D. Dipiridamol
E. Asparcam

226. In cancer patients who have been continuously receiving methotrexate,


the target cells of tumor with time become insensitive to this drug. In this case,
gene amplification of the following enzyme is observed:
A. *Dihydrofolate reductase
B. Thiaminase
C. Deaminase
D. Thioredoxin reductase
E. Octadine

227. A patient with signs of emotional lability that result in troubled sleep has
been prescribed nitrazepam. Specify the sleep-inducing mechanism of this
drug:
A. Blockade of opiate receptors
B. Inhibition of stimulating amino acids
C. H1-histamine receptors stimulation
D. Supression of serotonergic neurotransmission
E. *GABA-ergic system activation

228. After implantation of a cardiac valve a young man systematically takes


indirect anticoagulants. His state was complicated by hemorrhage.What
substance content has decreased in blood?
A. Haptoglobin
B. Heparin
C. Creatin
D. *Prothrombin
E. Ceruloplasmin

229. A 4-year-old child was admitted to an orthopaedic department with


displaced shin fracture. Bone fragments reposition requires analgesia. What
drug should be chosen?
A. Analgin
B. *Promedol
C. Morphine hydrochloride
D. Panadol
E. Gentamycin

230. Monoamine oxidase inhibitors are widely used as


psychopharmacological drugs. They change the level of nearly all
neurotransmitters in synapses, with the following neurotransmitter being the
exception:
A. Noradrenaline
B. Adrenaline
C. *Acetylcholine
D. Dopamine
E. Serotonin

231. A patient with urolithiasis has developed severe pain attacks. For pain
shock prevention he was administered an antispasmodic narcotic analgesic
along with atropine. Name this drug:
A. *Promedol
B. Nalorphine
C. Tramadol
D. Ethylmorphine hydrochloride
E. Morphine hydrochloride

232. Despite the administration of cardiotonics and thiazide diuretic a patient


with chronic heart failure has persistent edemas and the risk of ascites
arose.What medication should be administered to enhance the diuretic effect of
the administered drugs?
A. Furosemide
B. Amiloride
C. *Spironolactone
D. Clopamide
E. Manithol

233. A 66-year-old woman had intravenous injection of magnesium sulfate


solution to stop hypertensive crisis. However her arterial pressure did not
decrease and after repeated introduction of the same preparation she developed
sluggishness, slow response to stimuli; the patient is unconsciousness and her
respiration is inhibited. What preparation is antagonist of magnesium sulfate
and can remove the symptoms of its overdose?
A. *Calcium chloride
B. Potassium chloride
C. Sodium chloride
D. Activated carbon
E. Potassium permanganate

234. A patient was prescribed loratadine to treat allergic cheilitis. What is the
mechanism of action of this drug?
A. *Blockade of H1-histamine receptors
B. Blockade of adrenergic receptors
C. Increases activty of monoamine oxidase
D. Suppresses activity of Na+/K+-ATPase
E. Suppresses activity of choline esterase

235. During introduction of local anesthesia a patient has gone into


anaphylactic shock. What drug must be administered to the patient?
A. *Epinephrine hydrochloride
B. Diazepam
C. Atropine sulfate
D. Propranolol
E. Nitroglycerin

236. When treating a patient with chronic cardiac failure a doctor detected
bradycardia and deterioration of the patient’s general state. Such condition is
caused by cumulative effect of a drug. Which drug of those listed below has
cumulative action?
A. *Digoxin
B. Diphenhydramine (Dimedrol)
C. Hydrochlorothiazide
D. Isosorbide
E. Retinol acetate

237. A patient has developed paroxysmal ventricular tachycardia against the


background of cardiac infarction.What antiarrhythmic drug should be chosen
to avoid lowering cardiac output?
A. Procainamide
B. Verapamil
C. Propranolol
D. Potassium chloride
E. *Lidocaine hydrochloride

238. Prolonged treatment of hypothyroidism has caused general dystrophy,


dental caries, tachycardia, tremor of extremities. What drug is the cause of
these side effects?
A. Humulin (Human insulin)
B. *L-thyroxin
C. Parathyreoidinum
D. Thyrocalcitonin
E. Prednisolone

239. Name the halogen-containing antiseptic with fungicidal properties,


which is used to treat dermatomycosis:
A. Formalin solution
B. Methylene blue
C. *Iodine solution
D. Brilliant green
E. Boric acid solution

240. Due to severe pain syndrome a patient was prescribed a narcotic


analgesic. Name this drug:
A. Metamizole (Analgin)
B. *Morphine
C. Nimesulide
D. Dimethyl sulfoxide
E. Indometacin

241. To treat rheumatoid arthritis a 65-year-old woman was prescribed an


immunosuppressive hormonal drug as a part of her complex therapy. Name
this drug:
A. Thymus cytomedins (Thymalin)
B. *Prednisolone
C. Chloropyramine (Suprastin)
D. Riboflavin
E. Fercovenum

242. During narcosis a patient developed a risk of cerebral edema.What drug


should be administered in this case?
A. Dopamine
B. Phenazepam
C. *Furosemide
D. Triamterene
E. Sodium bromide

243. Name the drug that inhibits excretory function of pancreas during
treatment of acute pancreatitis:
A. Allochol
B. Раnzynorm
C. *Contrykal (Aprotinin)
D. Pancreatin (Mezym forte)
E. Festal

244. An 18-year-old patient has developed candidiasis after the case of


pneumonia treated with beta-lactam antibiotic. What antimycotic agent should
be prescribed?
A. Streptomycin
B. Ampicillin
C. *Fluconazole
D. Phthalylsulfathiazole
E. Trimethoprim/sulfamethoxazole (Biseptol)

245. A woman poisoned with unknown substance was hospitalised in a


toxicological department. What group of drugs can be administered to decrease
absorption and introduction of the poison to her body?
A. Neuroleptics
B. Antioxidants
C. *Adsorbents
D. Organic nitrates
E. Cholinesterase inhibitors

246. A patient after disrupted cerebral circulation has developed paralysis.


Choose the anticholinesterase drug to be prescribed in this case:
A. Cordiamin
B. Aceclidine
C. *Proserin
D. Methacin
E. Hexamethonium (Benzohexonium)

247. A patient is diagnosed with diabetic coma. Blood sugar is 18,44 mmol/l.
What glucose-regulating drug should be prescribed in the given case?
A. Intermediate-acting insulin
B. Long-acting insulin
C. *Rapid-acting insulin
D. Biguanide
E. Sulfonylurea derivative

248. A patient complaining of dizziness, thirst, difficult swallowing, and


impaired vision of close objects has addressed a doctor. Objectively:
respiratory rate is increased, pupils are dilated, general agitation, talkativeness,
though the speech is indistinct. BP is 110/70 mm Hg, heart rate is 110/min.
Given symptoms can indicate overdosage of the following drug:
A. Morphine
B. *Atropine
C. Ephedrine
D. Aminazine
E. Caffeine

249. A 4-year-old child has been admitted to an orthopaedic department with


displaced shin fracture. Bone fragments reposition requires analgesia. What
drug should be chosen?
A. Analgin
B. *Promedol
C. Morphine hydrochloride
D. Panadol
E. Caffeine

250. What drug will be the most appropriate for a patient suffering from
chronic gastritis with increased secretion?
A. Pancreatine
B. Pepsin
C. *Pirenzepine
D. Aprotinin
E. Chlorphentermine

251. A patient has been diagnosed with gonorrhea. As fluoroquinolones are


the drugs of choce for treatment of gonorrhea the patient should be prescribed:
A. Furazolidone
B. Fluorouracil
C. Sulfacarbamide (Urosulfanum)
D. *Ciprofloxacin
E. Cefazolin

252. A 40-year-old patient suffers from bronchial asthma and prolonged


tachycardia. Choose the optimal drug for rapid relief of bronchial spasm in the
given case:
A. Adrenalin hydrochloride
B. Ephedrine hydrochloride
C. *Salbutamol
D. Orciprenaline
E. Isoprenaline (Isadrinum)

253. A patient suffers from acute cardiopulmonary failure with pulmonary


edema. What diuretic should be prescribed in the given case?
A. Triamterene
B. *Furosemide
C. Spironolactone
D. Hydrochlorothiazide (Dichlothiazidum)
E. Acetazolamide (Diacarb)

254. A 45-year-old woman suffers from arterial hypertension with high blood
concentration of angiotensin II. What antihypertensive drug is the most
recommended in the given case?
A. *Lisinopril
B. Prazosin
C. Metoprolol
D. Reserpine
E. Verapamil

255. An alcoholic suffers from alcoholic psychosis with evident psychomotor


agitation. What neuroleptic drug should be administered for emergency aid?
A. Diazepam
B. Sodium bromide
C. *Aminazine
D. Reserpine
E. Interferon
256. A 26-year-old woman with bronchitis has been administered a broad
spectrum antibiotic as a causal treatment drug. Specify this drug:
A. Interferon
B. *Doxycycline
C. BCG vaccine
D. Ambroxol
E. Dexamethasone

257. A 16-year-old young man suffering from seasonal allergic rhinitis has
been prescribed a highly active secondgeneration H1 blocker, which can be
characterized by absence of marked sedative action. Name this drug:
A. Pipolphen
B. Chloropyramine (Suprastin)
C. *Loratadine
D. Indometacin
E. Erythromycin

258. A 36-year-old patient has been administered a depolarizing muscle


relaxant during a surgery. Name this drug:
A. Proserin
B. Pipecuronium bromide (Arduan)
C. Diazepam
D. Aminazine
E. *Dithylinum

259. UN volunteers have arrived in Nigeria to assist the locals in after math
of earth quakes. What drug should they prescribe for individual
chemoprophylaxis of malaria?
A. Pyrantel
B. Pyrimethamine (Chloridinum)
C. Primaquine
D. *Chingamin
E. Interferon (Laferon)

260. After an extended treatment with sulfanamides a patient has developed


macrocytic anemia. Production of active forms of the following vitamin is
disrupted in such a condition:
A. Thiamine
B. Riboflavin
C. *Folic acid
D. Pyridoxine
E. Cyanocobalamin
261. Local anesthetics (novocaine, lidocaine and others) decreases pain
sensitivity of tissues by blocking Na+ and K+ ions from permeating membranes
of nerve fibers and endings. Such mechanism of drug action is called:
A. Receptor
B. Enzyme
C. *Membrane ionic
D. Antienzyme
E. Direct chemical

262. A patient presents with dry peeling skin, frequent cases of acute
respiratory diseases, xerophthalmia. What vitamin preparation should be
prescribed in this case?
A. Thiamine
B. *Retinol acetate
C. Cyanocobalamin
D. Menadione (Vikasolum)
E. Ergocalciferol

263. Prescription of penicillin G sodium salt has caused development of


neurotoxic effects (hallucinations, convulsions). Such reaction is the result of
antagonism with the following neurotransmitter:
A. Dopamine
B. Serotonin
C. *GABA
D. Adenosine
E. Acetylcholine

264. A patient with arthritis has been prescribed an anti-inflammatory


selective COX2 inhibitor. Select this drug among those given below:
A. Phenylbutazone (Butadion)
B. Dimethylsulfoxide (Dimexid)
C. Indometacin
D. Metamizole (Analgin)
E. *Celecoxib

265. The first-aid center has received a victim of a traffic accident diagnosed
with closed displaced fracture of the middle third of the thigh. For
repositioning of bone fragments the patient received 10 ml of 2 % dithylinum
solution intravenously, which resulted in prolonged period of apnoea and
muscle relaxation. What enzyme is deficient, resulting in such
pharmacogenetic enzymopathy?
A. *Pseudocholinesterase
B. Uridine diphosphate glucuronyltransferase
C. Glucose 6-phosphate dehydrogenase
D. Methemoglobin reductase
E. N-acetyltransferase

266. A 42-year-old man with gout presents with high content of uric acid in
blood. The patient was prescribed allopurinol to lower the concentration of uric
acid. Allopurinol is a competitive inhibitor of the following enzyme:
A. *Xanthine oxidase
B. Adenosine deaminase
C. Adenine phosphoribosyl transferase
D. Hypoxanthine phosphoribosyl transferase
E. Guanine deaminase

267. A patient complains of dizziness, thirst, difficult swallowing, and


impaired vision of close objects. Objectively: respiratory rate is increased,
pupils are dilated, general agitation, talkativeness, though the speech is
indistinct. BP is 110/70 mm Hg, heart rate is 110/min. Given symptoms can
indicate overdosage of the following drug:
A. *Atropine
B. Morphine
C. Ephedrine
D. Aminazine
E. Caffeine

268. A patient used an indirect-acting adrenergic agonist to treat rhinitis.


After the patient has been putting in the nose drops for several days, the
vasoconstrictive effect of the drug gradually diminished. Name this
phenomenon:
A. *Tachyphylaxis
B. Idiosyncrasy
C. Teratogenicity
D. Allergy
E. Cumulation

269. A 56-year-old man complains of thirst and frequent urination. The


endocrinologist diagnosed this patient with diabetes mellitus and prescribed
him glibenclamide. What mechanism of action does this drug have?
A. *Stimulation of beta-cells of islets of Langerhans
B. Facilitates glucose uptake by the tissues
C. Facilitates glucose transport through cell membranes
D. Suppression of alpha-cells of islets of Langerhans
E. Inhibits glucose absorption in the intestine

270. Patients with bile duct obstruction typically present with inhibited blood
clotting and develop hemorrhages due to insufficient assimilation of vitamin:
A. *K
B. A
C. D
D. E
E. C

271. Condition of a patient with diabetes mellitus sharply deteriorated after a


regular injection of insulin. The patient became anxious and broke out in cold
sweat; tremor of the extremities, general weakness, and dizziness appeared.
What medicine can remove these symptoms?
A. *Adrenaline
B. Tolbutamide
C. Caffeine
D. Noradrenaline
E. Glibutid (Buformin)

272. A schizophrenia patient was prescribed aminazine. What


pharmacodynamic action of this drug is the grounds for its prescription in this
case?
A. *Antipsychotic
B. Antiemetic
C. Hypothermic
D. Muscle relaxant
E. Hypotensive

273. A man with ischemic heart disease has been taking his medicine too
often throughout a day, which resulted in poisoning. Examination detects
cyanosis of skin and mucosa, sharp drop of blood pressure, tachycardia, and
respiratory depression. Blood methemoglobin is high. What type of medicine
did the patient overdose on?
A. *Organic nitrates
B. Alpha-adrenergic blockers
C. Calcium channel blockers
D. Adenosine-based drugs
E. Myotropic antispasmodics

274. Preoperative examination revealed prothrombin deficiency in the blood


of the patient. What drug should be preliminarily prescribed tomitigate blood
lossin the patient during the surgery?
A. *Vicasol (Menadione)
B. Thrombin
C. Aminocapronic acid
D. Phenylin (Phenindione)
E. Contrykal (Aprotinin)
275. A patient with streptococcal infection of the gingiva was prescribed a
drug with beta-lactam ring in its structure. What drug of those listed below
belongs to this pharmacological group?
A. *Benzylpenicillin
B. Rifampicin
C. Erythromycin
D. Streptomycin sulfate
E. Levomycetin (Chloramphenicol)

276. A 46-year-old patient suffering from ulcer disease of the stomach is


diagnosed with rheumatoid arthritis. What antiinflammatory drug would be the
most advisable in this case?
A. *Celecoxib
B. Prednisolone
C. Analgin (Metamizole)
D. Promedol (Trimeperidine)
E. Paracetamol

277. A patient with myocardial infarction has acute heart failure. Among the
drugs that increase the force of heart contractions the least dangerousin this
case will be:
A. *Dobutamine
B. Adrenaline
C. Isadrinum (Isoprenaline)
D. Euphyllin (Aminophylline)
E. Caffeine

278. Monoamine oxidase inhibitors are widely used as


psychopharmacological drugs. They change the level of nearly all
neurotransmitters in synapses, with the following neurotransmitter being the
exception:
A. *Acetylcholine
B. Noradrenaline
C. Adrenaline
D. Dopamine
E. Serotonin

279. A patient suffers from acute cardiopulmonary failure with pulmonary


edema. What diuretic should be prescribed in the given case?
A. *Furosemide
B. Triamterene
C. Spironolactone
D. Hydrochlorothiazide (Dichlothiazidum)
E. Acetazolamide (Diacarb)
280. A 60-year-old man diagnosed with chronic heart failure was brought to
the hospital. After a long course of treatment the patient developed signs of
intoxication: dyspnea, extrasystole, nausea, and disturbed perception of colors.
What medicine has caused such side-effects?
A. *Digoxin
B. Anaprilin (Propranolol)
C. Nitroglycerine
D. Drotaverine
E. Furosemide

281. A patient with exacerbated peptic ulcer disease of the stomach has made
an appointment with the doctor. What type of drugs should be included in the
complex therapy of this patient?
A. *H2 antagonists
B. H1 antagonists
C. Alpha-adrenergicantagonists
D. Beta-adrenergicantagonists
E. Alpha-adrenergicagonists

282. A patient with parkinsonism was prescribed levodopa, which led to


rapid improvement of the patient’s condition. What is the mechanism of action
of this drug?
A. *Stimulation of dopamine synthesis
B. Muscarinic acetylcholine receptor blockade
C. Stimulation of dopamine receptors
D. Anticholinesterase action
E. Muscarinic acetylcholine receptor stimulation

283. A patientwith contactdermatitisneeds to be prescribed an antihistamine


drug without somnolescent effect. Select this drug from the list:
A. *Loratadine
B. Dimedrol (Benadryl)
C. Suprastin (Chloropyramine)
D. Diprazine (Promethazine)
E. Ranitidine

284. Tostimulatethelaboractivityaparturient woman was prescribed a drug – a


posterior pituitary hormone that does not affect the blood pressure. As the
pregnancy progresses, the sensitivity to this hormone increases. Name the
prescribed drug:
A. *Oxytocin
B. Dinoprostone
C. Dinoprost
D. Pituitrin
E. Ergotal
285. A patient, who has been taking beta-adrenergic blockers, developed a
bronchial spasm. What group of bronchodilators should be chosen to stop the
bronchial spasm?
A. *Myotropic antispasmodics
B. Beta-adrenergicblockers
C. Indirect adrenergic agonists
D. Muscarinic cholinomimetic agents
E. Cholinesterase inhibitors

286. After a nephrectomy the patient developed enteroparesis. What


cholinergic agent with anticholinesterase action should be prescribed in this
case?
A. *Proserin
B. Carbacholine
C. Aceclidine
D. Pilocarpine
E. Acetylcholine

287. A 25-year-old young man came to the doctor complaining of general


weakness, rapid fatigability, irritability, reduced working ability, and bleeding
gums. What vitamin is likely to be deficient in this case?
A. *Ascorbic acid
B. Riboflavin
C. Thiamine
D. Retinol
E. Folic acid

288. A patient with pulmonary tuberculosis is prescribed the most effective


antituberculous antibiotic. Name this drug:
A. *Rifampicin
B. Tetracycline
C. Streptocide
D. Furasolidone
E. Bactrim (Co-trimoxazole)

289. An 18-year-old patient has developed candidiasis after the case of


pneumonia treated with beta-lactam antibiotic. What antimycotic agent should
be prescribed?
A. *Fluconazole
B. Streptomycin
C. Ampicillin
D. Phthalylsulfathiazole
E. Trimethoprim/sulfamethoxazole (Biseptol)
290. A patient was prescribed atropine sulfate to treat intestinal colic. What
concomitant disease can be a contraindication for this drug?
A. *Glaucoma
B. Bronchial asthma
C. Sinus bradycardia
D. Hypotension
E. Vertigo

291. A patient has developed status epilepticus. What medicine should be


used in this case to stop the seizures?
A. *Diazepam
B. Cyclodol (Trihexyphenidyl)
C. Diprazine (Promethazine)
D. Sodium bromide
E. Valerian extract

292. A 4-year-old child presents with numerous carious cavities and


yellowcolored teeth. The mother has a history of antibiotic treatment during
her pregnancy. What antibiotic was the most likely taken by the child’s
mother?
A. *Doxycycline
B. Streptomycin sulfate
C. Ampicillin
D. Erythromycin
E. Cefazolin

293. A 45-year-old women has an attack of cardiac fibrillation. She suffers


from stage II essential hypertension. What is the drug of choice for stopping
this attack?
A. Lidocaine
B. Sustac forte (Nitroglycerin)
C. Strophanthin
D. Potassium chloride
E. *Anaprilin (Propanolol)

294. A 40-year-old man with pulmonary tuberculosis was prescribed


isoniazid. Prolonged taking of this drug can result in development of the
following vitamin deficiency:
A. Folic acid
B. Cobalamin
C. Biotin
D. *Pyridoxine
E. Thiamine
295. A patient has been taking bisacodyl for a long time to treat chronic
constipation. However, several weeks later the aperients effect of the drug
diminished. What is the possible cause of this?
A. Drug dependence
B. Sensitization
C. Functional cumulation
D. *Acquired tolerance
E. Material cumulation

296. A patient with inoperable lung cancer accompanied by unbearable pain


was prescribed an analgesic. Against the background of analgesic therapy the
patient developed signs of intestinal obstruction. What analgesic could have
caused this complication?
A. Promedol (Trimeperidine)
B. Morphine
C. Fentanyl
D. *Analgin (Metamizole)*
E. Omnopon (Papaveretum)

297. A women with allegric neurodermatitis was prescribed a second-


generation antihistamine without depressing effect on the CNS. Name this
drug:
A. Diazolin (Mebhydrolin)
B. Ketotifen
C. *Loratadine
D. Dimedrol (Diphenhydramine)
E. Tavegyl (Clemastine)

298. During gastric resection the patient received mixed anesthesia with
tubocurarin chloride muscle relaxant; to restore spontaneous respiration the
patient received proserin. What pharmacological group does this drug belong
to?
A. *Cholinesterase inhibitors
B. Muscarinic antagonists
C. Calcium channel blockers
D. Muscarinic agonists
E. Angiotensin-converting-enzyme inhibitors

299. A patient with streptococcal infection of the gingival was prescribed a


drug with β-lactam ring in its structure. What drug of those listed below
belongs to this pharmacological group?
A. Erythromycin
B. *Benzylpenicillin
C. Rifampicin
D. Streptomycin sulfate
E. Levomycetin (Chloramphenicol)

300. A women with polyarticular rheumatoid arthritis was prescribed a non-


steroidal anti-inflammatory drug-diclofenac sodium. After the patient has been
taking it for some time, her concomitant disease exacerbated, which forced the
doctor to cancel the prescription of this drug. What concomitant disease could
necessitate cancellation of this drug prescription?
A. Bronchial asthma
B. *Ulcer disease
C. Essential hypertension
D. Ischemic heart disease
E. Diabetes mellitus

301. Before a surgery the patient was prescribed a synthetic antiprotozoal


drug for prevention of wound infection. The prescribed drug is highly effective
against Helicobacter pylori. Name this drug:
A. Chingamin (Chloroquine)
B. Isoniazid
C. *Metronidazole
D. Doxycycline hydrochloride
E. Acyclovir

302. A patient was administered a certain drug for relief of cardiac rhythm
disturbance. This drug can be used as a local anesthetic as well. Name this
drug:
A. Dicain (Tetracaine)
B. Diphenine (Phenytoin)
C. Cocaine hydrochloride
D. Anaesthesin (Benzocaine)
E. *Lidocaine hydrochloride

303. A 33-year-old women, who for a long time has been treated for chronic
polyarthritis, complains of elevated blood pressure, changes in adipose tissue
distribution, and disturbed menstrual cycle. What drug does this patient take?
A. Synaflan (Fluocinolone acetonide)
B. Beclometasone
C. Indometacin
D. *Prednisolone
E. Butadion (Phenylbutazone)

304. A 39-year-old man presents with hyperkeratosis, disturbed twilight


vision, and high risk of infectious processes. What vitamin preparation should
he be prescribed?
A. Tocopherol acetate
B. *Retinol acetate
C. Riboflavin
D. Ergocalciferol
E. Pyridoxine hedrochloride

305. The neurological department received a patient complaining of memory


deterioration and loss of mental work capacity that developed after a head
trauma. Recommend him a medicine for improvement in cerebral metabolism:
A. *Piracetam (Nootropil)
B. Sydnocarb (Mesocarb)
C. Analgin (Metamizole)
D. Caffeine
E. Meridil ( Methylphenidate)

306. To treat the burns, a patient was prescribed a drug with antiseptic
properties that are based on formation of atomic oxygen in the presence of
organic substances. This drug has also an astringent (anti-inflammatory) effect
due to formation of albuminates. Name this drug:
A. Hydrogen peroxide
B. *Potassium permanganate
C. Chlorhexidine digluconate
D. Ethyl alcohol
E. –

307. A patient with essential hypertension was prescribed a drug that inhibits
angiotensin-converting enzyme (ACE). What drug is it?
A. Carvedilol
B. Lozartan
C. Nafedipine
D. Colestyramine
E. *Lisinopril

308. A patient with myocardial infarction in the acute phase has been
hospitalized into the cardiology unit. To induce platelet lysis in the patient’s
coronary vessels during the early hours of infarction, the following enzyme
should be used:
A. Lysozyme
B. Trypsin
C. Hyaluronidase
D. Chymotrypsin
E. *Streptokinase

309. A patient has elevated blood pressure due to increased vascular tone. To
lower the blood pressure in this case it is necessary to prescribe the blockers of:
A. Alpha-adrenoceptors
B. Alpha- and beta-adrenoceptors
C. Muscarinic acetylcholine receptors
D. Histamine H1 receptors
E. *Beta-adrenoceptors

310. A patient with peptic ulcer disease was prescribed famotidine. As a


result his gastric juice acidity significantly significantly decreased. What is the
mechanism of action of this drug?
A. Blockade of histamine receptors in the sympathetic ganglia
B. Histamine H1 receptor blockade
C. Muscarinic M1 receptor blockade
D. Inhibition of H+, K+-ATPase activity
E. *Histamine H2 receptor blockade

311. A patient with arrhythmia was hospitalized into the cardiology unit.
What antiarrhythmic drug should be prescribed?
A. Furacilin (Nitrofural)
B. Diclofenac sodium
C. *Amiodarone
D. Drotaverine hydrochloride
E. Acetylsalicylic acid

312. A 46-year-old man presents with fatigue and joint pain in his fingers and
wrists for the last 2 months. The pain is present in both hands and the wrists are
swollen. Furthermore, he describes morning stiffness in his joints lasting about
2 hours, which improves with use. His past medical history reveals he has been
successfully treated for H. pylori related ulcers last year. He denies smoking
and stopped drinking when his gastric symptoms started. Which of the
following drugs is the best choice for his joints’ pain management?
A. *Celecoxib
B. Prednisone
C. Paracetamol
D. Aspirin
E. Morphine

313. A 64-year-old man presents with a tremor in his legs and arms. He says
he has had the tremor for «many years», but it has worsened in the last year.
The tremor is more prominent at rest and nearly disappears on movement. His
daughter mentions that his movements have become slower. The patient is
afebrile and vital signs are within normal limits on physical examination, the
patient is hunched over and his face is expressionless throughout examination.
There is a «pill-rolling» resting tremor that is accentuated when the patient is
asked to clench the contralateral hand and alleviated by finger-nose testing.
When asked to walk across the room, the patient has difficulty taking the first
step, has a stopped posture and takes short rapid shuffling steps. A doctor
initiates pharmacotherapy and drug of first line, levodopa, is prescribed. Which
of the following is the most likely mechanism of action of this drug?
A. Activation of M2- cholinergic receptors
B. Cholinesterase inhibition
C. –
D. *Stimulation of dopamine production
E. Inhibition of M2- cholinergic receptors
Назва наукового напрямку (модуля): Семестр: 6
Pharmacology (independent work) 2018-2019
Опис:
Факультет іноземних студентів (медицина), 3 курс
Перелік питань:
1. Methotrexate (structural analogue of the folic acid which is competitive inhibitor of the
dihydrofolatreductase) is prescribed for treatment of the malignant tumour. On which level does
methotrexate hinder synthesis of the nucleic acids?
A. Replication
B. Transcription
C. * Mononucleotide synthesis
D. Reparation
E. Processing
2. A 60-year-old patient was hospitalised to the surgical department because of infection caused by blue
pus bacillus (Pseudomonas aeruginosa) which is sensative to penicillin antibiotics. Indicate which of
the given penicillins has marked activity to the Pseudomonas aeruginosa?
A. Benzylpenicillin
B. Phenoxymethylpenicillin
C. * Carbenicillin disodium
D. Oxacillin
E. Methicillin
3. A 45-year-old woman suffers from allergic seasonal coryza caused by the ambrosia blossoming.
What adipose cells group stabilizer medicine can be used for prevention of this disease?
A. Diazoline
B. Phencarol
C. Tavegyl
D. Dimedrol
E. * Ketotifen
4. The alternate usage of dichlotiazide, etacrin acid and lasex did not cause marked diuretic effect in the
patient with marked peripheral edema. The aldosterone level in the blood is increased.
Indicatewhichmedicine should be prescribed:
A. Mannit
B. * Spironolacton
C. Clopamid
D. Urea
E. Amilorid
5. A 56-year-old patient complaining of thirst and frequent urination was diagnosed with diabete
mellitus. Butamin was prescribed. How does the medicine act?
A. It helps to absorb the glucose by the cells of the organism tissues
B. * It stimulates beta-cells of Langergans islets
C. It relieves transport of glucose through the cells’ membranes
D. It inhibits alpha-cells of Langergans islets
E. It inhibits absorption of glucose in the intestines
6. A 50-year-old male farm worker has been brought to the emergency room. He was found confused in
the orchard and since then has remained unconscious. His heart rate is 45 and his blood pressure is
80/40 mm Hg. He is sweating and salivating profusely. Which of the following should be prescribed?

A. * Atropine
B. Norepinephrine
C. Proserine
D. Physostigmine
E. Pentamine
7. A 58-year-old female has undergone surgery for necrotic bowel. Despite having been treated with
antibiotics, on postoperative day 5, she develops symptoms (fever, hypotension, tachycardia,
declining urine output, and confusion) consistent with septic shock. What hemodynamic support
would be helpful?
A. Dobutamine infusion
B. Antibiotic administration
C. Fluid administration
D. * Fluids and Dobutamine infusion
E. Atropine administration
8. A 42-year-old man who has been injured in a car accident is brought into the emergency room. His
blood alcohol level on admission is 250 mg/dL. Hospital records show a prior hospitalization for
alcohol related seizures. His wife confirms that he has been drinking heavily for 3 weeks. What
treatment should be provided to the patient if he goes into withdrawal?
A. Phenobarbital
B. Pentobarbital
C. * Diazepam
D. Phenytoin
E. None
9. A 13-year-old girl with history of asthma complained of cough, dyspnea and wheezing. Her
symptoms became so severe that her parents brought her to the emergency room. Physical
examination revealed diaphoresis, dyspnea, tachycardia and tachypnea. Her respiratory rate was
42/min, pulse rate was 110 beats per minute, and blood pressure was 130/70 mm Hg. Choose from
the following list the most appropriate drug to reverse the bronchoconstriction rapidly:
A. Cromolyn
B. * Salbutamol
C. Beclomethasone
D. Methylprednidsolone
E. Ipratropium
10. A doctor administered Allopurinol to a 26-year-old young man with the symptoms of gout. What
pharmacological action of Allopurinol ensures therapeutical effect?
A. By increasing uric acid excretion
B. By inhibiting leucocyte migration into the joint
C. By general anti-inflammatory effect
D. By general analgetic effect
E. * By inhibiting uric acid synthesis
11. A patient had been taking glucocorticoids for a long time. When the preparation was withdrawn he
developed the symptoms of disease aggravation, decreased blood pressure and weakness. What is the
reason of this condition?
A. Hyperproduction of ACTH
B. Sensibilization
C. * Appearance of adrenal insufficiency
D. Habituation
E. Cumulation
12. The patient with pneumonia was treated with antibiotics for a long period. After treatment patient
complains of frequent and watery stool, abdomenal pain. What is the reason of intestine function
disorder?

A. Antibiotics toxic influence on the GIT


B. Autoimmune reaction development
C. Bacteria toxins influence
D. * Intestinal disbacteriosis development
E. Hereditary enzyme defect
13. A 55-year-old patient with continuing ventricular arrhythmias was admitted to the hospital. The
patient is taking timolol drops for glaucoma, daily insulin injections for diabetes mellitus, and an
ACE inhibitor for hypertension. You have decided to use phenytoin instead of procainamide. What is
the reason?
A. The local anesthetic effect of procainamide would potentiate diabetes
B. The hypertensive effects of procainamide would aggravate the hypertension
C. The local anesthetic effect of procai-namide would aggravate the hypertension
D. The cholinergic effects of procainamide would aggravate the diabetes
E. * The anticholinergic effect of procainamide would aggravate glaucoma
14. A 35-year-old man under the treatment for pulmonary tuberculosis has acute-onset of right big toe
pain, swelling, and low-grade fever. The gouty arthritis was diagnosed and high serum uric acid level
was found. Which of the following antituberculosis drugs are known for causing high uric acid
levels?
A. * Pyrazinamide
B. Cycloserine
C. Thiacetazone
D. Rifampicin
E. Aminosalicylic acid
15. A 25-year-old woman with red and itchy eczematoid dermatitis visits your office. She had a dental
procedure one day earlier with administration of a local anesthetic. There were no other findings,
although she indicated that she had a history of allergic reactions. Which of the following drugs is
most likely involved?
A. Cocaine
B. * Procaine
C. Lidocaine
D. Bupivacaine
E. Etidocaine
16. The CNS stimulation produced by methylxanthines, such as caffeine, is most likely due to the
antagonism of one of the following receptors:
A. Glycine receptors
B. Glutamate receptors
C. * Adenosine receptors
D. GABA receptors
E. Cholinergic muscarinic receptors
17. A 37-year-old man was admitted to the surgical department with the symptoms of acute pancreatitis:
vomiting, diarrhea, bradycardia, hypotention, weakness, dehydration of the organism. What medicine
should be used first of all?
A. * Contrycal
B. No-spa
C. Platyphylline
D. Etaperazine
E. Ephedrine

18. A patient has been taking a mixture prescribed by neuropathologist for neurasthenia for two weeks.
The patient feels better but has developed coryza, conjunctivitis, rash, inertia, decrease of memory.
She is diagnosed with bromizm. What should be prescribed to decrease the symptoms?
A. Glucose solution 5 %
B. * Natrium chloride
C. Asparcam
D. Polyglucin
E. No-spa
19. Analeptical remedy of reflective type from the H-cholinomimetics group was given to the patient for
restoration of breathing after poisoning with carbon monoxide. What medicine was prescribed to the
patient?
A. Atropine sulphate
B. Adrenalin hydrochloride
C. * Lobeline hydrochloride
D. Mesaton
E. Pentamin
20. A patient with bronchial asthma had been taking tablets which caused insomnia, headache, increased
blood pressure. What medecine can cause such complications?
A. Adrenaline
B. Chromolin sodium
C. * Ephedrine
D. Euphyline
E. Izadrine
21. To anaesthetize the manipulation related to burn surface treatment, a patient was intravenously
injected a medication for short-acting narcosis. 1 minute later the patient being under anaesthesia had
increased blood pressure, tachycardia, increased tone of sceletal muscles; reflexes remained. After
awakening the patient had desorientation and visual hallucinations. What medication was the patient
injected?
A. Sombrevin
B. Diethyl ether
C. * Ketamine
D. Thiopental sodium
E. Nitrous oxide
22. Intake of oral contraceptives containing sex hormones inhibits secretion of the hypophysiae
hormones. Secretion of which of the indicated hormones is inhibited while taking oral contraceptives
with sex hormones?
A. Vasopressin
B. Thyrotropic
C. Somatotropic
D. * Follicle-stimulating
E. Oxytocin
23. A 60-year-old patient was admitted to the surgical department because of infection caused by blue
pus bacillus (Pseudomonas aeruginosa) which is sensative to penicillin antibiotics. Indicate which of
the given penicillins has marked activity to the Pseudomonas aeruginosa?
A. * Carbenicillin disodium
B. Benzylpenicillin
C. Phenoxymethylpenicillin
D. Oxacillin

E. Methicillin
24. A 52-year-old patient has the following diagnosis: systemic amebiasis with involvment of intestines,
liver, lungs. What drug should be prescribed?
A. Quiniofone
B. Tetracycline
C. * Metronidasol
D. Quingamine
E. Enteroseptol
25. A 38-year-old man who poisoned himself with mercury dichloride was taken to the admission room
in grave condition. What antidote should be immediately introduced?
A. Dipiroxim
B. Atropine
C. Nalorphine
D. Isonitrosine
E. * Unithiol
26. A patient with complaints of dryness in the mouth, photophobia and vision impairment was admitted
to the reception-room. Skin is hyperemic, dry, pupils are dilated, tachycardia. Poisoning with
belladonna alkaloids was diagnosed on further examination. What medicine should be prescribed?
A. * Prozerin
B. Diazepam
C. Pilocarpine
D. Armine
E. Dipiroxim
27. A patient with rheumatoid arthritis who had been treated with indometacin has got signs of
gastropathy. What activity of the drug can this complication be connected with?
A. Antiserotonin
B. Antihistamine
C. * Anticyclooxygenase
D. Antikinine
E. Locally irritating
28. A patient who was previously ill with mastectomy as a result of breast cancer was prescribed
radiation therapy. What vitamin preparation has marked radioprotective action caused by antioxidant
activity?
A. Ergocalciferol
B. * Tocopherol acetate
C. Thiamine chloride
D. Riboflavin
E. Folic acid
29. A patient has got a spasm of smooth muscles of bronchi. Activators of what membrane cytoreceptors
are phisiologically reasoned to stop an attack?
A. Alpha-аdrenoreceptors
B. Alpha- and beta-аdrenoreceptors
C. * Beta-adrenoreceptors
D. Н-cholinoreceptors
E. М-cholinoreceptors
30. A 42-year-old man suffering from gout has increased level of urinary acid in blood. Allopurinol was
prescribed to decrease the level of urinary acid. Competitive inhibitor of what enzyme is allopurinol?
A. Adenosine deaminase

B. Adenine phosphoribosiltransferase
C. * Xanthine oxidase
D. Hypoxanthine phosphoribosiltransferase
E. Guanine deaminase
31. A patient with infectious mononucleosis had been taking glucocorticoids for two weeks. He was
brought into remission, but he fell ill with acute attack of chronic tonsillitis. What action of
glucocorticoids caused this complication?
A. * Immunosuppressive
B. Anti-inflammatory
C. Antishock
D. Antiallergic
E. Antitoxic
32. A patient working at a chemical plant was admitted to the toxicological department with mercury
poisoning. What medicine should be used?
A. Isonitrozin
B. Naloxone
C. * Unithiol
D. Activated carbon
E. Enterosorbent
33. A patient with continious bronchopneumonia was admitted to the therapeutic department. Antibiotic
therapy didn’t give much effect. What medication for improvement of immune state should be added
to the complex treatment of this patient?
A. Analgin
B. * Timaline
C. Sulfocamphocaine
D. Benadryl
E. Paracetamol
34. Patient with abscess of the cut wound applied to the traumatological department. In order to clean the
wound from the pus doctor washed it with 3 % hydrogen peroxide. Foam was absent. What caused
the absence of the drug activity?
A. Low concentration H2O2
B. Inherited insufficiency of erythrocyte phosphatdehydrogenase
C. Shallow wound
D. Pus in the wound
E. * Inherited insufficiency of catalase
35. A 50-year-old patient with typhoid fever was treated with Levomycetin, next day his condition
became worse, temperature rose to 39,6. What caused the complication?
A. * The effect of endotoxin agent
B. Allergic reaction
C. Irresponsiveness of an agent to the levomycetin
D. Secondary infection addition
E. Reinfection
36. Tuberculosis can be treated by means of combined chemotherapy that includes substances with
different mechanisms of action.What antituberculous medication inhibits transcription of RNA into
DNA in mycobacteria?
A. Isoniazid
B. * Rifampicin
C. Streptomycin

D. Ethionamide
E. Para-aminosalicylic acid
37. A patient who has been treated with diazepam on account of neurosis complains of toothache. Doctor
administered him an analgetic, but its dose was lower than average therapeutic dose. What
phenomenon did the doctor take into account while prescribing the patient an underdose?
A. Summation
B. Cumulation
C. * Potentiation
D. Drug dependence
E. Tolerance
38. To anaesthetize the surgical treatment of burn surface, a patient was intravenously injected a
medication for short-acting narcosis. 1 minute later the patient being under anaesthesia showed
increased blood pressure, tachycardia, increased tone of skeletal muscles; reflexes remained. After
recovering from anaesthesia the patient had disorientation and visual hallucinations. What medication
was the patient injected?
A. Sombrevin
B. * Ketamine
C. Diethyl ether
D. Thiopental sodium
E. Nitrous oxide
39. A patient had to go through an operation. Doctors introduced him dithylinum (listenone) and
performed intubation. After the end of operation and cessation of anesthesia the independent
respiration wasn’t restored. Which enzyme deficit prolongs the action of muscle relaxant?
A. Succinate dehydrogenase
B. Carbanhydrase
C. N-acetyltransferase
D. * Pseudocholinesterase
E. K-Na-adenosine triphosphatase
40. A patient consulted a doctor about bowels disfunction. The doctor established symptoms of
duodenitis and enteritis. Laboratory examination helped to make the following diagnosis: lambliosis.
What medication should be administered?
A. Erythromycin
B. Monomycin
C. Chingamin
D. * Metronidazole
E. Tetracycline
41. Introduction of a pharmaceutical substance to an experimental animal resulted in reduction of
salivation, pupil mydriasis. Next intravenous introduction of acetylcholine didn’t lead to any
significant changes of heart rate. Name this substance:
A. * Atropine
B. Adrenaline
C. Propranolol
D. Proserin
E. Salbutamol
42. Continious taking of a drug can result in osteoporosis, erosion of stomach mucous membrane,
hypokaliemia, retention of sodium and water, reduced content of corticotropin in blood. Name this
drug:
A. Hydrochlorothiazide
B. * Prednisolone

C. Digoxin
D. Indometacin
E. Reserpine
43. A 63 y.o. man with collapse symptoms was delivered to the emergency hospital. A doctor chose
noradrenaline in order to prevent hypotension. What is the action mechanism of this medication?
A. Activation of serotonin receptors
B. Activation of beta-adrenoreceptors
C. * Activation of alpha1-adrenoreceptors
D. Activation of dopamine receptors
E. Block of M -cholinoreceptors
44. A 35 y.o. patient who often consumes alcohol was treated with diuretics. There appeared serious
muscle and heart weakness, vomiting, diarrhea, AP-100/60 mm Hg, depression. This condition is
caused by intensified excretion with urine of:
A. Sodium
B. Chlorine
C. Calcium
D. Phosphates
E. * Potassium
45. A patient was diagnosed with active focal pulmonary tuberculosis. What drug should be prescribed in
the first place?
A. * Isoniazid
B. Sulfalen
C. Cyclocerine
D. Ethionamide
E. Ethoxide
46. Examination of a patient revealed extremely myotic pupils, sleepiness, infrequent Chain-Stoke’s
respiration, urinary retention, slowing-down of heart rate, enhancement of spinal reflexes. What
substance caused the poisoning?
A. Atropine
B. Phosphacole
C. * Morphine
D. Caffeine
E. Barbital
47. A doctor administered a patient with allergic dermatitis a H1-histamine blocker as a part of complex
treatment. Name this medication:
A. Cromolyn sodium
B. Prednisolone
C. Adrenaline
D. * Loratadine
E. Hydrocortisone
48. A patient with II stage hypertension has been taking one of hypotensive medications for the purpose
of treatment. After a time arterial pressure decreased, but the patient started complaining of
flaccidity, sleepiness, indifference. A bit later he felt stomach pain. He was diagnosed with ulcer.
What hypotensive medication has the patient been taking?
A. Dibazole
B. * Reserpine
C. Furosemide
D. Verapamil

E. Captopril
49. For the preparation of a patient’s burn skin surface a certain medication was used. Its antiseptic
action is provided by free oxygen that segregates in presence of organic substances. Choose the right
answer:
A. * Potassium permanganate
B. Furacilin
C. Chlorhexidine
D. Boric acid
E. Sodium bicarbonate
50. A 50 y.o. patient with chronic cardiac insufficiency and tachyarrythmia was prescribed a cardiotonic
drug. What drug was prescribed?
A. Dopamine
B. Dobutamine
C. * Digoxin
D. Amyodarone
E. Mildronate
51. A 36 y.o. man has a craniocerebral trauma. Objectively: diminished breath sounds, thready pulse, no
reflexes. What way of pyracetam introduction will be the most apropriate in this case?
A. Rectal
B. Subcutaneous
C. Peroral
D. * Intravenous
E. Inhalation
52. A patient ill with bronchial asthma didn’t inform his doctor that he had attacks of stenocardia. Doctor
administered him a medication, which taking resulted in less frequent attacks of bronchial asthma,
but stenocardia attacks became more frequent. What medication was administered?
A. Salbutamol
B. Aminophylline
C. * Isadrin
D. Cromolyn sodium
E. Phenotherol
53. A patient who has been suffering from cardiac insufficiency for several months has been taking
digoxin on an outpatient basis. At a certain stage of treatment there appeared symptoms of drug
overdose. What phenomenon underlies the development of this complication?
A. Habituation
B. Sensibilization
C. * Material cumulation
D. Functional cumulation
E. Tachyphylaxis
54. A patient suffers from vision impairment – hemeralopy (night blindness). What vitamin preparation
should be administered the patient in order to restore his vision?
A. * Retinol acetate
B. Vicasol
C. Pyridoxine
D. Thiamine chloride
E. Tocopherol acetate
55. A patient suffers from severe postoperative pseudomonadous infection. What of the following
antibiotics should be administered in this case?

A. Benzylpenicillin
B. Cephazolin
C. * Amicacin sulfate
D. Erythromycin
E. Doxycycline
56. After a tooth extraction a patient felt persistent pain behind his breast bone. After sublingual intake of
an antianginal drug the pain behind the breast bone disappeared, but the patient complained of
headache and dizziness. What drug are these properties typical for?
A. Propranolol
B. * Nitroglycerin
C. Metoprolol
D. Validol
E. Verapamil
57. A patient with fracture of his lower jaw was admitted to the maxillofacial department. It was decided
to fix his bones surgically under anaesthetic. After intravenous introduction of muscle relaxant there
arose short fibrillar contractions of the patient’s facial muscles. What muscle relaxant was applied?
A. Tubocurarin chloride
B. * Dithylinum
C. Pipecuronium bromide
D. Diazepam
E. Melictine
58. Examination of a 60 y.o. patient revealed hyperglycemia and glucosuria. A doctor administered him a
medication for internal use. What medication is it?
A. Furosemide
B. Oxytocin
C. Pancreatine
D. Corglycon
E. * Glibenclamid
59. A liquidator of a breakdown at a nuclear power plant who was irradiated complained about vomiting
that occurs all of a sudden. What medication should be prescribed?
A. Reserpine
B. Atropine
C. * Metoclopramide
D. Aeron
E. De-Nol
60. A patient with chronic cardiac insufficiency has been treated with cardiotonic drugs and a thiazide
diuretic, but in spite of it there are still edemata and risk of ascites. What medication should be
prescribed to amplify diuretic effect of the applied drugs?
A. Furosemide
B. Amyloride
C. * Spironolactone
D. Clopamide
E. Manitole
61. A patient ill with collagenesis has been taking prednisolone for a long time. Hypokaliemia
development caused spastic pain of skeletal muscles. What medication should be used in order to
correct potassium exchange?
A. Dithylinum
B. * Panangin

C. Diazepam
D. Noshpa
E. Thyrocalcitonin
62. A 37 year old patient suffering from obliterating vascular endarteritis of lower limbs takes daily 60
microgram/kilogram of phenylin. Because of presentations of convulsive disorder (craniocerebral
trauma in anamnesis) he was prescribed phenobarbital. With holding this drug caused nasal
hemorhage. What is this complication connected with?
A. Aliphatic hydroxylation of phenobarbital
B. Conjugation of phenylin with glucuronic acid
C. Oxidative deamination of phenylin
D. * Induction of enzymes of microsomal oxidation in liver caused by phenobarbital
E. Inhibition of microsomal oxidation in liver caused by phenobarbital
63. A patient ill with neurodermatitis has been taking prednisolone for a long time. Examination revealed
high rate of sugar in his blood. This complication is caused by the drug influence upon the following
link of carbohydrate metabolism:
A. Glycogenogenesis activation
B. * Gluconeogenesis activation
C. Intensification of glucose absorption in the bowels
D. Inhibition of glycogen synthesis
E. Activation of insulin decomposition
64. A patient with frequent attacks of stenocardia was prescribed sustak-forte to be taken one tablet twice
a day. At first the effect was positive but on the second day stenocardia attacks resumed. What can
explain inefficiency of the prescribed drug?
A. * Tachyphylaxis
B. Cumulation
C. Sensibilization
D. Idiosyncrasy
E. Dependence
65. Examination of a 70 year old patient rrevealed insulin-dependent diabetes. What drug should be
administered?
A. Insulin
B. Mercazolilum
C. * Glibenclamid
D. Parathyroidin
E. Cortisone
66. A patient suffering from chronic cardiac insufficiency was recommended to undergo a prophylactic
course of treatment with a cardiological drug from the group of cardiac glycosides that is to be taken
enterally. What drug was recommended?
A. Strophanthine
B. * Digoxin
C. Corglycon
D. Cordiamin
E. Cordarone
67. An ophthalmologist used a 1 % mesaton solution for the diagnostic purpose (pupil dilation for
eye-ground examination). What is the cause of mydriasis induced by the drug?
A. Activation of alph2 adrenoreceptors
B. * Activation of alpha1 adrenoreceptors
C. Block of alph1 adrenoreceptors

D. Activation of beta1 adrenoreceptors


E. Activation of M-cholinoreceptors
68. Mother of a 2 year old child consulted a stomatologist. In the period of pregnancy she was irregularly
taking antibiotics for an infectious disease. Examination of the child revealed incisor destruction,
yellow enamel, brown rim around the dental cervix. What drug has apparent teratogenic effect?
A. Furosemide
B. * Doxacycline
C. Ampiox
D. Xantinol nicotinate
E. Octadine
69. Examination of a 60 year old patient revealed hyperglycemia and glucosuria. A doctor administered
him a medication for internal use. What medication is it?
A. Furosemide
B. * Glibenclamid
C. Oxytocin
D. Pancreatine
E. Corglycon
70. A patient ill with essential hypertension was recommended a drug that prevents thrombosis. It is to be
taken parenterally. What drug is it?
A. * Heparin
B. Amben
C. Protamine sulfate
D. Neodicumarin
E. Syncumar
71. A 7 year old child is ill with bronchitis. It is necessary to administer him an antibacterial drug. What
drug of fluoroquinolone group is CONTRA-INDICATED at this age?
A. Ampicillin
B. * Cyprofloxacin
C. Amoxicillin
D. Sulfadimethoxine
E. Ampiox
72. If a man has an attack of bronchiospasm it is necessary to reduce the effect of vagus on smooth
muscles of bronchi. What membrane cytoreceptors should be blocked for this purpose?
A. N - cholinoreceptors
B. Alpha-adrenoreceptors
C. * M - cholinoreceptors
D. Beta-adrenoreceptors
E. Alpha- and beta-adrenoreceptors
73. A patient takes digoxin for treatment of cardiac insufficiency. What diuretic may increase digoxin
toxicity due to the intensified excretion of K+ ions?
A. Spironolactone
B. Panangine
C. Siliborum
D. Lisinopril
E. * Hydrochlorothiazide
74. A patient ill with amebiasis was prescribed a certain drug. The use of alcohol together with this drug
is contraindicated because the drug inhibits metabolism of ethyl alcohol. What drug is it?
A. Reserpine

B. Clonidine
C. Diazepam
D. * Metronidazole
E. Aminazine
75. A patient ill with chronic cardiac insufficiency was prescribed an average therapeutic dose of
digoxin. Two weeks after begin of its taking there appeared symptoms of drug intoxication
(bradycardia, extrasystole, nausea). Name the phenomenon that caused accumulation of the drug in
the organism?
A. Functional cumulation
B. * Material cumulation
C. Tolerance
D. Tachyphylaxis
E. Idiosyncrasy
76. On the 2-3rd day after stomach resection intestinal peristalsis wasn’t restored. What is to be
administered for stimulation of gastrointestinal tract?
A. Prasosin
B. * Proserin
C. Cyclodole
D. Atropine sulfate
E. Acetylcholine
77. A stomatologist injected a patient with a certain drug in order to reduce salivation during tooth
filling. What drug is it?
A. Adrenaline hydrochloride
B. Proserin
C. Pilocarpine hydrochloride
D. * Atropine sulfate
E. Mesaton
78. A patient consulted a stomatologist about purulent inflammation of his gums. What drug will be the
most effective if it is suspected that a causative agent is an anaerobe?
A. Gentamicin
B. Oxacillin sodium
C. * Metronidazole
D. Co-trimoxazole
E. Nitroxoline
79. Examination of a 35 year old patient revealed high acidity of gastric juice. What receptors should be
blocked in order to reduce it?
A. Alpha1-adrenoreceptors
B. Alpha2-adrenoreceptors
C. * Histamine
D. Beta1-adrenoreceptors
E. Beta2-adrenoreceptors
80. A 12 year old child has intolerance to some food stuffs. Their consumption causes an allergic
reaction in form of itching skin eruptions. What antihistaminic drug should be admistered so that the
child could attend school ?
A. Dimedrol
B. Diclofenac
C. * Loratadine
D. Aminophylline

E. Ephedrine
81. A patient that entered the admission office had the following signs of acute cardiac insufficiency:
paleness, acrocyanosis, frequent shallow respiration. What drug is indicated in this case?
A. Digitoxin
B. Cordiamin
C. * Corglycon
D. Nitroglycerine
E. Adrenaline hydrochloride
82. A patient had hemorrhagic stroke. Blood examination revealed strengthened kinin concentration.The
patient was prescribed contrical. It was administered in order to inhibit the following proteinase:
A. Pepsin
B. Trypsin
C. * Kallikrein
D. Chemotrypsin
E. Collagenase
83. A patient with bacterial pneumonia was prescribed benzylpenicillin. What is the mechanism of its
antibacterial effect?
A. Inhibition of intracellular protein synthesis
B. Abnormal permeability of cytoplasmic membrane
C. * Inhibition of synthesis of microorganism wall
D. Inhibition of SH-groups of microorganism enzymes
E. Antagonism with p-amino-benzoic acid
84. A patient who has been treated in a neural clinic and has been taking a sedative for a long time got
the following complication: cough, rhinitis, epiphora. What drug caused these disturbances?
A. Diazepam
B. * Sodium bromide
C. Valerian
D. Phenazepam
E. Reserpine
85. In order to accelerate healing of a radiation ulcer a vitamin drug was administered. What drug is it?
A. Retabolil
B. Prednisolone
C. * Retinol acetate
D. Levamisole
E. Methyluracil
86. Pharmacological effects of antidepressants are connected with inhibition of an enzyme catalyzing
biogenic amines noradrenaline and serotonine in the mitochondrions of cerebral neurons. What
enzyme participates in this process?
A. Transaminase
B. Decarboxylase
C. Peptidase
D. * Monoamine oxidase
E. Lyase
87. An oncological patient was prescribed methotrexate. With the lapse of time target cells of the tumour
lost susceptibility to this drug. There is change of gene expression of the folowing enzyme:
A. Thiaminase
B. * Dehydrofolate reductase

C. Deaminase
D. Folate oxidase
E. Folate decarboxylase
88. A patient with hypertensic crisis was admitted to the cardiological department, he was injected
intravenously with an antihypertensive drug – salt of an alkaline-earth metal. What drug was
injected?
A. Potassium chloride
B. Sodium hydrocarbonate
C. * Magnesium sulfate
D. Calcium lactate
E. Benzohexamethonium
89. A patient with acute morphine poisoning was delivered to a hospital. What specific narcotic
antagonist should be chosen in this case?
A. Paracetamol
B. Methacin
C. * Naloxone
D. Digoxin
E. Unithiol
90. A 30-year-old patient complains about having abdominal pain and diarrhea; body temperature rise up
to 37, 5oC along with chills. The day before a patient had been in a forest and drunk from an open
water reservoir. Laboratory analyses enabled to make the following diagnosis: amebic dysentery.
What is the drug of choice for its treatment?
A. Furazolidonum
B. Levomycetin
C. * Metronidazole
D. Phthalazol
E. Emetine hydrochloride
91. A patient suffering from stomach ulcer has been treated with an antacid drug almagel. For acute
bronchitis treatment he was prescribed the antibiotic methacycline. However within next 5 days the
fever didn’t fall, cough and sputum nature remained unchanged. A physician came to the conclusion
that the drugs were incompatible. What type of drug incompatibility is the case?
A. Pharmacokinetic, biotransformation stage
B. Pharmaceutic
C. * Pharmacokinetic, absorption stage
D. Pharmacodynamic
E. Direct antagonism
92. Proserin increases skeletal muscle tone when given systematically. Halothane induces relaxation of
skeletal muscles and reduces proserin effects. What is the nature of proserin and halothane
interaction?
A. Direct functional antagonism
B. Competitive antagonism
C. * Indirect functional antagonism
D. Independent antagonism
E. Noncompetitive antagonism
93. A patient suffering from myasthenia has been administered proserin. After its administration the
patient has got nausea, diarrhea, twitch of tongue and skeletal muscles. What drug would help to
eliminate the intoxication?
A. * Atropine sulfate

B. Physostigmine
C. Pyridostigmine bromide
D. Isadrine
E. Mesatonum
94. A patient who had myocardial infarction was administered 75 mg of acetylsalicinic acid a day. What
is the purpose of this administration?
A. Inflammation reduction
B. Pain relief
C. Temperature reduction
D. * Reduction of thrombocyte aggregation
E. Coronary vessel dilatation
95. To prevent the transplant rejection after organ transplantation it is required to administer
hormonotherapy for the purpose of immunosuppression. What hormones are used for this purpose?
A. Mineralocorticoids
B. Sexual hormones
C. Catecholamines
D. Thyroid
E. * Glucocorticoids
96. A patient suffering from chronic hyperacidic gastritis takes an antacid drug for heartburn elimination.
After its ingestion the patient feels better but at the same time he has a sensation of stomach swelling.
Which of the following drugs might be the cause of such side effect?
A. Magnesium oxide
B. Magnesium trisilicate
C. Aluminium hydrooxide
D. * Sodium hydrocarbonate
E. Pepsin
97. Continuous use of a certain drug may cause osteoporosis, erosions of stomach mucosa, hypokaliemia,
retention of sodium and water in the organism, decreased concentration of corticotropin in blood.
What drug is it?
A. Hypothiazide
B. * Prednisolone
C. Digoxin
D. Indometacin
E. Reserpine
98. A 63-year-old patient with collapse presentations was delivered to the emergency hospital. A
physician has chosen noradrenalin against hypotension What is its mechanism of action?
A. Activation of serotonin receptors
B. Activation of beta-adrenoreceptors
C. * Activation of alpha1-adrenoreceptors
D. Activation of dopamine receptors
E. Block of M -cholinoreceptors
99. A patient suffering from syphilis has been treated with bismuth preparations. As a result of it some
grey spots turned up on the mucous membrane of the oral cavity; nephropathy symptoms were also
present. What drug should be used for treatment of bismuth intoxication?
A. Methylene blue
B. * Unithiol
C. Naloxone
D. Bemegride

E. Nalorphine
100. A patient suffering from coronary artery disease had taken a certain drug many times a day in order
to arrest stenocardia attacks. Overdose of this drug ?nally caused intoxication. Objectively: cyanotic
skin and mucous membranes, dramatic fall in the arterial pressure, tachycardia, respiration inhibition.
Blood has increased concentration of methemoglobin. The drug the patient had taken relates to the
following group:
A. Alpha-adrenoceptor blockers
B. Calcium channel blockers
C. * Organic nitrates
D. Adenosine drugs
E. Myotropic spamolytics
101. An elderly female patient suffers from the type 2 diabetes mellitus accompanied by obesity,
atherosclerosis, coronary artery disease. Basal hyperinsulinemia is also present. What treatment
would be the most appropriate?
A. Insulin
B. Retabolil
C. Lovastatin
D. * Glibenclamid
E. Amlodipine
102. In spite of treatment with cardiotonics and thiazide diuretic a patient suffering from chronic cardiac
failure still presents with edemata and faces a risk of ascites. What medication should be
administered in order to increase the diuretic effect of the above mentioned drugs?
A. * Spironolactone
B. Furosemide
C. Amiloride
D. Clopamide
E. Manithol
103. A patient presents with twilight vision impairment. Which of the following vitamins should be
administered?
A. Cyanocobalamin
B. Pyridoxine hydrochloride
C. * Retinol acetate
D. Ascorbic acid
E. Nicotinic acid
104. A female patient consulted a doctor about pain and limited movements in the knee joints. Which of
the following nonsteroid anti-inflammatory drugs should be administered taking into consideration
that the patient has a history of chronic gastroduodenitis?
A. Diclofenac sodium
B. Promedol
C. * Celecoxib
D. Acetylsalicilic acid
E. Butadiounum
105. A student came to see a doctor and asked to administer him a drug for treatment of allergic rhinitis
that occurs in the period of linden fowering. What drug may be used?
A. * Loratadine
B. Noradrenaline hydrotartrate
C. Propanolol
D. Ambroxol

E. Losartan
106. A patient consulted a physician about muscle rigidity, constrained movements, permanent arm
tremor. The patient was diagnosed with Parkinson’s disease. What preparation should be
administered?
A. Phenytoin
B. Phenobarbital
C. * Levodopa
D. Diazepam
E. Ethosuximide
107. A patient with coronary artery disease was admitted to the cardiological department. For stenocardia
prevention a drug from the group of beta-adrenoceptor blockers was administered. What drug is it?
A. * Metoprolol
B. Atropine sulfate
C. Morphine hydrochloride
D. Oxytocin
E. Furosemide
108. A patient with drug intoxication presented with the dryness of oral mucous membrane and mydriatic
pupils. Such action of this drug is associated with the following effect:
A. Muscarinic cholinoreceptor stumulation
B. Nicotinic cholinoreceptor stumulation
C. * Muscarinic cholinoreceptor block
D. Adrenoreceptor stimulation
E. Adrenoreceptor block
109. A patient with a limb fracture must be administered a depolarizing drug from the myorelaxant group
for the purpose of a short-time surgery. What drug is it?
A. * Dithylinum
B. Tubocurarine chloride
C. Cytitonum
D. Atropine sulfate
E. Pentaminum
110. A patient suffering from chronic bronchitis takes a synthetic mucolytic drug that facilitates the
sputum thinning. What drug is it?
A. * Acetylcysteine
B. Diazepam
C. Heparin
D. Furosemide
E. Enalapril
Назва наукового напрямку (модуля): Семестр: 6
Pharmacology (текстові тести) 2019-2020
Опис:
3 курс, Факультет іноземних студентів, Медицина
Перелік питань:
1. For high hepatic" first-pass metabolism" drugs, their bioavailability from patient to patient may vary
due to differences in:
A. Presence of kidney disease
B. * Activity of microsomals enzymes
C. Speed of absorption from intestine
D. Level of distribution
E. Onset
2. Antibiotics, that influence on bones and tooth formation and are forbidden in pediatrics
A. Penicyllines
B. Macrolides
C. Carbapenemes
D. Monobactames
E. * Tetracyclines
3. «Pharmacokinetics» means
A. Kinds of actions of medicinal matters
B. Effects, caused medicinal matters
C. Localizations actions of drugs
D. * Movement of drugs in the body
E. Mechanism of actions of medicinal matters
4. «Pharmacodynamics» means
A. Biotransformation of drugs
B. * Effects of drug on the body
C. Pharmacokinetics of drugs in an organism
D. Excretion of drugs from an organism
E. Absorption of drugs in an organism
5. Parenteral way of introduction of drugs
A. * Subcutaneous
B. Oral
C. Rectal
D. Sublingual
E. Intraduodenal
6. Enteral route of introduction of drugs
A. Intramuscular
B. * Rectal
C. Subcutaneous
D. Inhalation
E. Intravenous
7. Increasing of action at the repeated drug introduction is called
A. * Material accumulation
B. Tolerance
C. Physical dependence
D. Psychological dependence
E. Tachyphylaxis
8. Indicate complication in case of Streptomycin and Kanamycin usage:
A. Hypertension
B. * Ototoxicity and nephrotoxicity
C. Dryness of the oral cavity
D. Blood disorders
E. Inflammatory processes of different localization
9. Which of the following mechanism of action of drugs is correct?
A. * digoxin - inhibits Na+ - K+ ATPase pump
B. dobutamine - inhibits MAO
C. amrinone - inhibits ACE
D. dopamine - inhibits ACE
E. acebutolol - stimulate beta 2 adrenergic receptors
10. Calcium channel blocker for arrhythmia treatment is:
A. * Verapamil
B. Metoprolol
C. Dipiridamole
D. Aspirin
E. Captopril
11. Reflex tachycardia is associated with such group of drugs:
A. betablockers
B. * calcium channel blockers
C. beta 2-agonists
D. ACE-inhibitors
E. M-cholinoreceptors agonists
12. Reflex tachycardia is associated with such drug:
A. Propranolol
B. Metoprolol
C. * Nifedipine
D. Timolol
E. Salbutamol
13. Prinzmetal angina, variant angina, hypertension, paroxysmal supraventricular tachycardia - the
pathological statuses which need use of:
A. Lozartan
B. Captopril
C. Nitroglycerin
D. * Diltiazem
E. Dophamine
14. A drug of a first choice for acute angina attack is:
A. Atenolol
B. Captopril
C. Diltiazem
D. Dophamine
E. * Nitroglycerin
15. Antihistaminic agent that does not produce sedation is:
A. Suprastin

B. Diprasine
C. Dimedrol
D. Paracitamol
E. * Diazolin
16. The direct-acting plasminogen activator, fibrinolitic drug is:
A. streptomycin
B. * urokinase
C. epsilon-aminocaproic acid
D. vikasol
E. heparin
17. The antithrombotic drug which inhibits cyclooxygenase enzymes is:
A. prednisone
B. dipyridamole
C. ascorbic acid
D. * aspirin
E. vikasol
18. Correct statements about the action of warfarin include:
A. warfarin is not structurally similar to vitamin K, warfarin inhibits vitamin K epoxide reductase, the
onset of warfarin action may be prolonged due to the long half-life of factor VII;
B. warfarin is not structurally similar to vitamin K, the onset of warfarin action may be prolonged due to
the long half-life of factor VII;
C. * warfarin inhibits vitamin K epoxide reductase, it slows down the blood's normal clotting ability;
D. strong headache appear if warfarin is administered in therapeutic doses;
E. All of the above
19. Manifestations of overdose with dimedrol may include:
A. blurred vision
B. dry mouth
C. hallucinations
D. sinus tachycardia
E. * All answers are correct
20. Histamine-1 receptor antagonists include:
A. ranitidine
B. nizatidine
C. famotidine
D. * loratidine
E. cimetidine
21. Gallstones, abdominal pain and myalgia are side effects of such hypolipidemic drug:
A. neomycin
B. aspirine
C. levamisole
D. ascorbic acid
E. * clofibrate
22. Hypolipidemic drug which indirectly enhance the activity of the hepatic enzyme HMG CoA
reductase include:
A. paracetamol
B. L-thyroxine
C. * cholestyramine

D. levomycetine
E. gentamycine
23. Agents that are mainly used to treat hypercholesterolemia, which act by inhibiting the HMG-CoA
reductase enzyme include:
A. * lovastatin
B. gentamycine
C. nialamide
D. ascorbic acid
E. amlodipine
24. The agent which reduces triglyceride levels is:
A. neomycin
B. * nicotinic acid
C. fluconasole
D. coffeine
E. beta adrenoblocker
25. Gynecomastia, hyperkalemia, and menstrual disorders are potential adverse effects associated with
such diuretic drug:
A. furosemide
B. captopril
C. acebutolol
D. * spironolactone
E. digitoxine
26. Diuretic which may result hypokalemia when used in a patient not receiving potassium
supplementation include:
A. Spironolactone
B. * Furosemide
C. Triamterene
D. Potassium chloride
E. None of the above
27. An agent that causes an antihypertensive effect by blocking autonomic ganglia, thus reducing the
sympathetic tone of blood vessels, is:
A. * hygronium
B. diazoline
C. aspirine
D. nifedipine
E. alpha-methyldopa
28. The most useful drugs in the long-term treatment of essential hypertension, which decrease the
quantity of sodium ions in blood vessels and eliminate them via the kidneys are:
A. * thiazide diuretics
B. osmotic diuretics
C. sympatholitics
D. sedative agents
E. cardiac glycosides
29. Tachycardia is a common side effect of:
A. * nifedipine
B. aspirine
C. acebutolol

D. all of above
E. none of above
30. A diuretic agent use which has been associated with hearing loss is:
A. spironolactone
B. triamterene
C. * furosemide
D. amiloride
E. acetazolamide
31. The potassium sparing diuretic agent which acts by inhibiting aldosterone at the distal tubular renal
cell is:
A. * spironolactone
B. furosemide
C. ethacrynic acid
D. chlorothiazide
E. indapamide
32. A diuretic agent which is potassium-sparing and the action of which does not depend on the adrenal
cortex is:
A. furosemide
B. * triamterene
C. ethacrynic acid
D. chlorothiazide
E. indapamide
33. Which of the following diuretic agents may cause hypokalemic alkalosis but it is a highly effective
drug for the treatment of volume-dependent hypertension?
A. spironolactone
B. triamterene
C. aspirine
D. * dichlothiazide
E. teophylline
34. Which of the following diuretics may cause hyperkalemia:
A. ethacrynic acid
B. furosemide
C. thiazides
D. * triamterene
E. indapamide
35. Triamterene:
A. antagonizes aldosterone
B. * can produce systemic hyperkalemia
C. is classified as a sodium-sparing diuretic
D. has no clinical use
E. has diuretic activity as furosemide
36. Which of the following drugs may be dangerous if given to a patient receiving spironolactone?
A. epinephrine
B. propranolol
C. * potassium chloride
D. monoamine oxidase inhibitors
E. All of the above

37. A side effect of spironolactone is:


A. * hyperkalemia
B. acne
C. hypomagnesemia
D. male pattern baldness
E. bronchospasm
38. The main site of action of furosemide is the:
A. * Ascending loop of Henle
B. All of above
C. Proximal tubule
D. Distal tubule
E. None of the above
39. The site of action of mannitol is:
A. Loop of Henle
B. All of above
C. * Tubule of kidney
D. Liver
E. None of the above
40. Antihistaminic preparation with somnolent effect is:
A. Phenobarbital
B. * Dimedrole
C. Sodium oxybutas
D. Bromisovale
E. Diazepam
41. What is the drug of choice for treatment of anaphylactic shock:
A. Anapriline
B. * Adrenaline
C. Morphine
D. Codeine
E. Analgine
42. Mechanism of anti-allergic actions of antihistamines is:
A. Inactivation of histamine
B. Block cholinoreceptor of peripheral ganglions
C. * Block H1-histamine receptors
D. Activate H2-histamine receptors
E. Diminish formation of antibodies
43. Antihistaminic drug without sedative effect is:
A. Tavegil
B. * Loratidine
C. Dimedrole
D. Suprastine
E. Diprasine
44. Which diuretic agent is the most effective in a patient with primary hyperaldosteronism?
A. * Spironolactone
B. Ethacrynic acid
C. Furosemide

D. Chlorothiazide
E. Indapamide
45. Nitroglycerin is not given orally because it
A. * has a high first-pass metabolism
B. has a higher risk of thrombosis
C. has a high liver toxicity
D. has a very short latent period
E. can cause a cholestasis
46. Which of the following routes of drug administration bypasses the GI tract?
A. Intravenous
B. Inhalation
C. * All of indicated
D. Intramuscular
E. Subcutaneously
47. Which of the following groups of drugs is used as nasal decongestants in rhinitis?
A. * Alpha1-agonists
B. Beta2-agonists
C. M-cholinomimetics
D. Ganglion blockers
E. Beta-adrenoblockers
48. Hyperactivity of cyclooxygenase 2 is only associates with
A. * inflammation
B. depression
C. constipation
D. mucosa protection
E. low blood pressure
49. Which of the following drugs may cause xerostomia?
A. * Atropine
B. Norepinephrine
C. Dopamine
D. Galantamine
E. Pilocarpine
50. Which of the following drugs is used for relaxing bronchial muscles in case of bronchial asthma?
A. * Salbutamol
B. Timolol
C. Reserpine
D. Dobutamine
E. Norepinephrine
51. Which of the following should be limited in patients taking a tricyclic antidepressant?
A. * Epinephrine
B. Acetylcholine
C. Dopamine
D. Serotonin
E. Papaverinum
52. Indicate the anticholinergic drug which causes mydriasis, tachycardia and increases the intraocular
pressure.

A. * Atropine sulfate
B. Aceclidine
C. Norepinephrine
D. Dopamine
E. Galanthamine
53. Choose inotropic agent which improves contractility of the myocardium in patient with congestive
heart failure without increasing of the heart rate.
A. * Dobutamine
B. Norepinephrine
C. Atenolol
D. Caffeine benzoate sodium
E. Sodium hydrocarbonate
54. Which drug consists of amylase, lipase and protease and is used for treatment of chronic pancreatitis?

A. * Pancreatin
B. Vikasol
C. Propranolol
D. Caffeine
E. Ethyl alcohol
55. This antihypertensive drug is contraindicated in patients with bilateral renal artery stenosis:
A. * Lisinopril
B. Propranolol
C. Prednisolone
D. Paracetamol
E. Terazosin
56. Choose the medicine for treatment of schizophrenia with concomitant psycho-motor excitement.
A. * Aminazine
B. Cyclodolum
C. Nacom
D. Nitrazepam
E. Amitriptylinum
57. In patient with rheumatism after long-term administration of NSAIDs leukopenia is diagnosed. What
drug may be used for the adverse reaction?
A. * Methyluracilum
B. Heparinum
C. Vicasolum
D. Cyanocobalaminum
E. Phthalazolum
58. What complication may occur as a result of incorrect introduction of calcium chloride solution?
A. * Necrosis
B. Abscess
C. Hematoma
D. Lipodystrophy
E. Drug embolism
59. What medicine should be given for a teenager with food allergy phenomenon?
A. * Loratadine
B. Mannitol

C. Methyluracilum
D. Clopheline
E. Metronidazol
60. Indicate a laxative drug, which is most effective for treatment chronic constipation:
A. * Bisacodyl
B. Magnesium sulfate
C. Cholosasum
D. Panangin
E. Loperamide
61. Which cholinomimetic is commonly used in the treatment of glaucoma?
A. * Pilocarpini hydrochloridum
B. Lobelini hydrochloridum
C. Cytitone
D. Dibazole
E. Galanthamini hydrobromidum
62. Which of the following cholinomimetics is a drug of choice for reversing the effects of
nondepolarizing neuromuscular relaxants?
A. * Proserine (Physostigmine)
B. Lobelini hydrochloridum
C. Pilocarpini hydrochloridum
D. Methacine
E. Phosphacolum
63. Indicate the skeletal muscle relaxant with a depolarizing mechanism of action:
A. * Dithylline
B. Pipecuronii bromidum
C. Benzohexonium
D. Tubocurarini chloridum
E. Amizyle
64. Indicate an antimuscarinic drug, which is effective in the treatment of mushroom poisoning:
A. * Atropini sulfas
B. Scopolamini hydrobromidum
C. Alloxinum
D. Carbacholinum
E. Aceclidinum
65. Permanent stains on a patient?s teeth may be the result of taking which drug, administered during the
period of tooth formation in a child
A. * doxycycline
B. amoxicillin
C. cephalexin
D. amoxiclav
E. penicillin V
66. An overdose of acetaminophen may cause damage to the which organ
A. * liver
B. eye
C. ear
D. lungs
E. tongue

67. Which drug is commonly used to prevent or to relieve an attack of angina pectoris?
A. * Nitroglycerin
B. Warfarin
C. Theophylline
D. Hidazepam
E. Tripsinum
68. Patients with congestive heart failure who suffer from edema due to increased water retention which
drug may be prescribed?
A. * diuretic
B. beta blocker
C. calcium channel blocker
D. alpha blocker
E. antibiotic
69. Acute poisoning with Atropine sulfate includes all of the following symptoms except:
A. * Bradycardia, orthostatic hypotension
B. Hyperthermia, hot and flushed skin
C. Dry mouth
D. Agitation and delirium
E. Mydriasis, cycloplegia
70. Epinephrine produces all of the following effects except:
A. * Decrease in oxygen consumption
B. Increase in oxygen consumption
C. Bronchodilation
D. Hyperglycemia
E. Mydriasis
71. Before the operation of the maxillofacial area dentist to reduce the secretion of salivary glands and to
prevent broncho- and laryngospasm used which group of drug:
A. * M-cholinoblockers
B. Muscle relaxants
C. Sympatholytics
D. Adrenergic agonists
E. Adrenergic antagonists
72. Drug of choice in the treatment of immediate allergic reactions
A. Analeptics
B. * Adrenomimetics
C. Sympathomimetics
D. Cholinomimetics
E. NSAID
73. Hormonal preparation used in case of immediate allergic reactions is:
A. Insulin
B. Tireoidine
C. Vasopressine
D. * Dexametasone
E. Cromoline sodium
74. Indicate, what is typical for Diazoline
A. Acts shortly
B. Connected with H2-histamine receptors

C. Stimulate secretion of salivary glands


D. * Does not penetrate through BBB (blood brain barrier)
E. Possesses expressed sedative action
75. Cromoline sodium mechanism of action is
A. * Stabilization of mast cells membranes
B. Block H1-receptors
C. Block H2-receptors
D. Bronchodilator action
E. Stimulation of adrenergic receptors
76. Hormonal preparation used in case of anaphylactic shock is:
A. Anaprilin
B. * Prednisolone
C. Morphine
D. Codeine
E. Analgin
77. Antihistaminic agents of III generations, does not have sedative action is:
A. Dimedrole
B. Diprasine
C. Suprastine
D. * Loratadine
E. Phencarole
78. Antihistaminic drug - phenothiazine derivative with alpha - adrenoblocking activity is:
A. Dimedrole
B. Diazoline
C. Suprastine
D. * Diprasine
E. Phencarole
79. Adverse effect of antihistaminic preparations of I generation
A. Excitation of the central nervous system
B. * Dryness in oral cavity
C. Reduction the effect of sedative drugs
D. Acceleration of psychiatry reactions
E. Cramps
80. Diazoline unlike Dimedrole:
A. Causes a somnolent effect
B. Acts shortly
C. * Does not penetrate through BBB (blood brain barrier)
D. Possesses expressed cholinolitic action
E. Blocks histamine receptors
81. Calcium channel blocker for tachyarrhythmia is
A. * Verapamil
B. Metoprolol
C. Dipiridamole
D. Aspirin
E. Captopril
82. High degree of reflex tachycardia is associated with

A. * Calcium channel blockers


B. Beta - blockers
C. Beta 2-agonists
D. ACE-inhibitors
E. M-cholinoreceptors agonists
83. High degree of reflex tachycardia is associated with
A. Propranolol
B. Metoprolol
C. * Nifedipine
D. Timolol
E. Salbutamol
84. Prinzmetal?s angina, variant angina, hypertension, paroxysmal supraventricular tachycardia are the
pathological statuses which need uses of
A. Lozartan
B. Captopril
C. Nitroglycerin
D. * Diltiazem
E. Dophamine
85. A drug of a first choice for acute angina attack is:
A. Propranolol
B. Captopril
C. Diltiazem
D. Dophamine
E. * Nitroglycerin
86. The period of onset of reserpine is:
A. 2-3 weeks after initiation of therapy
B. Immediate
C. * 2-5 days after initiation of therapy
D. 1-3 hours after initiation of therapy
E. 1 month after initiation of therapy
87. Which calcium channel blocker have a long half-life and can be dosed once a day?
A. Nifedipine
B. * Amlodipine
C. Nitroglycerin
D. Atropine
E. Dophamine
88. ACE-inhibitor is
A. Nitroglycerin
B. Nifedipine
C. * Captopril
D. Metoprolol
E. Hygronium
89. The drug of the pharmacological group of nitrates is:
A. * Nitroglycerin
B. Nifedipine
C. Captopril

D. Metoprolol
E. Hygronium
90. Calcium channel blocker is
A. Nitroglycerin
B. * Nifedipine
C. Captopril
D. Metoprolol
E. Hygronium
91. Beta-adrenoreceptors antagonist is
A. Nitroglycerine
B. Nifedipine
C. Captopril
D. * Metoprolol
E. Hygronium
92. What is the mechanism of action of angiotensin II receptor antagonists?
A. * Competitively bind to the angiotensin II receptors
B. Competitively bind to the beta-adrenoreceptors
C. Competitively bind to the alpha-adrenoreceptors
D. Inhibit angiotensin-converting enzyme (ACE)
E. Dilate the veins
93. What is mechanism of action of ACE-inhibitors
A. Competitively bind to the angiotensin II receptors
B. Competitively bind to the beta-adrenoreceptors
C. Competitively bind to the alpha-adrenoreceptors
D. * Inhibit angiotensin-converting enzyme (ACE)
E. Dilatation the veins
94. What is mechanism of action of nitrates for treatment of ischemic heart disease?
A. Competitively bind to the angiotensin II receptors
B. Competitively bind to the beta-adrenoreceptors
C. Competitively bind to the alpha-adrenoreceptors
D. Inhibit angiotensin-converting enzyme (ACE)
E. * Dilatation of the peripheral vessels
95. Therapeutic use of calcium channel blockers is
A. Therapy of hypertension
B. Therapy of angina
C. Therapy of arrhythmias
D. Prevention of heart?s ischemia
E. * All of above
96. Nifedipine is:
A. * Calcium channel blocker
B. Beta-adrenoreceptor antagonist
C. Beta- adrenoreceptor agonist
D. ACE inhibitor
E. Angiotensin II receptors antagonists
97. Amlodipine is
A. * Calcium channel blocker

B. Beta-adrenoreceptor antagonist
C. Beta- adrenoreceptor agonist
D. ACE inhibitor
E. Angiotensin II receptors antagonists
98. Verapamil is
A. * Calcium channel blocker
B. Beta-adrenoreceptor antagonist
C. Beta-adrenoreceptor agonist
D. ACE inhibitor
E. Angiotensin II receptor antagonists
99. Verapamil is used for:
A. * tachyarrhythmia
B. congestive heart failure
C. bronchial asthma
D. bronchitis
E. hepatitis
100. Adverse effect of calcium channel blockers is:
A. * Flushing (dilatation of skin vessels)
B. Hypertension
C. Bronchial spasm
D. Heart ischemia
E. Dry mouth
101. The dihydropyridines - calcium channel blockers including nifedipine, amlodipine that:
A. * Have prominent vascular effects, rather than cardiac effects
B. Have prominent cardiac effects, rather than vascular effects
C. Have antiarrythmic effect
D. Have antiallergic action
E. Are used for headache
102. Which calcium channel blockers are most effective to control supraventricular arrhythmias?
A. * Verapamil, diltiazem
B. Verapamil, atropine
C. Diltiazem, nifedipine
D. Verapamil, metoprolol
E. Metoprolol, lidocaine
103. Prinzmetals angina is due to coronary vasospasms, the most effective group of drugs for treatment is:
A. Alpha-adrenoreceptor agonists
B. * Calcium channel blockers
C. Beta-adrenoreceptor agonists
D. ACE inhibitors
E. Angiotensin II receptor antagonists
104. Prinzmetal's angina is due to coronary vasospasms, the most effective drug for treatment is:
A. * Amlodipine
B. Nitroglycerin
C. Metoprolol
D. Captopril
E. Losartan

105. Sublinglival nitroglycerin is used for:


A. * Acute angina attacks
B. Acute bronchospasm
C. Arrhythmias
D. Hepatitis
E. Bronchitis
106. What is the mechanism of action of class I antiarrhythmic drugs?
A. * Block sodium channels
B. Block alpha-adrenoreceptors
C. Increase intracellular potassium amount
D. Block calcium channels
E. Block beta-adrenoreceptors
107. What is the mechanism of action of class II antiarrhythmic drugs?
A. Block sodium channels
B. Block alpha-adrenoreceptors
C. Increase intracellular potassium amount
D. Block calcium channels
E. * Block beta-adrenoreceptors
108. What is the mechanism of action of class III antiarrhythmic drugs?
A. Block sodium channels
B. Block alpha-adrenoreceptors
C. * Block potassium channels
D. Block calcium channels
E. Block beta-adrenoreceptors
109. What is the mechanism of action of class IV antiarrhythmic drugs?
A. Block sodium channels
B. Block alpha-adrenoreceptors
C. Block potassium channels
D. * Block calcium channels
E. Block beta-adrenoreceptors
110. What is the mechanism of action of cardiac glycosides?
A. Block the calcium channels
B. Block the beta-adrenoreceptors
C. * Block the activity of adenosine-triphosphatase (ATP-ase)
D. Block the activity of COX2
E. Cause the dilatation of coronary vessels
111. What is the mechanism of positive ionotropic effect of cardiac glycosides?
A. * Increase the force of contraction of the cardiac muscles
B. Decrease the force of contraction of the cardiac muscles
C. Increase the sensitivity of heart to epinephrine
D. Cause tachycardia
E. Cause the dilatation of heart vessels
112. What anticoagulant activates antithrombin III?
A. * Heparin
B. Warfarin
C. Aspirin

D. Alteplase
E. Streptokinase
113. Heparin is
A. * Anticoagulant
B. Analgesic
C. Cardiac glycoside
D. Antianginal medicine
E. Drug for treatment of bronchial asthma
114. Drugs that inhibit the clotting cascade, thus preventing clot formation are:
A. * Anticoagulants
B. Nitrates
C. Coagulants
D. Antihypertensive drugs
E. Cardiac glycosides
115. Sulfonylurea drugs for diabetes mellitus:
A. are active in the absence of the pancreas
B. * increase insulin release
C. are given subcutaneously
D. are used for type 1 diabetes
E. are used for bronchial asthma
116. During a treatment of arterial hypertension in a patient develops a drug-induced dry cough. The drug
which possibly caused this cough is:
A. Propanolol
B. Indomethacine
C. Nitroglycerin
D. * Captopril
E. Acetaminophen
117. Antithyroid drugs (Mercazolil, Propilthyouracil):
A. inhibit the effects of thyroid hormones in peripheral tissues
B. * inhibit the synthesis of thyroid hormones
C. are used for treatment of hypothyroidism
D. are used for treatment of congestive heart failure
E. are used for treatment of glaucoma
118. Oxytocin
A. is a steroid hormone
B. * causes uterine contractions
C. is used to prevent preterm delivery
D. is used to prevent heart ischemia
E. is used for depression
119. International drug names which ends with the suffix -pril are:
A. Cardiac glycosides
B. Antiarrhythmics
C. * ACE (angiotensin converting enzyme) inhibitors
D. Agonists of alpha-adrenergic receptors
E. Laxative agents
120. International drug names which ends with the suffix -sartan are:

A. Antineoplastic agents
B. Laxatives
C. Antivomiting drugs
D. * Angiotensin II receptor antagonists
E. Opioid analgesics
121. The drug that is used in the case of increased sensitivity to nitroglycerin for removing attack of
angina pectoris is:
A. Aspirin
B. Atropine
C. Piracetame
D. Papaverine
E. * Molsidomine
122. Medication that inhibits blood clot formation, a phosphodiesterase inhibitor that blocks uptake and
metabolism of adenosine is:
A. Papaverine
B. Propranolol
C. Reserpine
D. * Dipiridamole
E. Trental
123. The cardioselective beta-adrenoblocker is:
A. * Atenolol
B. Adrenalin
C. Riboxinum
D. Enalapril
E. Nifedipine
124. The direct-acting plasminogen activator, fibrinolitic drug is:
A. streptomicine
B. * urokinase
C. epsilon-aminocaproic acid
D. vikasol
E. heparin
125. The antithrombotic drug which inhibits cyclooxygenase enzymes is:
A. prednisone
B. dipyridamole
C. ascorbic acid
D. * aspirin
E. vikasol
126. Manifestations of overdose with dimedrole may include:
A. blurred vision
B. dry mouth
C. hallucinations
D. sinus tachycardia
E. * all of the above
127. Histamine-1 receptor antagonists include:
A. ranitidine
B. nizatidine
C. famotidine

D. * terfenadine
E. cimetidine
128. Gallstones, abdominal pain and myalgia are side effects of such hypolipidemic drug:
A. neomycin
B. aspirine
C. levamisole
D. ascorbinic acid
E. * clofibrate
129. Hypolipidemic Drug which indirectly enhance the activity of the hepatic enzyme HMG CoA
reductase include:
A. paracetamol
B. L-thyroxine
C. * cholestyramine
D. levomycetine
E. gentamycine
130. Gynecomastia, hyperkalemia, and menstrual disorders are potential adverse effects associated with
such diuretic drug:
A. furosemide
B. captopril
C. acebutolol
D. * spironolactone
E. digitoxine
131. Diuretic which may result hypokalemia when used in a patient not receiving potassium
supplementation include:
A. spironolactone
B. * furosemide
C. triamterene
D. potassium chloride
E. none of the above
132. An agent that causes an antihypertensive effect by blocking autonomic ganglia, thus reducing the
sympathetic tone of blood vessels, is:
A. * hygronium
B. diazoline
C. aspirine
D. nifedipine
E. alpha-methyldopa
133. The most useful drugs in the long-term treatment of essential hypertension, which decrease the
quantity of sodium ions in blood vessels and eliminate them via the kidneys are:
A. * thiazide diuretics
B. osmotic diuretics
C. sympatholitics
D. sedative agents
E. spasmolytics of myotropic action
134. Tachycardia is a common side effect of:
A. * nifedipine
B. aspirine
C. acebutolol

D. all of above
E. none of above
135. A diuretic agent use which has been associated with hearing loss is:
A. spironolactone
B. triamterene
C. * furosemide
D. amiloride
E. acetazolamide
136. A diuretic agent which is antagonist of aldosterone, it is potassium sparing:
A. indapamide
B. * spironolactone
C. furosemide
D. ethacrynic acid
E. chlorothiazide
137. The potassium sparing diuretic agent which acts by inhibiting aldosterone at the distal tubular renal
cell is:
A. * spironolactone
B. furosemide
C. ethacrynic acid
D. chlorothiazide
E. indapamide
138. A diuretic agent which is potassium-sparing and the action of which does not depend on the adrenal
cortex is:
A. furosemide
B. * triamterene
C. ethacrynic acid
D. chlorothiazide
E. indapamide
139. Which of the following diuretic agents may cause hypokalemic alkalosis but it is a highly effective
drug for the treatment of volume-dependent hypertension?
A. spironolactone
B. triamterene
C. aspirine
D. * dichlothiazide
E. teophylline
140. Which insulin has the longest duration?
A. Regular
B. Lente
C. NPH
D. * Ultra Lente
E. Don’t answers are correct
141. Which statement describes the mechanism of action of rosiglitazone (Avandia)?
A. It stimulates beta cells to produce insulin
B. * It decreases insulin resistance
C. It inhibits hepatic glucose production
D. It increases the sensitivity of peripheral tissue to insulin
E. Don’t answers are correct

142. In which of the following situations would androgens be prescribed for a woman?
A. Development of secondary sex characteristics
B. Postmenopausal osteoporosis prevention
C. Ovarian cancer
D. * Treatment of endometriosis
E. Don’t answers are correct
143. Which of the following is a common side effect of antitussive medications?
A. * Drowsiness and dizziness
B. Diarrhea and abdominal cramping
C. Tremors and anxiety
D. Headache and hypertension
E. Don’t answers are correct
144. Which of the following medications is a nonsedating antihistamine?
A. * loratadine (Claritin)
B. diphenhydramine (Benadryl)
C. dimenhydrinate (Dramamine)
D. meclizine (Antivert)
E. don’t answers are correct
145. Antitussives are used primarily to:
A. Relieve nasal congestion
B. Thin secretions to ease removal of excessive secretions
C. * Stop the cough reflex when the cough is nonproductive
D. Suppress productive and non-productive coughs
E. Don’t answers are correct
146. Discharge teaching to a patient receiving a beta-agonist bronchodilator should emphasize reporting
which of the following side effects?
A. * Tachycardia
B. Nonproductive cough
C. Hypoglycemia
D. Sedation
E. Don’t answers are correct
147. Nonselective adrenergic agonist bronchodilators should be used cautiously in patients with:
A. * Coronary artery disease
B. COPD
C. Hyperkalemia
D. Premature labor
E. Don’t answers are correct
148. Which of the following respiratory medications is an antileukotriene agent?
A. Isoproterenol (Isuprel)
B. Epinephrine (Adrenalin)
C. Cromolyn sodium (Intal)
D. * Montelukast (Singulair)
E. Don’t answers are correct
149. When teaching a patient about cromolyn sodium, the doctor stresses that:
A. * Cromolyn is not effective in treating an acute asthma attack
B. The capsule should not be opened when administered orally

C. Cromolyn can only be given at bedtime secondary to sedating side effects


D. They can expect a therapeutic response within 12 hours
E. Don’t answers are correct
150. A priority assessment of a patient on antibiotic therapy includes questioning the patient about:
A. History of seizure activity
B. Cardiac history
C. * Allergies
D. Immunizations
E. One answer is correct
151. If tetracycline is administered to children under 8 years of age, it can cause:
A. Delayed achievement of developmental milestones
B. Neurotoxicity
C. GI bleeding
D. * Permanent discoloration of the teeth
E. Don’t answers are correct
152. You should assess a 75-year-old patient for nephrotoxicity and ototoxicity when administering which
of the following antimicrobials?
A. Cefazolin (a cephalosporin antibiotic)
B. Clindamycin (a lincosamide antibiotic)
C. * Gentamicin (an aminoglycoside antibiotic)
D. Erythromycin (a macrolide antibiotic)
E. Don’t answers are correct
153. A common drug used to treat candidal infections is:
A. * Nystatin
B. Oseltamivir
C. Amantadine
D. Griseofulvin
E. None answer is correct
154. Which of the following antifungal agents can be given intravenously to treat severe yeast infections
as well as a one-time oral dose to treat vaginal yeast infections?
A. nistatin (Mycostatin)
B. caspofungin (Cancidas)
C. voriconazole (Vfend)
D. * fluconazole (Diflucan)
E. none answer is correct
155. Which of the following would be used to treat thrash in an infant?
A. amphotericin В
B. fluconazole
C. * nystatin
D. miconazole
E. none answer is correct
156. Which medication is often used as a 1-day dose treatment of vaginal candidiasis?
A. nistatine
B. * fluconazole
C. griseofulvin
D. imidazole
E. none answer is correct

157. Which of is the following agent of choice for treatment of many severe systemic fungal infections?
A. * amphotericin В
B. fluconazole
C. griseofulvin
D. flucytosine
E. none answer is correct
158. A patient taking an anthelmintic drug should be instructed to notify the physician immediately if he
or she experiences:
A. * Darkened urine
B. Light-colored stools
C. Anal irritation
D. All of the above
E. None answer is correct
159. Why would a sulfonamide or tetracycline be given with quinine therapy in mild cases of malaria?
A. The antibiotic treats bacterial infections that accompany malaria
B. The antibiotic reduces the severe side effects of quinine
C. * These antibiotics have a synergistic effect with quinine
D. The antibiotic therapy is also needed to kill the parasite that causes malaria
E. None answer is correct
160. Which of the following is true regarding patients receiving anthelmintic therapy?
A. The medication can be stopped once symptoms disappear.
B. * The medication must be taken exactly as ordered for the length of time ordered
C. Anthelmintics are broad in their actions and can be substituted easily if one medication is not
well-tolerated.
D. Anthelmintics are more effective in their parenteral forms.
E. None answer is correct
161. A patient receiving rifampin must be told that:
A. * A nonharmful side effect of the drug is red-orange discoloration of urine, sweat, and tears
B. Oral contraception is the preferred method of birth control when using rifampin
C. Peripheral neuropathy is an expected side effect, and the patient should report any numbness or
tingling of the extremities
D. The patient will only need to take this medication for the prescribed 14-day period
E. None answer is correct
162. The doctor would explain to family members exposed to tuberculosis that they can expect to take
prophylactic medication for:
A. 2 to 4 weeks
B. 2 to 4 months
C. * 6 months to 1 year
D. 2 to 3 years
E. None answer is correct
163. What would you expect to include in the teaching plan for a patient who is taking izomiazide (INH)?
A. Urine and saliva may be reddish-orange in color
B. * Pyridoxine may be needed to prevent neurotoxicity
C. Injection sites should be rotated daily
D. Take medications with an antacid to reduce gastric distress
E. None answer is correct
164. Most nonsteroidal antiinflammatory drugs (NSAIDs) work by:

A. Ameliorating pain perception


B. * Inhibiting prostaglandin production
C. Increasing blood flow to painful areas
D. Increasing the supply of natural endorphins
E. None answer is correct
165. The advantage of cyclooxygenase-2 (COX-2) inhibitors over other NSAIDs is that they:
A. Have a longer duration of action
B. Are less likely to cause hepatic toxicity
C. * Do not decrease the cytoprotective lining of the stomach
D. Have a more rapid onset of action
E. None answer is correct
166. A common side effect of NSAID therapy is:
A. diarrhea
B. * gastrointestinal distress
C. hypotension
D. palpitations
E. none answer is correct
167. Contraindications for the use of NSAIDs include:
A. pericarditis
B. osteoarthritis
C. * bleeding disorders
D. juvenile rheumatoid arthritis
E. none answer is correct
168. General adverse effects of chemotherapy include which of the following? (choose all that apply)
A. Leukocytosis
B. * Thrombocytopenia, alopecia
C. Urinary retention
D. All answers are correct
E. None answer is correct
169. Nausea and vomiting are frequent adverse effects associated with antineoplastic therapy. Patients
who are experiencing this unpleasant effect should be advised:
A. To avoid eating any food during chemotherapy
B. That there is nothing that can be done for this effect and it will pass with time
C. * To try to maintain hydration and nutrition, which are very important during treatment
D. To use antacids to relieve the irritation to the stomach, which should stop the nausea
E. None answer is correct
170. A major dose-limiting side effect of doxorubicin (Adriamycin) is:
A. Hemorrhagic cystitis
B. * Cardiomyopathy
C. Hepatoxicity
D. Nephrotoxicity
E. None answer is correct
171. The doctor would suspect which type of cancer in a patient receiving tamoxifen?
A. Lung cancer
B. Renal cancer
C. * Breast cancer

D. Colon cancer
E. None answer is correct
172. Combinations of chemotherapeutic agents are frequently used today in order to:
A. * prevent drug resistance
B. reduce the incidence of side effects
C. decrease the cost of treatment
D. reduce treatment time
E. none answer is correct
173. Which agent is used to stimulate the production of red blood cells?
A. filgrastim
B. * epoetin
C. sargramostim
D. interferon beta-la
E. none answer is correct
174. Immunosuppressants, in general, work by:
A. Increasing antibody response
B. Increasing natural killer (NK) cellular activity
C. * Suppressing T-lymphocytes
D. Suppressing hepatic metabolism of steroids
E. None answer is correct
175. The doctor would teach patients that antacids are effective in treatment of hyperacidity because they:
A. * Neutralize gastric acid
B. Decrease stomach motility
C. Decrease gastric pH
D. Decrease duodenal pH
E. None answer is correct
176. The doctor would monitor for which of the following adverse reactions to aluminum-containing
antacids such as aluminum hydroxide (Amphojel)?
A. Diarrhea
B. * Constipation
C. GI upset
D. Fluid retention
E. None answer is correct
177. Histamine-2-receptor antagonists:
A. * Compete with histamine for binding sites on the parietal cells
B. Irreversibly bind to H+/K+ATPase
C. Cause a decrease in stomach pH
D. Decrease signs and symptoms of allergies related to histamine release
E. None answer is correct
178. Neuroleptics are used to treat:
A. Neurosis
B. * Psychosis
C. Narcolepsy
D. Parkinsonian disorders
E. Don't answers are correct
179. Parkinsonian symptoms and tarditive dyskinesia are caused by blockade dopamine in:

A. * The nigrostriatal system


B. The mesolimbic and mesofrontal systems
C. The chemoreceptor trigger zone of the medulla
D. The tuberoinfundibular system
E. Don't answers are correct
180. Which of the following antipsychotic drugs is typical?
A. Clozapine
B. Quetiapine
C. * Haloperidol
D. Olanzapine
E. Don't answers are correct
181. Indicate the atypical antipsychotic drug:
A. Haloperidol
B. * Clozapine
C. Aminazine
D. Amitryptilline
E. Don't answers are correct
182. Indicate the antipsychotic drug, which is a phenothiazine aliphatic derivative:
A. Thiothixene
B. Risperidone
C. * Chlorpromazine
D. Clozapine
E. Don't answers are correct
183. Indicate the antipsychotic drug, which is a butyrophenone derivative:
A. * Droperidol
B. Thioridazine
C. Sertindole
D. Fluphenazine
E. Don't answers are correct
184. Indicate the antipsychotic drug, which is a thioxanthene derivative:
A. Haloperidol
B. Clozapine
C. Chlorpromazine
D. * Thiothixene
E. Don't answers are correct
185. Phenothiazine derivatives (chlorpromazine) are able to:
A. * Course hypothermia
B. Decrease levels of prolactin
C. Increase corticotrophin release and secretion of pituitary growth hormone
D. Decrease appetite and weight
E. Don't answers are correct
186. Most phenothiazine derivatives (chlorpromazine) have:
A. Antihistaminic activity
B. Anticholinergic activity
C. Antidopaminergic activity
D. * All of the above

E. Don't answers are correct


187. Indicate the antipsychotic drug having significant peripheral alpha-adrenergic blocking activity:
A. Haloperidol
B. * Chlorpromazine
C. Clozapine
D. Risperidone
E. Don't answers are correct
188. Indicate the antipsychotic drug having a muscarinic-cholinergic blocking activity:
A. * Chlorpromazine
B. Clorprothixene
C. Risperidone
D. Haloperidol
E. Don't answers are correct
189. Parkinsonos syndrome, acute dystonic reactions, tardive dyskinesia, antimuscarinic actions,
orthostatic hypotension, galactorrhea are possible adverse effects of:
A. Haloperidol
B. Clozapine
C. * Chlorpromazine
D. Risperidone
E. Don't answers are correct
190. Adverse peripheral effects, such as loss of accommodation, dry mouth, tachycardia, urinary retention,
constipation develop due to:
A. Alpha adrenoreceptor blockade
B. * Muscarinic cholinoreceptor blockade
C. Beta adrenoreceptor blockade
D. Dopamine receptor blockade
E. Don't answers are correct
191. Which of the following drugs is phenothiazine derivatives and a potent local anesthetic?
A. Xycaine
B. Procaine
C. * Chlorpromazine
D. Atropine sulfate
E. Don't answers are correct
192. Which of the following phenothiazine derivatives may produce cardiac toxicity, including ventricular
arrhythmias, cardiac conduction block, and sudden death?
A. * Thioridazine
B. Chlorpromazine
C. Perphenazine
D. Fluphenazine
E. Don't answers are correct
193. Which of the following antipsychotic agents is drug of choise for patients with coronary and
cerebrovascular disease?
A. Chlorpromazine
B. Fluphenazine
C. * Haloperidol
D. Perphenazine
E. Don't answers are correct

194. Which of the following antipsychotic agents is used in combination with an opioid drug fentanyl in
neuroleptanalgesia?
A. Haloperidol
B. * Droperidol
C. Chlorpromazine
D. Clozapine
E. Don't answers are correct
195. Which of the following statements is correct for clozapine?
A. * Has potent anticholinergic activity
B. Has high affinity for D1 and D2 dopamine receptors
C. Produces significant extrapyramidal toxicity
D. Is related to typical antipsychotic agents
E. None answer is correct
196. Which of the following antipsychotic drugs has the high risk of potentially fatal agranulocytosis and
risk of seizures at high doses?
A. Haloperidol
B. Risperidone
C. * Clozapine
D. Chlorpromazine
E. Don't answers are correct
197. Which of the following statements is correct for lithium?
A. Stimulate dopamine and beta-adrenergic receptors
B. * Decrease catecholamine-related activity
C. Stimulate the development of dopamine receptor supersensitivity
D. Decrease cholinergic activity
E. Don't answers are correct
198. The principal mechanism of action of antidepressant agents is:
A. Stabilization of dopamine and beta-adrenergic receptors
B. Inhibition of the storage of serotonin and epinephrine in the vesicles of presynaptic nerve endings
C. * Blocking epinephrine or serotonin reuptake pumps
D. Stimulation of alfa2-norepinephrine receptors
E. Don't answers are correct
199. Which of the following agents is related to tricyclic antidepressants?
A. Nefazodon
B. * Amitriptyline
C. Fluoxetine
D. Isocarboxazid
E. Atropine
200. Indicate the second-generation heterocyclic antidepressant drug:
A. * Maprotiline
B. Imipramine
C. Aminazine (Chlorpromazine)
D. Fluoxetine
E. Atropine
201. Which of the following antidepressants is a selective serotonin reuptake inhibitor?
A. Amitriptiline
B. Desipramine

C. Maprotiline
D. * Fluoxetine
E. Atropine
202. Which of the following antidepressant agents is a selective inhibitor of norepinephrine reuptake?
A. Fluvoxamine
B. * Maprotiline
C. Amitriptyline
D. Tranylcypromine
E. Don't answers are correct
203. Indicate the antidepressant, which blocks the reuptake pumps for serotonin and norepinephrine:
A. * Amitriptyline
B. Fluoxetine
C. Maprotiline
D. Amphetamine
E. None answer is correct
204. Which of the following antidepressants is an unselective MAO blocker?
A. Amitriptyline
B. * Imirine
C. Selegiline
D. Fluoxetine
E. Don't answers are correct
205. Block of which type of Monoamine Oxydase might be more selective for depression?
A. * MAO-A
B. MAO-B
C. Both MAO-A and MAO-B
D. MAO-C
E. Don't answers are correct
206. The principal mechanism of MAO inhibitor action is:
A. Blocking the amine reuptake pumps, which permits to increase the concentration of the
neurotransmitter at the receptor site
B. * Blocking a major degradation pathway for the amine neurotransmitters, which permits more amines
to accumulate in presynaptic stores
C. Inhibition the storage of amine neurotransmitters in the vesicles of presynaptic nerve endings
D. Antagonism of alfa2-norepinephrine receptors
E. Don't answers are correct
207. The irreversible MAO inhibitors have a very high risk of developing:
A. Respiratory depression
B. Cardiovascular collapse and CNS depression
C. * Hypertensive reactions to tyramine ingested in food
D. Potentially fatal agranulocytosis
E. Don't answers are correct
208. The therapeutic response to antidepressant drugs is usually over a period of:
A. 2-3 days
B. * 2-3 weeks
C. 24 hours
D. 2-3 month
E. Don't answers are correct

209. Which of the following are common features to MAO inhibitors and tricyclic antidepressants?
A. Act postsynaptically to produce their effect
B. Can precipitate hypotensive crises if certain foods are ingested
C. * Increase levels of biogenic amines
D. Are useful for the manic phase of bipolar disorder
E. None answer is correct
210. Tricyclic antidepressants are:
A. Highly selective serotonin reuptake inhibitors
B. Monoamine oxidase inhibitors
C. Selective norepinephrine reuptake inhibitors
D. * Mixed norepinephrine and serotonin reuptake inhibitors
E. Don't answers are correct
211. Indicate an effective antidepressant with minimal autonomic nervous system toxicity:
A. Amitrityline
B. * Fluoxetine
C. Imipramine
D. Doxepin
E. Don't answers are correct
212. Fluoxetine has fewer adverse effects because of:
A. Mixed norepinephrine and serotonin reuptake inhibition
B. Depleted stores of amine neurotransmitters
C. * Minimal binding to cholinergic, histaminic, and alfa-adrenergic receptors
D. All of the above
E. Don't answers are correct
213. Which of the following tricyclic and heterocyclic antidepressants has the greatest sedation?
A. Fluoxetine
B. * Amitriptyline
C. Imipramine
D. All of the above
E. Don't answers are correct
214. Which of the following tricyclic and heterocyclic antidepressants agent has the least sedation?
A. * Protriptyline
B. Trazodone
C. Amitriptyline
D. Mitrazapine
E. Don't answers are correct
215. Indicate a tricyclic or a heterocyclic antidepressant with the most expressed antimuscarinic effects:
A. Fluoxetine
B. * Amitriptyline
C. Imipramine
D. Aminazine
E. None answer is correct
216. Sedation, peripheral atropine-like toxicity (e.g. Cycloplegia, tachycardia, urinary retention, and
constipation), orthostatic hypotension, arrhythmias, weight gain and sexual disturbances are possible
adverse effects of:
A. Sertaline
B. * Amitriptyline

C. Phenelsine
D. Bupropion
E. Don't answers are correct
217. Choose the antidepressant agent –a phenyltolylpropylamine derivate:
A. Paroxetine
B. Maprotiline
C. * Fluoxetine
D. Amitriptyline
E. Don't answers are correct
218. The mechanism of fluoxetine action includes:
A. * Selective inhibition of serotonine reuptake in the CNS
B. Selective inhibition of norepinephrine and serotonine reuptake in the CNS
C. Nonselective inhibition of monamines reuptake in the CNS
D. Nonselective inhibition of monamines and acetylcholine reuptake in the CNS
E. All of the above
219. Which of the following antidepressants is used for treatment of eating disorders, especially buliemia?
A. Amitriptyline
B. * Fluoxetine
C. Imipramine
D. Tranylcypromine
E. Don't answers are correct
220. A highly selective serotonine reuptake inhibitor is:
A. Fluoxetine
B. * Amitriptyline
C. Imipramine
D. Diazepam
E. Clorazepate
221. Anxiolytics are used to treat:
A. * Neurosis
B. Psychosis
C. Narcolepsy
D. Bipolar disorders
E. Don't answers are correct
222. Anxiolytic agents should:
A. Relieve pain
B. * Reduce anxiety and perform sedating effect
C. Improve mood and behavior in patient with psychotic symptoms
D. Produce drowsiness, encourage the onset and maintenance of a state of sleep
E. Don't answers are correct
223. Anxiolytics are also useful for:
A. * Treatment of epilepsy and seizures
B. Bulimia
C. Psychosis
D. Narcolepcy
E. Atrophic gastritis
224. Choose the group of drugs which are the best in the majority anxiety states treatment:

A. Barbiturates
B. * Benzodiazepines
C. Lithium salts
D. Phenothiazines
E. Adrenergic blockers
225. The choice of benzodiazepines for anxiety is based on:
A. A relatively low therapeutic index
B. * Availability of flumazenil for treatment of overdose
C. A extremely high risk of physiologic dependence
D. All of the above
E. Don't answers are correct
226. Indicate the benzodiazepine derivate with the shortest elimination half-life:
A. Phenobarbital
B. * Triazolam
C. Diazepam
D. Clorpromazine
E. Don't answers are correct
227. Which of the following benzodiazepines has the shortest duration of action?
A. * Triazolam
B. Clorazepate
C. Phenobarbita
D. Clorpromazine
E. Don't answers are correct
228. Which of the following benzodiazepines is less likely to cause cumulative and residual effects with
multiple doses?
A. Chlorpromazine
B. Diazepam
C. * Lorazepam
D. Amitriptilline
E. Don't answers are correct
229. Which of the following anxiolytics is preferred in patient with hepatic dysfunction?
A. * Buspirone
B. Quazepam
C. Diazepam
D. Chlordiazepoxide
E. Don't answers are correct
230. Indicate the mechanism of hypnotic benzodiazepine action:
A. Increasing the duration of the GABA-gated Cl- channel openings
B. Directly activating the chloride channels
C. * Increasing the frequency of Cl- channel opening events
D. All of the above
E. Don't answers are correct
231. Indicate the competitive antagonist of benzodiazepine receptors:
A. * Flumazenil
B. Buspirone
C. Picrotoxin

D. Diazepam
E. Don't answers are correct
232. Which of the following disadvantages does not limit using benzodiazepines as antianxiety agents?
A. Tendency to develop psychologic dependence
B. * A high risk of drug interactions based on liver enzyme induction
C. Synergic CNS depression with concomitant use of other drugs
D. The formation of active metabolites
E. Don't answers are correct
233. Choose the anxiolitic agent, which relieves anxiety and do not course significant sedative effect:
A. Diazepam
B. Chlordiazepoxid
C. * Buspirone
D. Clorazepate
E. Don't answers are correct
234. Which of the following anxiolytics has minimal ability to course abuse?
A. Oxazepam
B. * Buspirone
C. Flumazenil
D. Diazepam
E. Phenobarbital
235. In contrast to benzodiazepines, buspirone:
A. Interact directly with gabaergic system
B. Has more marked hypnotic, anticonvulsant, or muscle relaxant properties
C. * Causes less psychomotor impairment and does not affect driving skills
D. Has maximal abuse liability
E. Don't answers are correct
236. Which of the following sedative-hypnotic drugs does not potentiate the CNS depressant effects of
ethanol, phenothiazines, or tricyclic antidepressants?
A. * Buspirone
B. Phenobarbital
C. Diazepam
D. Chloralhydrate
E. Don't answers are correct
237. What limits usage of buspirone as anxiolytic agent?
A. A low therapeutic index
B. * An extremely slow onset of action
C. A high potential of development of physical dependence
D. Impairment of mentation or motor functions during working hours
E. Don't answers are correct
238. Restlessness, anxiety, orthostatic hypotension, generalized seizures, severe tremor, vivid
hallucination, and psychosis. What are possible symptoms of:
A. Tolerance
B. * Withdrawal
C. Drug interactions between barbiturate and diazepam
D. None of the above
E. Don't answers are correct
239. Flumazenil is mainly used to:
A. * Stop benzodiazepines action in case of their overdose
B. Decrease cardiac dysrhythmias
C. Increase gastric secretion
D. Decrease the breathing rate
E. None answer is correct
240. Flumazenil given intravenously
A. Has intermediate onset and duration of action about 2 hours
B. * Acts rapidly but has a short half-life
C. Has an effect lasting 3-5 hours
D. Has duration of action longer than 6 hours
E. One answer is correct
241. Agents, stimulating CNS are all of the following except:
A. Fluoxetine
B. * Clozapine
C. Nootropil
D. Sydnocarb
E. Don't answers are correct
242. Which of the following CNS stimulants are the agents of selective effect?
A. Analeptics
B. General tonics
C. * Psychostimulants
D. Actoprotectors
E. Don't answers are correct
243. Indicate CNC stimulating drugs, which are the agents of general action:
A. Nootropic agents
B. * Analeptics
C. Psychostimulants
D. Antidepressants
E. Don't answers are correct
244. Indicate the nootropic agent:
A. Sydnocarb
B. Eleuterococci extract
C. Fluoxetine
D. * Piracetam
E. Don't answers are correct
245. Which of the following agents is a respiratory analeptic?
A. Piracetam
B. Sydnocarb
C. * Bemegride
D. Pantocrin
E. Don't answers are correct
246. Indicate the CNC stimulating drug, which belongs to adaptogens:
A. Amphetamine
B. * Eleuterococci extract
C. Caffeine
D. Sydnocarb

E. Don't answers are correct


247. Indicate a general tone-increasing drug, which is an agent of animal origin?
A. * Pantocrin
B. Amphetamine
C. Sydnocarb
D. Camphor
E. Don't answers are correct
248. Choose the drug which increases cardiac output:
A. * Noradrenalin
B. Methyldopa
C. Phenylephrine
D. Angiotensinamide
E. Don't answers are correct
249. Which type of receptors can be activated by angiotensinamide:
A. Adrenergic receptors
B. Cholinergic receptors
C. Dopaminergic receptors
D. * Angiotensin’s receptors
E. All answers are correct
250. General unwanted effects of vasoconstrictors is:
A. Increase of arterial pressure
B. Increase of cardiac output
C. * Decrease of peripheral blood flow
D. Increase of blood volume
E. All answers are correct
251. Which agent is used to increase blood pressure in case of low cardiac output?
A. Ganglioblockers
B. Vasoconstrictors
C. * Positive inotropic drugs
D. Diuretics
E. All answers are correct
252. Choose the positive inotropic drug of glycoside structure:
A. Dopamine
B. * Digoxin
C. Dobutamine
D. Adrenalin
E. All answers are correct
253. Choose the positive inotropic drug of non-glycoside structure:
A. Digitoxin
B. Digoxin
C. * Dobutamine
D. Strophanthin
E. All answers are correct
254. Dopamine at low doses influences mainly:
A. Alfa-adrenoreceptors (leads to peripheral vasoconstriction)
B. * Dopamine receptors (leads to vasodilation of renal and mesenterial vessels)

C. Beta-1 adrenoreceptors (leads to enhanced cardiac output)


D. All of the above
E. Don't answers are correct
255. Dopamine at medium doses influences mainly:
A. Alfa-adrenoreceptors (leads to peripheral vasoconstriction)
B. Dopamine receptors (leads to vasodilation of renal and mesenterial vessels)
C. * Beta-1 adrenoreceptors (leads to enhanced cardiac output)
D. All of the above
E. Don't answers are correct
256. Dopamine in high doses influences mainly the:
A. * Alfa-adrenoreceptors (leads to peripheral vasoconstriction)
B. Dopamine’s receptors (leads to vasodilation of renal and mesenterial vessels)
C. Beta-1 adrenoreceptors (leads to enhancing of cardiac output)
D. All of the above
E. Don't answers are correct
257. Choose the group of drugs for chronic hypotension treatment:
A. Positive inotropic drugs
B. Vasoconstrictors
C. Plasmoexpanders
D. * Analeptics and tonics
E. Don't answers are correct
258. Choose the drug influencing the blood flow which belongs to antiplatelet agents:
A. Heparin
B. * Aspirin
C. Pyracetam
D. Atropine
E. Don't answers are correct
259. Which of the following drugs belongs to anticoagulants and may be used for disorders of cerebral
circulation treatment?
A. Aspirin
B. Cinnarizine
C. Nicergoline
D. * Heparin
E. Don't answers are correct
260. Indicate the drugs which are Ca-channel blockers influencing the brain blood flow:
A. Aminalon, Picamilon
B. * Nimodipine, Cinnarizine
C. Heparin, Warfarin
D. Vinpocetine, Nicergoline
E. All answers are correct
261. Indicate the drugs influencing the blood flow in the brain - GABA derivative:
A. * Aminalon, Picamilon
B. Nimodipine, Cinnarizine
C. Heparin, Warfarin
D. Vinpocetine, Nicergoline
E. All answers are correct

262. Indicate the nootropic agent which is used for brain circulation disorders treatment:
A. Aspirin
B. * Pyracetam
C. Warfarin
D. All the above
E. Don't answers are correct
263. What is the main action of GABA derivatives in disorders of brain circulation treatment?
A. Decrease of vessel permeability
B. * Stimulation of the metabolic processes in neurons
C. Brain vessel constriction
D. Intracranial pressure increase
E. All answers are correct
264. Choose the appropriate mechanism of vinpocetine action in brains:
A. * It dilates cerebral vessels and improves blood supply
B. It constricts cerebral vessels and decreases blood supply
C. It stimulates GABA-receptors and thus increases cerebral metabolic processes
D. It constricts peripheral vessels and increases blood pressure
E. Don't answers are correct
265. The following Indol derivative is used for treatment of acute migraine attack:
A. Paracetamol
B. * Sumatriptan
C. Ergotamine
D. Metoclopramide
E. Don't answers are correct
266. The following Ergot derivative is used for treatment of acute migraine attack:
A. Paracetamol
B. Sumatriptan
C. * Ergotamine
D. Metoclopramide
E. Don't answers are correct
267. In seriously ill patients the medical use of morphine as an analgesic is most limited by morphine's:
A. * Respiratory effects
B. Endocrine effects
C. Gastrointestinal effects
D. Dependence potential
E. Hyperthermia
268. Withdrawal from morphine can be precipitated by administering:
A. Valeriana
B. * Naloxone
C. Flumazenil
D. Nicotine
E. Muscarine
269. Tamoxifen is most active in postmenopausal women with breast cancer because:
A. Endogenous estradiol levels are lower
B. Greater likelihood of estrogen receptors in tumor tissue
C. Not true; tamoxifen works best in premenopausal patients

D. * Endogenous estradiol levels are lower; greater likelihood of estrogen receptors in tumor tissue
E. Don’t answer is correct
270. The benzodiazepine, diazepam, produces its anticonvulsant effects by:
A. Stimulating adenosine receptors
B. * Augmenting GABA activity in the brain
C. Blocking voltage-gated sodium channels
D. Augmenting serotonin activity in the brain
E. All of the above
271. The process by which drugs influence cell physiology is called:
A. Pharmacokinetics
B. Pharmacotherapeutics
C. * Pharmacodynamics
D. Pharmacology
E. All answers are correct
272. Pharmacokinetics involves the study of:
A. Pharmacological effects
B. * Distribution rates between various body compartments
C. Interactions between various drugs
D. Adverse reactions to medications
E. All answers are correct
273. Drugs can exert their action on the body by:
A. Interaction with receptors
B. None answer is correct
C. Inhibiting the action of a specific enzyme
D. Altering metabolic chemical processes
E. * All answers are correct
274. Another name for biotransformation of a drug is:
A. Absorption
B. Dilution
C. Excretion
D. * Metabolism
E. All answers are correct
275. Drugs given by which route are altered by the first-pass effect?
A. Sublingual
B. Subcutaneous
C. * Oral
D. Intravenous
E. All answers are correct
276. Which of the following medications is used to treat a patient suffering from severe adverse effects of
a narcotic analgesic?
A. * naloxone (Narcan)
B. acetylcysteine (Mucomyst)
C. methylprednisolone (Solu-Medrol)
D. protamine sulfate
E. don’t answer is correct
277. A 23-year-old has been taken to the emergency room for a suspected overdose of morphine tablets.
Which agent may be used to treat this overdose?

A. Demerol
B. Naprosyn
C. Aspirin
D. * Naloxone (Narcan)
E. Don’t answer is correct
278. During the 3 years old boy examination dentist have found the incisor destruction, yellow enamel,
and brown limbs of dental cervix. Doctor found that his mother had been taken some antibiotics
during pregnancy. What drug did she take?
A. * Doxycycline
B. Ampicilline
C. Furosemide
D. Octadine
E. Nitroglycerine
279. The patient’s blood examination shows increased level of aldosterone. Which of the following
diuretic drugs is the drug of choice for this patient?
A. * Spironolactone
B. Furosemide
C. Mannit
D. Hydrochlortiazide
E. Torasemide
280. The 32 years old patient admitted to the hospital. The pneumonia caused with Streptococcus
pneumonia was diagnosed. Which of the following is the drug of choice for the pneumococcal
pneumonia treatment?
A. * Benzylpenicillinum natrium
B. Isoniazidum
C. Dimedrolum
D. Celecoxib
E. Acidum acetylsalicylicum
281. The patient took hydrochlorothiazide for the treatment of hypertension. What is the mechanism of its
hypotensive action?
A. * Increased excretion of sodium and water
B. Increased angiotensin II production
C. Calcium channels block
D. Potassium channels block
E. Block of cyclooxygenase
282. Patient suffers from syndrome of water-ion disbalance in blood after prolonged treatment of his
chronic edema with furosemide. Which of the following suplements should be used to prevent this
side effect of furosemide?
A. * Potassium chloride
B. Calcium chloride
C. Sodium hydrocarbonate
D. Magnesium sulfate
E. Vikasol
283. Patient with cerebral trauma was prescribed diuretic drug for the brain edema prevention. Which of
the following is the drug of choice for this patient?
A. * Furosemide
B. Magnesium sulfate
C. Drotaverine

D. Enalapril
E. Vikasol
284. The patient complains on severe headache, light headedness, which developed after tooth extraction.
He is found to have a blood pressure of 200/110 mm Hg. Hypertensive crisis is diagnosed. Which of
the following drugs does belong to the diuretic group and is the drug of choice for this patient?
A. * Furosemide
B. Sodium hydrocarbonate
C. Enalapril
D. Magnesium sulfate
E. Calcium chloride
285. A patient was admitted to intensive care unit with signs of brain edema. Physician to prevent adverse
reaction of furosemide uses another diuretic triamterene. Specify which effect of such combination
was a main target?
A. * Potassium-sparing
B. Spasmolytic
C. Analgesic
D. Sedative
E. Irritatative
286. A patient with arterial hypertension was prescribed furosemide. Which supplement should be
prescribed for hypokalemia prevention?
A. * Asparcam
B. Atropine
C. Almagel
D. Magnesium sulfate
E. Adrenalin
287. A 49-years-old patient with toxic pulmonary edema was injected loop diuretic drug. Which of the
following drugs was used?
A. * Furosemide
B. Sodium hydrocarbonate
C. Enalapril
D. Magnesium sulfate
E. Calcium chloride
288. The patient was admitted to intensive care unit. He complains on fatigue, feeling of weakness in his
legs, palpitation. ECG showed tachyarrhythmia. Doctor discovered that loop diuretic drug had been
taken during last 3 month. Which of the following was the most likely to course this patients health
disorders?
A. * Hypokaliemia
B. Hypercalciemia
C. Hyponatriemia
D. Hyperkaliemia
E. Hyperuricemia
289. The 42 years old patient was diagnosed hypertension. He was prescribed thiazide diuretic. Which of
the following was the most likely prescribed to the patient?
A. * Hydrochlorothiazide
B. Furosemide
C. Adrenaline hydrochloride
D. Spironolactone
E. Anaprilin

290. The 22 years old male patient was admitted to intensive care unit with signs of acute intoxication
with morphine. After naloxone injection he undergoes forced diuresis. Which of the following drugs
is the most likely to be used for this aim?
A. * Furosemide
B. Spironolactone
C. Galantamine hydrobromide
D. Analginum, diclofenac sodium
E. Magnesium sulfate, atropine sulfate
291. A 45 years old male patient suffers from arterial hypertension. Usually his blood pressure is well
controlled with the drug combination prescribed by his doctor, but now it is elevated up to 190 and
100 mm Hg. His wife usually takes diuretic drug in case of hypertensive crisis. He knows that
spironolactone he takes is diuretic drug. So he desided to take spironolactone to decrease his blood
pressure. Why spironolactone intake had no significant effect on his blood pressure?
A. * It has long period before the onset of action and weak diuretic effect
B. It increases blood pressure
C. It causes vessel constriction
D. It should be used in higher dose
E. It does not perform any diuretic action
292. A 62 years old male patient suffers from kidney insufficiency. He complaines on hearing loss,
tinnitus and vertigo. He have been taken a diuretic drug during last 6 month. Which of the following
drugs is the most likely to course such effects?
A. * Furosemide
B. Magnesium sulfate
C. Spironolactone
D. Prednisolone
E. Triamterene
293. A 62 years old male patient suffers from heart insufficiency and from gout experiences severe gout
attack. He receives allopurinol for his gout. Yesterday he started diuretic drug intake due to his ankle
edema. It was found a significant increase of serum urate in his blood. What diuretic drug could
cause increased level of serum urates and provoke gout attack?
A. * Furosemide
B. Vikasol
C. Mannit
D. Magnesium sulfate
E. Sodium chloride
294. A 35-years-old patient was admitted to the intensive care unit. Furosemide is a part of combined
therapy which he receives. Which of the following pathological processes could be a reason of his
hospitalization?
A. * All answers are correct
B. Acute heart failure
C. Pulmonary edema
D. Edema of brain
E. Renal insufficiency
295. A 52-years-old male patient with acute myocardial infarction, acute left ventricular insufficiency and
pulmonary edema was admitted to the intensive care unit. Which of the following drugs is the most
likely to be included in his complex therapy?
A. * Furosemide
B. Celecoxib
C. Dithyllin

D. Piracetam
E. Azithromycin
296. A 27-years-old male patient admitted to intensive care unit with symptoms of acute intoxication.
Which of the following drugs should be used to produce forced diuresis?
A. * Furosemide
B. Theophylline
C. Unithiol
D. Furacilinum
E. Hydrochlorothiazide
297. A 65-years-old male patient with chronic heart failure has been taken furosemide during last three
month. What irreversible side effect can occur in this case?
A. * Deafness
B. Diarrhea
C. Headache
D. Vomiting
E. Constipation
298. A 61-years-old man with congestive heart failure has taken loop diuretic furosemide during last two
days. Because of side effects development furosemide was discontinued. Next day "the ricochet
phenomenon" was diagnosed. What does the “ricochet phenomenon" mean?
A. * Increased reabsorption of sodium and water
B. Increased excretion of calcium to the urine
C. Inhibition of reabsorption of magnesium ions
D. The appearance of metabolic acidosis
E. The appearance of respiratory acidosis
299. The 29 years old male patient admited to the hospital with acute kidney injury. His glomerular
filtration rate decreased from 125 ml / min. to 30 ml / min. What diuretic drug should be used in this
case?
A. * Furosemide
B. Atropine sulfate
C. Magnesium sulfate
D. Asparcam
E. Furaciline
300. The 29 years old male patient suffers from arterial hypertension. It was found increased renin level in
his blood. To which of the following groups does drug of choice for this patient belong?
A. * ACE inhibitors
B. Myotropic vasodilators
C. Calcium channel blockers
D. Potassium channels activators
E. Sympatholytics
301. The 48 years old male patient suffers from chronic heart failure. Cardiac glycosides were prescribed
in complex treatment. What pharmacokinetic parameter is used for calculating the maintenance dose
of digoxin?
A. * Clearence of drug in the body
B. Way of drug excretion
C. Route of administration
D. Rate of dissolution
E. Diffusion coeficient

302. The patient, who used to take diazepam for treatment of neurosis, diclofenac sodium was prescribed
for removing symptoms of arthritis. What pharmacological phenomenon of drug-drug interaction
should be mentioned by physician in this case?
A. * Potentiation
B. Antagonism
C. Accumulation
D. Dependence
E. Tolerance
303. The 29 years old male patient suffers from rheumatoid arthritis. Methotrexate was included in his
complex therapy. Which of the following side effects can you expect?
A. * All answers are correct
B. Nausea
C. Thrombocytopenia
D. Severe skin rahes
E. Leukopenia
304. Atropine is used to treat the organophosphate compounds poisoning in 22 years old male patient. The
mechanism of antidote action of this drug is based on:
A. * Physiological antagonism
B. Synergism
C. Addiction
D. Chemical antagonism
E. Accumulation
305. The 24years old woman is diagnosed with neurosis. She was prescribed diazepam to eliminate
feelings of fear and anxiety. What pharmacological effect of this drug is used in this case?
A. * Anxiolytic
B. Antiarrhythmic
C. Antiphlogistic
D. Antianginal
E. Hypotensive
306. A patient was admitted to intensive care unit with signs of acute poisoning with mercury dichloride.
It was used specific antidote, containing sulfhydryl groups for the treatment of this patient. What
drug was used?
A. * Unithiol
B. Protamine sulfate
C. Atropine
D. Lidocaine
E. Naloxone
307. Patient presents to the emergency department with pulmonary edema. Which of the following drugs
should be given to the patient to reduce blood volume and for lung dehydration?
A. * Furosemide
B. Magnesium sulfate
C. Spironolactone
D. Oxytocin
E. Propranolol
308. A patient was admitted to intensive care unit with signs of pulmonary edema. Which drug from the
diuretic group should be injected to the patient?
A. * Furosemide
B. Lidocaine

C. Isoniazide
D. Oxytocin
E. Acetylsalicylic acide
309. Woman experienced intensive psychological stress. She complains on insomnia during last several
nights. Which of the following drugs should be given to treat this type of insomnia?
A. * Nitrazepam
B. Haloperidol
C. Atropini sulfate
D. Cofeinum-natrii benzoas
E. Diphenine
310. A 45-year-old man with acute myocardial infarction is injected intravenously very slowly 0.2 %
solution of local anesthetic – lidocaine hydrochloride. The purpose of this procedure is:
A. * Ventricular tachyarrhythmia treatment
B. Maintaining of infiltration anesthesia
C. Maintaining of conductive anesthesia
D. Reduction of pain
E. Maintaining of thrombolysis
311. The patient with HIV infection and pneumocystis pneumonia is treated with sulphonamide
Co-trimoxazole. The drug caused hemolysis of erythrocytes. The reason to this complication is
genetic deficiency of glucose-6-phosphate dehydrogenase of red blood cells. To what type of
reactions does this complication belong?
A. * Idiosyncrasy
B. Teratogenic action
C. Drug dependence
D. Tachyphylaxis
E. Tolerance
312. For prevention of rheumatism relapses patient had been taken prolonged form of penicillin. The
intradermal test for hypersensitivity to this drug was made (amount of the drug was 100 times lower
than therapeutic) before the next injection. The skin rash with red, raised, itchy bumps occurs in the
site of injection. How is this reaction to repeated drug administration called?
A. * Allergic reaction
B. The relative overdose
C. Acute intoxication
D. The therapeutic effect
E. Accumulation
313. The patient with bronchial asthma used remedy from beta-adrenergic agonists group. Which of the
following drugs is used in the therapy of bronchial asthma?
A. * Salbutamol
B. Nitrazepam
C. Oxytocinum
D. Calcii chloridum
E. Paracetamol
314. For the unconscious patient with symptoms of uncompensated metabolic acidosis, for correction pH
of tissues 100 ml 4 % solution of sodium hydrocarbonate in a form of enema was used. To what type
of drug administration does the rectal rout belong?
A. * Enteral
B. Intravenous
C. Inhalantion

D. Parenteral
E. Subcutaneous
315. The physician used lidocaine with adrenaline (epinephrine) to maintain local anesthesia. Which of
the following is a purpose such drug combination usage?
A. * It causes vasoconstriction and keeps the anesthetic local
B. This combination is harmfull
C. It enhances the pharmacological effects of the epinephrine
D. It helps calm the patient before the procedure
E. It minimizes the risk of allergic reaction
316. Patient with arterial hypertension has elevated level of renin in serum. To which of the following
groups does the drug of choice belong?
A. * ACE inhibitors
B. Ganglionblockers
C. Calcium channel blockers
D. Potassium channels activators
E. Myotropic vasodilators
317. A patient is admited to intensive care unit with symptoms of acute poisoning with heavy metal salts.
For the treatment of this state specific antidote, which contains sulfhydryl groups, was used. Which
of the following drugs was used?
A. * Unithiol
B. Scopalamine
C. Kalii permanganas
D. Heparine
E. Sodium bicarbonate
318. Patient admited to intensive care unit with infection caused by strains of pneumococcus (S.
pneumonia). Which of the following drugs should be used in this case?
A. * Benzylpenicillin
B. Natrii hydrocarbonas
C. Natrii oxybutyrate
D. Sulfocamphocaine
E. Pyrazinamide
319. Atropine is used as specific antagonist for acute poisoning with organophosphate compounds. What
is the pharmacological basis of this antidote usage?
A. * Physiological antagonism
B. Synergism
C. Addiction
D. Potentiation
E. Material cumulation
320. A 53-years-old woman arrives at her physicians office complaining of severe anxiety. She was
prescribed diazepam to eliminate feelings of fear and anxiety. Which of the following
pharmacological effects is used in this case?
A. * Anxiolytic
B. Antiarrhythmic
C. Anticonvulsive
D. Antipsychotic
E. Antiallergic
321. The patient is admitted to the intensive care unit with acute lung edema. Which of the following is a
drug of choice to maintain the blood volume reduction and pulmonary dehydration?

A. * Furosemide
B. Amiloride
C. Nitroglycerin
D. Triamteren
E. Indapamide
322. A woman with an endodontic abscesse is precribed an antibiotic drug. Which of the following is the
drug of choice in this case?
A. * Lincomycin
B. Nitroxoline
C. Penicillin
D. Amoxacillin
E. Clavulonic acid
323. A patient used acetaminophen with codeine for pain relief after tooth extraction during last 2 days.
Constipation developed. Which of the following terms describes this reaction?
A. * Adverse effect
B. Therapeutic effect
C. Mutagenic effect
D. Teratogenic effect
E. Embriotoxic effect
324. A patient takes hydrochlorothiazide for his arterial hypertension. Which of the following should be
monitored regularly?
A. * Potassium ions level
B. Liver transaminases
C. Creatinine level
D. Thrombocytes level
E. Leukocytes level
325. A patient with hypertensive crisis and the history of arterial hypertension and chronic heart failure
admitted to the hospital. Lung edema has developed. Which diuretic drug is drug of choice in this
case?
A. * Furosemide
B. Mannit
C. Spironolactone
D. Isadrinum
E. Amiloride
326. A patient with acute poisoning with aspirin admitted to the hospital. Choose the diuretic drug peform
the forced diuresis.
A. * Furosemide
B. Amiloride
C. Asparkam
D. Triamteren
E. Acetylcysteine
327. The patient with acute heart failure was injected dobutamine. What is the mechanism of its
cardiotonic action?
A. * Stimulates beta-1-adrenergic receptors
B. Blocks proton pump
C. Stimulates M-cholinergic receptors
D. Blocks M-cholinergic receptors

E. Inhibits the activity of cyclooxygenase 1


328. The patient with hypertension and tachyarrhythmia presents to the cardiologist. Which of the
following drugs is the drug of choice in this case?
A. * Metoprolol
B. Enalapril
C. Salbutamol
D. Minoxidil
E. Midazolam
329. Patient with pneumococcal pneumonia admitted to the hospital. Drugs from which pharmacological
group should be used in this case?
A. * Antibiotics
B. Hypotensive agents
C. Opioid analgesics
D. Hypnotic agents
E. Beta-adrenergic blockers
330. A 48-year-old homeless man with a history of alcoholism admitted to the emergency department
with acute pancreatitis. Which antienzyme preparation is drug of choice in this case?
A. * Contrycal
B. Festal
C. Mannitol
D. Mebendasole
E. Almagel
331. A patient takes famotidine for peptic ulcer. What is the main mechanism of its action?
A. * H2-histamine receptors block
B. H1-histamine receptors block
C. M1-cholinergic receptors block
D. N-cholinergic receptors block
E. Beta-adrenergic receptors stimulation
332. A farm worker whith the history of contact with phosphororganic compounds has been brought to
emergency room. He is unconscious, with profuse sweating and salivating. Severe bradycardia and
decreased arterial pressure were found. Which of the following drugs should be prescribed as an
antidote in this case?
A. * Atropine sulfas
B. Dipiroxine
C. Physostigmine
D. Norepinephrine
E. Pentamine
333. A woman with odontogenic osteomyelitis is prescribed an antibiotic. Whichof th following
antibiotics is a drug of choice in this case?
A. * Lincomycin
B. Amoxiclav
C. Retrovir
D. Mannitol
E. Penicillin
334. Forced diuresis is used for emergency care in case of acute intoxication. Indicate the diuretic drug
which is the most suitable for this procedure.
A. * Furosemide
B. Veroshpirone

C. Magnesii sulfas
D. Indapamide
E. Potassium chloride
335. Thiazide and related diuretics are inhibitors of Na+Cl-symport. Which of the following is indication
for Hydrochlorothiazide?
A. * Congestive heart failure
B. Diarrhea
C. Constipation
D. Dementia
E. Peptic ulcer
336. After surgical tooth extraction the bleeding occurred in patient. Which of the following drugs is
contraindicated for patients before such surgical procedures?
A. * Heparin
B. Fibrinogen
C. Vitmin K
D. Pyrazinamide
E. Amoxiclav
337. Process of breathing is controlled by the respiratory CNS center. Poisoning with which of the
following drugs performs the highest risk of respiratory arrest?
A. * Morphine
B. Acetominophen
C. Digitoxin
D. Norepinephrine
E. Metformine
338. Drug addiction is one of the adverse effects. Which of the following are the parts of physical
dependence?
A. * tolerance and withdrawal symptoms
B. idiosyncrasy and allergy
C. carcinogenesis and toxicity
D. favism and formation of methemoglobin
E. embriotoxicity and congenital defects
339. A 25-year-old woman with red and itchy eczematous dermatitis visits your office. She had a dental
procedure one day earlier with administration of a local anesthetic. There were no other findings,
although she indicated that she had a history of allergic reactions. Which of the following drugs was
most likely involved?
A. * Lidocaine
B. Buprenorphine
C. Acebutolol
D. Aethimizolum
E. Captopril
340. On the 2-3rd day after stomach resection intestinal peristalsis wasn't restored. What is to be
administered for stimulation of gastrointestinal tract motility?
A. * Proserine
B. Clozapine
C. Prazosin
D. Cyclodole
E. Acetylcholine

341. A dentist injected a patient with a certain drug in order to reduce salivation during tooth filling.
Which of the following drugs was injected to the patient?
A. * Atropine sulfate
B. Ramipril
C. Prazosine
D. Clozapine
E. Losartan
342. A test for novocaine sensitivity was made to the patient before the infiltrative anesthesia was carried
out. It was positive. Which of the following drugs could be used for anesthesia in this case?
A. * Lydocaine
B. Streptomycin
C. Cocaine
D. Benzocaine
E. Dicaine
343. The patient with hypertensive crisis was injected the drug from the ganglion blockers group. Which
side effect might be observed in this case?
A. * Orthostatic collapse
B. Hypertension
C. Cumulation
D. Hyperglycemia
E. Hypokalemia
344. The patient with angina pectoris receives antianginal drug. Bronchial asthma attack occured. Which
of the following drugs could cause such side effect?
A. * Propranolol
B. Nitroglycerin
C. Validol
D. Ramipril
E. Nifedipine
345. The patient with peptic ulcer was prescribed omeprazole. Which of the following is the mechanism
of omeprazole antisecretory action?
A. * Block of Na+ / K+ ATPase
B. Activation of Na+ / K+ ATPase
C. Stimulation of adrenergic receptors
D. Block of adrenergic receptors
E. Block of histamine receptors
346. A patient with the symptoms of acute arsenic poisoning was admitted to the intensive care unit.
Which of the following agents is antidote for arsenic poisoning?
A. * Unithiolum
B. Interferonum
C. Adrenalin hydrochloride
D. Sustac
E. Validol
347. At the dentist's office patient began to complain of intense retrosternal pain that spread in the left half
of the face, jaw and teeth. Which of the following drugs is the most effective in this case?
A. * Nitroglycerin
B. Diclofenac sodium
C. Aluminium hydroxide
D. Festal

E. Omeprazole
348. A patient is prescribed bile-containing preparation for improvement of food digestion. What
components of this drug are fat emulsifiers?
A. * Bile acids
B. Cholesterol
C. Diglyceride
D. Bilirubin-glucuronids
E. Omega 3 fatty acids
349. Antivitamins are substances of various structure that limit utilization of vitamins in an organism and
have an opposite to them action. Name antivitamin of vitamin K:
A. * Dicumarol
B. Sulfapyridasine
C. Deoxypyridoxine
D. Aminopterin
E. Isoniazid
350. A patient with low immunity and frequent colds is recommended to take ascorutine as a more
effective drug than ascorbic acid. What constituent substance of this preparation intensifies action of
vitamin C?
A. * Vitamin P
B. Vitamin A
C. Glucose
D. Lactose
E. Vitamin D
351. A patient with a history of hepatitis B is prescribed lipotropic factors in order to prevent fatty liver
degeneration. Which of the following might be prescribed to the patient?
A. * Tryptophane
B. Choline
C. Allopurinol
D. Contrical
E. Vicasol
352. Antibiotics are classified according to the sources of their origin. Which of the following is the one of
bacterial origin?
A. * Gramicidin
B. Penicillin
C. Tetracycline
D. Lysozyme
E. Gentamycin
353. A woman in labour is diagnosed with the weakness of labour activity. A preparation that activates
contractions of smooth uterine muscles is prescribed. What hormone does this drug contain?
A. * Oxytocin
B. Gastrin
C. Secretin
D. Angiotensin
E. Progesterone
354. Patient with tuberculosis takes a drug that is an antivitamin of nicotinic acid. Name this substance:
A. * Isoniazid
B. Sulfanilamide

C. Acrichine
D. Isoriboflavin
E. Oxythiamine
355. A patient was prescribed with an antitumoral antibiotic that inhibits synthesis of nucleic acids in the
cells. What of the following antibiotics has such mechanism of action?
A. * Actinomycin
B. Tetracycline
C. Nystatin
D. Lincomycin
E. Erythromycin
356. A patient suffers from the neurasthenic syndrome, diarrhea and dermatitis. The deficiency of which
vitamin is associated with the following symptoms?
A. * Nicotinic acid
B. Vitamin K
C. Vitamin D
D. Folic acid
E. Vitamin B12
357. A 32 -year-old woman presents to her physician complaining of the impaired mesopic vision. Her
photopic vision is normal. Which of the following is the most possible reason of such vision
anomaly?
A. * Vitamin A deficiency
B. Hyperopia
C. Cones disfunction
D. Myopia
E. Vitamin D deficiency
358. Enzyme hyaluronidase breaks down hyaluronic acid thus increasing intercellular permeability. Which
vitamin strengthens vascular walls and inhibits activity of hyaluronidase?
A. * P
B. A
C. B1
D. B2
E. D
359. A 58-year-old woman with chronic pancreatitis was prescribed a bile containing preparation for the
better digestion of fatty food. What components of this preparation cause fat emulsification?
A. * Bile acids
B. Cholesterol and its ethers
C. Diglycerides
D. Bilirubinglucuronids
E. Bile pigments
360. Antibiotics can be classified according to various principles. If they are classified according to the
mechanism of action, to which of the following groups do cephalosporins belong?
A. * Inhibitors of cell wall synthesis
B. Inhibitors of protein synthesis
C. Inhibitors of respiratory processes
D. Inhibitors of oxidative phosphorilation
E. Inhibitors of cytoplasmic membrane synthesis
361. Blockers of which membrane receptors should be used to lower the pumping function of the heart?

A. * Beta-adrenoreceptors
B. Nicotinic cholinoreceptors
C. Muscarinic cholinoreceptors
D. Alpha-adrenoreceptors
E. Dopamine receptors
362. During a few minutes after second injection of penicillin to a patient dyspnea, tongue numbness,
hyperemia occurred. Then patient become pale and lost consciousness. What is the most likely cause
of such complication?
A. * Anaphylactic shock
B. Serum sickness
C. Hemolytic anemia
D. Acute glomerulonephritis
E. Bronchial asthma
363. Glucocorticoids induce elevation of glucose level in blood. Which of the following processes is
activated in liver?
A. * Gluconeogenesis
B. Bile synthesis
C. Oxidation of fatty acids
D. Ketogenesis
E. Glycolysis
364. A patientcompains on gingival haemorrhage and petechial haemorrhage. Which of the following
vitamin preparation should be recommended to the patients?
A. * Ascorutinum
B. Thiamine hydrochloride
C. Cyanocobalamin
D. Nicotinic acid
E. Pyridoxine hydrochloride
365. A 50-year-old woman has been treated with antibiotics for a long time. During the microscopic
examination of the vaginal discharge the oval cells with well-defined nucleus and gemmae on the
surface were found. The vaginal candidosis was diagnosed. Which of the following groups of drugs
should be used in this case?
A. * Antifungal
B. Antibacterial
C. Antichlamydial
D. Antiviral
E. Antiprotozoal
366. A patient takes muscarinic cholinoreceptor blocker. What changes in the heart activity will be
observed?
A. * Increasing of the heart rate
B. Decreasing of the heart rate and force of the heart contractions
C. Decreasing of the heart rate l
D. Decreasing of the force of the heart contractions
E. Increasing of the atrioventricular delay
367. What mediator maintains nerve impulse transmission to the salivary glands?
A. * Acetylcholine
B. Serotonin
C. Histamine
D. GABA

E. Dopamine
368. What neurotransmitter provides impulse transmission from the motoneurons nerve terminations to
the fibers of skeletal muscles?
A. * Acetylcholine
B. Adrenaline
C. Noradrenaline
D. Serotonin
E. GABA
369. Which of the following drugs is a competitive inhibitor of cholinesterase?
A. * Proserin
B. Aspirin
C. Sodium diclophenac
D. Indometacin
E. Allopurinol
370. A 30 year-old patient with pulmonary tuberculosis was prescribed isoniazid. The long-term
administration of this drug may lead to the deficiency of which of the following vitamin?
A. * Tocopherol
B. Pyridoxine
C. Cobalamin
D. Ergocalciferol
E. Retinol
371. A patient with peptic ulcer has been administered omeprazole during the last two weeks. What is the
mechanism of its action?
A. * Inhibition of H+K+-ATPase
B. Neutralization of HCl
C. Blockade of histamine H2-receptors
D. M-cholinergic receptor blockade
E. Stimulation of mucus production
372. A patient with a heart rhythm disorder has been given lidocaine. Which of the following
pharmacological effects is a reason of lidocain usage in this case?
A. * Antiarrhythmic
B. Hypnotic
C. Antipyretic
D. Antidepressant
E. Nootropic
373. A patient with chronic constipation has been prescribed bisacodyl. After 3 weeks of treatment he
noticed a reduction of laxative effect. Which of the following processes is a reason of laxative effect
reduction?
A. * Dependence
B. Tolerance
C. Sensibilization
D. Cumulation
E. Dysbacteriosis
374. A patient with acute heart failure and cardiac glycosides intolerance was injected dobutamine. What
is the mechanism of its action?
A. * Stimulation of alpha1-adrenoceptors
B. Blockade of K+-, Na+-ATPase

C. Inhibition of phosphodiesterase activity


D. Stimulation of beta1-adrenoceptors
E. Stimulation of M-cholinergic receptors
375. Doctor prescribed a drug for headache with antiplatelet effect. Which of the following analgetics was
used?
A. * Acetylsalicylic acid
B. Promedol
C. Codeine phosphate
D. Tramadol
E. Fentanyl
376. A patient with kidney colic administered a spasmolytic drug from the M-cholinergic antagonists.
Which of the following drugs was used?
A. * Atropine
B. Proserin
C. Galantamine
D. Dithylinum
E. Benzohexonium
377. A patient with chronic bronchitis is prescribed glaucine hydrochloride. About which of the following
is typical side effect of this drug?
A. * Decrease in blood pressure
B. Arrhythmia
C. Excitement of the central nervous system
D. Increase in intraocular pressure
E. Allergic skin rash
378. A 19-year-old woman with chills has been prescribed an antipyretic medication. Which of the
following drugs does perform antipretic properties?
A. * Paracetamol
B. Ascorbic acid
C. Oxytocin
D. Famotidine
E. Cyanocobalamin
379. A patient has been taking diclofenac sodium for moderate pain. Doctor recommended to change the
drug and prescribed celecoxib. What disease might be the reason for the drug substitution?
A. * Peptic ulcer
B. Bronchial asthma
C. Urolithiasis
D. Arterial hypertension
E. Chronic hepatitis
380. A patient with bronchial asthma presents to his physician. Which of the following is the drug of
choice for the 1 stage of asthma treatment?
A. * Salbutamol
B. Paracetamol
C. Diclofenac sodium
D. Anapriline
E. Acetylcysteine
381. Parkinsonian symptoms and tarditive dyskinesia are caused by blockade dopamine in:
A. * The nigrostriatal system

B. The mesolimbic and mesofrontal systems


C. The chemoreceptor trigger zone of the medulla
D. The tuberoinfundibular system
E. Don't answers are correct
382. Which of the following antipsychotic drugs is typical?
A. Clozapine
B. Quetiapine
C. * Haloperidol
D. Olanzapine
E. Don't answers are correct
383. Indicate the atypical antipsychotic drug:
A. Haloperidol
B. * Clozapine
C. Thioridazine
D. Thiothixene
E. Don't answers are correct
384. A 69-year-old male with angina develops severe constipation following treatment with
A. Propranolol
B. Captopril
C. * Verapamil
D. Dobutamine
E. Nitroglycerin
385. Angiotensin converting enzyme (ACE) inhibitors are associated with a high incidence of which of
the following adverse reactions?
A. Hepatitis
B. Hypokalemia
C. Agranulocytosis
D. * Proteinuria
E. Hirsutism
386. Administration of which of the following antianginal agents results in antianginal effects for only 10
hours, despite detectable therapeutic plasma levels for 24 hours?
A. Atenolol
B. * Transdermal nitroglycerin
C. Amlodipine
D. Ampicillin
E. Acyclovir
387. Which of the following is unlikely to occur with low concentrations of nitroglycerin?
A. * Decreased heart rate
B. Decreased end-diastolic blood pressure
C. Decreased myocardial oxygen demand
D. Decreased preload and afterload
E. Increased coronary blood flow
388. Which of the following local anesthetics is useful for topical (surface) administration only?
A. Procaine
B. Bupivacaine
C. Etidocaine
D. * Benzocaine

E. Tannin
389. Which of the following routes of drug administration bypasses the GI tract?
A. Intravenous
B. Inhalation
C. * All of indicated
D. Intramuscular
E. Subcutaneously
390. Which of the following groups of drugs is used as nasal decongestants in rhinitis?
A. * Alpha1-agonists
B. Beta2-agonists
C. M-cholinomimetics
D. Ganglion blockers
E. Beta-adrenoblockers
391. Hyperactivity of cyclooxygenase 2 is only associates with
A. * inflammation
B. depression
C. constipation
D. mucosa protection
E. low blood pressure
392. Which of the following drugs may cause xerostomia?
A. * Atropine
B. Norepinephrine
C. Dopamine
D. Galantamine
E. Pilocarpine
393. Which of the following drugs is used for relaxing bronchial muscles in asthma?
A. * Salbutamol
B. Timolol
C. Reserpine
D. Dobutamine
E. Norepinephrine
394. Diuretic which may result hypokalemia when used in a patient not receiving potassium
supplementation include:
A. Spironolactone
B. * Furosemide
C. Triamterene
D. Potassium chloride
E. None of the above
395. Name the drug group that can reduce need of myocardium for oxygen, decrease force of heartbeat
and inhibit lipolysis:
A. * Beta-adrenoceptor blockers
B. Aalpha-adrenoceptor blockers
C. Sympatholytics
D. Selective beta-adrenoceptor agonists
E. Alpha-adrenoceptor agonists
396. Gynecomastia, hyperkalemia, and menstrual abnormalities are potential adverse effects associated
with such diuretic drug:

A. furosemide
B. captopril
C. acebutolol
D. * spironolactone
E. digitoxine
397. Recommend a patient with acute bronchitis a mucolytic drug that ease expectoration:
A. * Acetylcysteine
B. Glaucine
C. Loratadine
D. Diclofenac sodium
E. Drotaverine hydrochloride
398. What drug is used for the removal of hypertensive crisis?
A. * Clofelinum
B. Dihlotyazid
C. Spironolactonum
D. Reserpinum
E. Octadin
399. To indicate an antihypertensive drug of myotropic action
A. * Dibazolum
B. Anaprilinum
C. Reserpinum
D. Captopril
E. Verapamilum
400. To indicate an antihypertensive drug from the calcium channels blockers.
A. Anaprilinum
B. Captopril
C. Reserpinum
D. * Verapamilum
E. Dibazolum
Назва наукового напрямку (модуля): Семестр: 6
Pharmacology (ситуаційні задачі) 2019-2020
Опис:
3 курс, факультет іноземних студентів
Перелік питань:
1. All of the following drugs are used topically in the treatment of glaucoma. Which of these agents
reduces intraocular pressure?
A. Atropin sulfas
B. Scopolamine
C. Diazepam
D. Isofluorphate
E. * Proserine
2. A 60-year-old male with congestive heart failure (CHF) is treated with dobutamine. Select the
mechanism of action of dobutamine.
A. Alpha-adrenergic agonist
B. Alpha-adrenergic antagonist
C. * Beta1-adrenergic agonist
D. Beta2-adrenergic agonist
E. Beta-adrenergic antagonist
3. A 58-year-old male with angina is treated with atenolol. Select the mechanism of action of atenolol.
A. Alpha-adrenergic agonist
B. Alpha-adrenergic antagonist
C. Beta-adrenergic agonist
D. * Beta-adrenergic antagonist
E. Mixed alpha and beta-agonist
4. A 75-year-old female with CHF is treated with carvedilol. Select the mechanism of action of
carvedilol.
A. Alpha-adrenergic agonist
B. Alpha-adrenergic antagonist
C. Beta-adrenergic agonist
D. Beta-adrenergic antagonist
E. * Mixed alpha and beta antagonist
5. A 35-year-old male with a pheochromocytoma is treated with labetalol. Select the mechanism of
action of labetalol.
A. Alpha-adrenergic agonist
B. Alpha-adrenergic antagonist
C. Beta-adrenergic agonist
D. Beta-adrenergic antagonist
E. * Mixed alpha and beta antagonist
6. A male patient is brought to the emergency department (ED) following ingestion of an unknown
substance. He is found to have an elevated temperature, hot and flushed skin, dilated pupils, and
tachycardia. Of the following, which would most likely cause these findings?
A. Propranolol
B. Proserine
C. Prazosin
D. Cromolyn
E. * Atropine
7. A 65-year-old male has a blood pressure of 170/105 mmHg. Which of the following would be
effective in lowering this patient’s blood pressure?
A. Pilocarpine
B. Cromolyn
C. Proserine
D. * Prazosin
E. Scopalamine
8. A 60-year-old diabetic male is treated with pyoglytazone. What is the mechanism of action of
pyoglytazone?
A. Increased release of endogenous insulin
B. Decreased plasma glucagon levels
C. Increased hepatic gluconeogenesis
D. * Increased target tissue sensitivity to insulin
E. Decreased intestinal absorption of glucose
9. A 47-year-old premenopausal female with endometriosis is treated with danazol. Which of the
following adverse effects is associated with danazol?
A. Weight loss
B. * Abnormal liver function tests
C. Thrombocytopenia
D. Heavy menses
E. Mood disorder
10. A patient with acute poisoning was admitted to intensive care unit. Indicate which drug should be
used for forced diuresis in this case.
A. * Furosemide
B. Theophylline
C. Unithiol
D. Furacilinum
E. Аlmagel
11. Patient with chronic heart failure for a long period of time used furosemide. Specify the irreversible
side effect which is typical for this drug.
A. * Deafness
B. Diarrhea
C. Headache
D. Vomiting
E. Constipation
12. Patient with heart failure used loop diuretic furosemide. After two days of treatment because of side
effects the drug was canceled. The next day the patient was diagnosed "rebound phenomenon". What
is it?
A. * Increased reabsorption of sodium and water
B. Increased excretion of calcium to the urine
C. Inhibition of reabsorption of magnesium ions
D. The appearance of metabolic acidosis
E. The appearance of respiratory acidosis
13. In a patient with acute renal failure. Glomerular filtration rate decreased from 125 ml / min. to 30 ml /
min. Indicate the drug, which diuretic effect may manifest in the following conditions:
A. * Furosemidum
B. Аtropini sulfate
C. Magnesium sulfate

D. Аsparcam
E. Furacilinum
14. In patient with hypertension the elevated level of rennin was found. The drug of which group would
be the best to use in this situation?
A. * ACE inhibitors
B. Myotropic vasodilators
C. Calcium channel blockers
D. Potassium channels activators
E. Sympatholytics
15. For treatment of patients with chronic heart failure cardiac glycosides have been used. What
pharmacokinetic parameter is used for calculating maintenance doses of digoxin?
A. * Coefficient of elimination
B. Bioavailability
C. Drug bound to plasma protein
D. Half-life period
E. Volume of distribution
16. The patient, who used to take diazepam for treatment of neurosis, diclofenac –sodium was prescribed
for removing symptoms of arthritis. What pharmacological phenomenon of drug-drug interaction
should be mentioned by physician in this case?
A. * Potentiation
B. Antagonism
C. Accumulation
D. Dependence
E. Tolerance
17. Immunosuppressant was prescribed for treatment of rheumatoid arthritis. Which of the following side
effects can you expect?
A. * All of the above
B. Susceptibility to infections
C. Thrombocytopenia
D. Violation of regeneration processes
E. Leukopenia
18. Atropine is used to treat the organophosphate compounds poisoning. What is the pharmacological
basis of this antidote?
A. * Physiological antagonism
B. Synergism
C. Addiction
D. Chemical antagonism
E. Accumulation
19. Patient with neuroses and symptoms of anxiety and fear was prescribed diazepam by physician. What
pharmacological effect of the drug is used to treat this disease?
A. * Anxiolytic
B. Antiarrhythmic
C. Antiphlogistic
D. Antianginal
E. Hypotensive
20. A patient was admitted to intensive care unit with signs of acute poisoning with mercury dichloride.
For the treatment of this patient used specific antidote, containing sulfhydryl groups. What is a drug?
A. * Unithiolum

B. Calcii chloridum
C. Kalii permanganas
D. Lidocainum
E. Naloxone
21. The patient suffered with bronchial asthma used remedy from group beta-adrenergic agonists. Which
of the following drugs is used in the therapy of bronchial asthma?
A. * Salbutamol
B. Nitrazepam
C. Oxytocinum
D. Calcii chloridum
E. Paracetamol
22. For the unconscious patient with symptoms of uncompensated metabolic acidosis, for correction pH
of tissues 100 ml 4 % solution of sodium hydrocarbonate in a form of enema was used. To what type
of drug administration does the rectal rout belong?
A. * Enteral
B. Injection
C. Inhalantion
D. Parenteral
E. There is no correct answer
23. The physician has requested “Хусaine (lidocaine) with adrenaline (epinephrine)”. Which of the
following provide the most accurate rationale for adding epinephrine?
A. * It causes vasoconstriction and keeps the anesthetic local
B. This combination is irrational
C. It enhances the pharmacological effects of the epinephrine
D. It helps calm the patient before the procedure
E. It helps minimize the risk of allergic reaction
24. A patient repeatedly used acetaminophen with codeine as pain relievers after tooth extraction. Which
of the following terms describes the development of non-beneficial reaction – constipation?
A. * Adverse effect
B. Therapeutic effect
C. Mutagenic effect
D. Teratogenic effect
E. Embriotoxic effect
25. A patient is taking hydrochlorothiazide for hypertension. Which of the following signs should be
monitored?
A. * Level of potassium ions
B. Liver transaminases
C. Kidney function
D. Thrombocytes level
E. Leukocytes level
26. In patients with chronic heart failure developed hypertensive crisis, which was complicated by
pulmonary edema. Which remedy diuretic should be used to treat this condition?
A. * Furosemidum
B. Nitrong
C. Codeini phosphas
D. Isadrinum
E. Papaverinum

27. A patient is diagnosed acute poisoning with aspirin. Choose the diuretic remedy that should be used
in treatment for forced diuresis.
A. * Furosemide
B. Nitrong
C. Asparkam
D. Festal
E. Bromhexin
28. The patient with acute heart failure dobutamine was injected. What is the mechanism of its
cardiotonic action?
A. * Stimulates beta-1-adrenergic receptors
B. Blocks K+, Na+ - ATP-azu
C. Stimulates M-cholinergic receptors
D. Blocks M-cholinergic receptors
E. Inhibits the activity of cyclooxygenase
29. Which drug would be the most rationale for hypertension and tachyarrhythmia?
A. * Metoprolol
B. Enalapril
C. Salbutamol
D. Minoxidil
E. Midazolam
30. Procedure of forced diuresis is used in the case of emergency care of acute intoxication. Indicate the
diuretic drug which is the most suitable for this procedure.
A. * Furosemide
B. Veroshpirone
C. Magnesii sulfas
D. Indapamide
E. Potassium chloride
31. Thiazide and related diuretics are inhibitors of Na+Cl-symport. Which of the following state is
indication for Hydrochlorothiazide?
A. * Congestive heart failure
B. Diarrhea
C. Constipation
D. Dementia
E. Abscess
32. After surgical tooth extraction the bleeding occurred in patient. Which drug could provoke it?
A. * Heparin
B. Fraxiparine
C. Reserpine
D. Pyrazinamide
E. Amoxiclav
33. Process of birthing is under the control of respiratory CNS center. The highest risk of respiratory
arrest is seen in which drug poisoning.
A. * Morphine
B. Acetominophen
C. Digitoxin
D. Norepinephrine
E. Metformine

34. Drug dependence is one of the adverse drug’s effects. Physical dependence is characterized by which
of the two following?
A. * tolerance and withdrawal symptoms
B. idiosyncrasy and allergy
C. carcinogenesis and toxicity
D. favism and formation of methemoglobin
E. embriotoxicity and congenital defects
35. A 25-year-old woman with red and itchy eczematous dermatitis visits your office. She had a dental
procedure one day earlier with administration of a local anesthetic. There were no other findings,
although she indicated that she had a history of allergic reactions. Which of the following drugs was
most likely involved?
A. * Lidocaine
B. Buprenorphine
C. Acebutolol
D. Aethimizolum
E. Captopril
36. On the 2-3rd day after stomach resection intestinal peristalsis wasn't restored. What is to be
administered for stimulation of gastrointestinal tract?
A. * Proserine
B. Clozapine
C. Prazosin
D. Cyclodole
E. Acetylcholine
37. A dentist injected a patient with a certain drug in order to reduce salivation during tooth filling. What
drug is it?
A. * Atropine sulfate
B. Ramipril
C. Prazosine
D. Clozapine
E. Losartan
38. Before the infiltration anesthesia was carried out, the test for novocaine sensitivity was made to the
patient. It was positive. Which of the following drugs can be used for anesthesia in this case?
A. * Xycainum
B. Streptomycinum
C. Cocainum
D. Anaesthesinum
E. Diсainum
39. The patient with hypertensive crisis used the drug from the group ganglionic blockers. What
complications can arise in this case?
A. * Orthostatic collapse
B. Hypertension
C. Cumulation
D. Hyperglycemia
E. Hypokalemia
40. The patient received antianginal drug that caused the effects of bronchospasm. Which of the
following drugs could cause such side effects?
A. * Anaprilinum
B. Ofloxacinum

C. Validol
D. Ramiprilum
E. Interferonum
41. The patient with gastric ulcer omeprazole was appointed. Which is antisecretory mechanism of drug
action?
A. * Blockade of Na / K ATPase
B. Stimulation of Na / K ATPase
C. Stimulation of adrenergic receptors
D. Adrenergic receptors blockade
E. Blockade of histamine receptors
42. In the intensive care unit patient admitted with symptoms of acute poisoning with arsenic. What a
means of should be used in this case as detoxification agent?
A. * Unithiolum
B. Interferonum
C. Adrenalin hydrochloridum
D. Sustac-
E. Validol
43. At the dentist's office patient began to complain of intense retrosternal pain that spread in the left half
of the face, jaw and teeth. Which of the following drugs is the most effective in this case?
A. * Nitroglycerinum
B. Interferonum
C. Analginum
D. Festal
E. Norepinephrine
44. A patient was prescribed with bile preparation for the purpose of improvement of rich food digestion.
What components of this preparation take part in fat emulsification?
A. * Bile acids
B. Cholesterol and its ethers
C. Diglyceride
D. Bilirubin-glucuronids
E. Higher fatty acids
45. Antivitamins are substances of various structure that limit utilization of vitamins in an organism and
have an opposite to them action. Name antivitamin of vitamin K:
A. Sulfapyridasine
B. * Dicumarol
C. Deoxypyridoxine
D. Aminopterin
E. Isoniazid
46. A patient with low immunity, frequent colds is recommended to take ascorutine as a more effective
drug than ascorbic acid. What constituent substance of this preparation intensifies action of vitamin
C?
A. Vitamin A
B. Glucose
C. Lactose
D. * Vitamin P
E. Vitamin D
47. In order to prevent adipose liver degeneration after viral hepatitis it is necessary to prescribe the
patient lipotropic factors. Name one of them:

A. Tryptophane
B. * Choline
C. Allopurinol
D. Contrical
E. Vicasol
48. Antibiotics are classified by sources of production. Name an antibiotic of bacterial origin:
A. Penicillin
B. Tetracycline
C. Lysozyme
D. * Gramicidin
E. Gentamycin
49. A woman in labour was given a preparation that activates contractions of smooth uterine muscles.
What hormone is contained in this preparation?
A. * Oxytocin
B. Gastrin
C. Secretin
D. Angiotensin
E. Bradykinin
50. A patient was prescribed with an antitumoral antibiotic that inhibits synthesis of nucleic acids in the
cells. What of the following antibiotics has such a mechanism of action?
A. Tetracycline
B. Nystatin
C. * Actinomycin
D. Lincomycin
E. Erythromycin
51. A patient has neurasthenic syndrome, diarrhea, dermatitis. This is associated with deficiency of the
following vitamin:
A. Vitamin K
B. Vitamin D
C. * Nicotinic acid
D. Folic acid
E. Vitamin B12
52. A patient has impaired mesopic vision, his photopic vision is normal. What is the probable cause of
such vision anomaly?
A. Hyperopia
B. * Vitamin A deficiency
C. Cones disfunction
D. Myopia
E. Vitamin D deficiency
53. Enzyme hyaluronidase breaks down hyaluronic acid thus increasing intercellular permeability. Which
vitamin strengthens vascular walls and inhibits activity of hyaluronidase?
A. A
B. B1
C. B2
D. * P
E. D

54. A patient was prescribed a bile preparation for better digestion of fatty food. What components of
this preparation cause fat emulsification?
A. Cholesterol and its ethers
B. Diglycerides
C. * Bile acids
D. Bilirubinglucuronids
E. Bile pigments
55. Antibiotics can be classified according to various principles. According to the action mechanism
cephalosporins relate to the following group:
A. Inhibitors of protein synthesis
B. * Inhibitors of cell wall synthesis
C. Inhibitors of respiratory processes
D. Inhibitors of oxidative phosphorilation
E. Inhibitors of cytoplasmic membrane synthesis
56. It is required to diminish pump function of patient’s heart. This can be done by means of blockers of
the following membrane cytoreceptors:
A. Nicotinic cholinoreceptors
B. Muscarinic cholinoreceptors
C. Alpha-adrenoreceptors
D. * Beta-adrenoreceptors
E. Dopamine receptors
57. A few minutes after repeated introduction of penicillin a patient got dyspnea, tongue numbness,
hyperemia and then skin pallor. The patient also lost consciousness. What is the cause of such a grave
condition?
A. Serum sickness
B. Hemolytic anemia
C. Acute glomerulonephritis
D. * Anaphylactic shock
E. Bronchial asthma
58. Introduction of glucocorticoids induces strengthening of glucose concentration in blood. Which of
the following processes will be activated in liver?
A. Glycogenolysis
B. * Gluconeogenesis
C. Oxidation of fatty acids
D. Ketogenesis
E. Glycolysis
59. A patient complains about gingival haemorrhage, petechial haemorrhages. What vitamin preparation
should be recommended?
A. * Ascorutinum
B. Thiamine hydrochloride
C. Cyanocobalamin
D. Nicotinic acid
E. Pyridoxine hydrochloride
60. A female patient has been treated with antibiotics for a long time. Thereafter examination of smears
form vaginal secretion revealed oval cells with well-defined nucleus, some cells gemmate. What
preparations can help to confirm the diagnosis "candidosis"?
A. Antibacterial
B. Antichlamydial

C. * Antifungal
D. Antiviral
E. Antiprotozoal
61. A patient takes blocker of muscarinic cholinoreceptors of parasympathetic nerve organ synapses.
What changes of heart activity will be observed?
A. Heart rate and heart force fall
B. * Heart rate rise
C. Heart rate fall
D. Heart force fall
E. Prolongation of atrioventricular delay
62. Anxious condition can be characterized by reduced salivation and sense of dry mouth. What mediator
is exuded out of nerve terminals innervating salivary glands?
A. Acetylcholine
B. Serotonin
C. Histamine
D. GABA
E. * Noradrenaline
63. What mediator provides information transmission from nerve terminations of motoneurons to the
fibers of skeletal muscles?
A. Adrenaline
B. Noradrenaline
C. * Acetylcholine
D. Serotonin
E. GABA
64. A patient has been administered a competitive inhibitor of cholinesterase. Name it:
A. Aspirin
B. Sodium diclophenac
C. Indometacin
D. * Proserin
E. Allopurinol
65. A 30 year-old patient suffering from pulmonary tuberculosis, has been prescribed isoniazid.
Continuous taking of this drug may lead to the deficiency of the following vitamin:
A. Tocopherol
B. * Pyridoxine
C. Cobalamin
D. Ergocalciferol
E. Retinol
66. It is known that infectious type B hepatitis is a systemic disease caused by the type B hepatitis virus
and characterized by a predominant liver affection. Choose from the below given list the drugs for
the etiotropic therapy of this infection:
A. Penicillin
B. Tetracycline
C. * Acyclovir
D. Sulfanilamides
E. Fluoroquinolones
67. Colloidal protection is used while manufacturing drug preparations. Name the preparation of
colloidal silver protected by proteins:

A. Festal
B. Enzymtal
C. Argentum
D. * Protargol
E. Collagen
68. Water-soluble vitamins take coenzyme form in an organism. Thiamine diphosphate is coenzyme of
the following vitamin:
A. B2
B. C
C. * B1
D. B6
E. B12
69. Enzymes (biological catalysts) are used as pharmacologic preparations. What is the mechanism of
enzyme action in the biochemical reactions?
A. They increase the energy of reaction activation
B. * They reduce the energy of reaction activation
C. They inhibit the reaction process
D. They change the constant of the reaction rate
E. They change the reaction order
70. To relax skeletal muscles during complex surgeries, curarelike substances are applied. These
substances block the following structure:
A. Vegetative ganglions
B. Basal ganglions
C. Red nuclei of the mesencephalon
D. Synaptic structures of the spinal cord
E. * Neuromuscular synapses
71. An elderly woman complains of twilight vision impairment. Which of the following vitamins should
be administered in this case?
A. D
B. C
C. E
D. * A
E. PP
72. A drug store received a supply of a drug that is widely used for treatment of many virus diseases
since it is not virus specific. What drug is it?
A. Remantadin
B. * Interferon
C. Tetracycline
D. Immunoglobulin
E. Vaccine
73. Nonsteroid anti-inflammatory drugs are used in medical practice for treating the rheumatoid arthritis,
osteoporosis, inflammatory deseases of the connective tissue. These preparations inhibit the activity
of the following enzyme:
A. Hexokinase
B. Succinate dehydrogenase
C. Aminotransferase
D. * Cyclooxygenase

E. Xanthine oxidase
74. Cardiac diseases are treated with cocarboxylase preparation. This preparation is the coenzymatic
form of the following vitamin:
A. P
B. B6
C. B12
D. C
E. * B1
75. A woman in labour has been given a drug that activates contractions of the smooth muscles of uterus.
Which hormone is a part of this drug?
A. Angiotensin
B. Gastrin
C. Secretin
D. * Oxytocin
E. Bradykinin
76. A patient fell ill the day before, the disease is acute with a predominance of general toxic symptoms.
With an account for the epidemic situation in the city, the doctor diagnosed the patient with influenza
A. What emergency etiotropic treatment must be administered to this patient?
A. Oxolinic ointment
B. * Rimantadine
C. Gentamicin
D. Inactivated influenza vaccine
E. Human gamma globulin
77. A 3,5-year-old child has been diagnosed with dysbacteriosis in form of critical reduction of
gram-positive anaerobic bacteria and increased number of staphylococci and yeast fungi. Which
preparation should be used for the correction of dysbacteriosis?
A. Colibacterin
B. Coli-Proteus bacteriophage
C. * Bifidumbacterin
D. Furazolidone
E. Lactoglobulin
78. Drugs that block certain channels can prevent the transmission of excitation from presynaptic
membrane to the postsynaptic memebrane of synapse. What channels are blocked?
A. * Calcium
B. Sodium
C. Potassium potential-dependent
D. Potassium ATP-dependent
E. Chlorine
79. Depressive states can be treated by means of drugs inhibiting the enzyme that inactivates biogenic
amines. Specify this enzyme:
A. LDH (lactate dehydrogenase)
B. CPK (creatine phosphokinase)
C. * MAO (monoamine oxidase)
D. AST (aspartate aminotransferase)
E. ALT (alanine aminotransferase)
80. Sulfonamides are widely used as bacteriostatic agents. The mechanism of antimicrobial action of
sulfonamides is based on their structural similarity to:
A. Antibiotics

B. Glutamic acid
C. Folic acid
D. Nucleic acid
E. * Para-aminobenzoic acid
81. Before a surgical operation, a surgeon treated his hands with an alcohol-containing solution. Which
group of drugs does this solution relate to?
A. Detergents
B. Disinfectants
C. Sterilizing solutions
D. * Antiseptics
E. Surface-active substances
82. A patient with gastric ulcer has been administered omeprazole. What is the mechanism of its action?
A. Neutralization of HCl
B. Blockade of histamine H2-receptors
C. M-cholinergic receptor blockade
D. * Inhibition of H +K+-ATPase
E. Stimulation of mucus production
83. A patient with a heart rhythm disorder has been given lidocaine. Apart from the local anesthetic
effect, this drug has the following pharmacological effect:
A. Hypnotic
B. Antipyretic
C. * Antiarrhythmic
D. Antidepressant
E. Nootropic
84. A patient with chronic constipation has been prescribed bisacodyl. After 3 weeks of treatment, the
patient noticed a reduction of laxative effect. This is caused by the development of the following
side-effect:
A. Dependence
B. * Habituation
C. Sensibilization
D. Cumulation
E. Dysbacteriosis
85. A patient with acute heart failure and cardiac glycosides intolerance was given an injection of
dobutamine. What is the mechanism of its action?
A. Stimulation of alpha1-adrenoceptors
B. Blockade of K+-, Na+-ATPase
C. Inhibition of phosphodiesterase activity
D. * Stimulation of beta1-adrenoceptors
E. Stimulation of M-cholinergic receptors
86. A female patient asked a pharmacist to recommend her a drug for headache with antiplatelet effect.
Specify this drug:
A. Promedol
B. Codeine phosphate
C. * Acetylsalicylic acid
D. Tramadol
E. Fentanyl

87. A patient with renal colic has been administered a spasmolytic from the group of M-cholinergic
antagonists as a part of the complex therapy. Specify this drug:
A. Proserin
B. * Atropine
C. Galantamine
D. Dithylinum
E. Benzohexonium
88. A pharmacy dispenses glaucine hydrochloride to a patient with chronic bronchitis. The patient must
be warned about the following typical side effect of the drug:
A. Arrhythmia
B. Excitation of the central nervous system
C. * Blood pressure fall
D. Rise of intraocular pressure
E. Allergic skin rash
89. A female student with a cold has been prescribed an antipyretic medication. Specify this drug:
A. Ascorbic acid
B. Oxytocin
C. * Paracetamol
D. Famotidine
E. Cyanocobalamin
90. A patient has been taking diclofenac sodium for a long time. The family physician withdrew this drug
and administered celecoxib. What disease was the reason for the drug substitution?
A. Bronchial asthma
B. * Peptic ulcer
C. Urolithiasis
D. Arterial hypertension
E. Chronic hepatitis
91. A patient has been diagnosed with bronchial asthma. Specify a medicament that can be administered
for asphyxiation:
A. Paracetamol
B. Diclofenac sodium
C. * Salbutamol
D. Anapriline
E. Acetylcysteine
92. Sulfanilamides inhibit the growth and development of bacteria. The mechanism of their action is
based on the impairment of the following acid synthesis:
A. Nicotinic
B. Lipoic
C. * Folic
D. Pantothenic
E. Pangamic
93. Sulfonamides are widely used as bacteriostatic agents. The mechanism of antimicrobial action of
sulfanilamides is based on their structural similarity to:
A. Glutamic acid
B. * Para-aminobenzoic acid
C. Folic acid
D. Nucleic acid

E. Antibiotics
94. A patient with symptoms of chronic bronchitis has been administered acetylcysteine. What is the
mechanism of its expectorant action?
A. Stimulation of adrenergic receptors
B. Inhibition of cough center
C. * Depolymerization of sputum mucopolysaccharides
D. Stimulation of respiratory center
E. Anesthesia of respiratory mucosa
95. A 40-year-old patient has a history of bronchial asthma and bradyarrhythmia. In order to eliminate
bronchospasm, the drugs of the following pharmacological group should be administered
A. Beta-adrenergic blocking agents
B. * M-anticholinergics
C. M-cholinergic agents
D. Anticholinesterase agents
E. Muscle relaxants
96. The anti-tumor preparation Methotrexate is a structural analogue of folic acid. The mechanism of its
action is based on the inhibition of the following enzyme:
A. Xanthine oxidase
B. Hexokinase
C. * Dihydrofolate reductase
D. Creatine kinase
E. Lactate dehydrogenase
97. A 40-year-old female farmworker has been diagnosed with brucellosis and administered causal
chemotherapy. What group of drugs will be used for this purpose?
A. Donor immunoglobulin
B. * Antibiotic
C. Inactivated therapeutic vaccine
D. Polyvalent bacteriophage
E. Antitoxic serum
98. A parturient woman diagnosed with uterine inertia has been delivered to the maternity ward. The
doctor gave her an injection of the drug that activates the contraction of smooth muscles of the
uterus. What hormone is a component of this drug?
A. Gastrin
B. Secretin
C. Angiotensin
D. * Oxytocin
E. Bradykinin
99. A patient with chronic constipation had been prescribed bisacodyl. After 3 weeks of treatment, the
patient noticed a reduction of laxative effect. This is caused by the development of the following side
effect:
A. Dependence
B. * Tolerance
C. Sensibilization
D. Cumulation
E. Dysbacteriosis
100. Diet of a human must contain vitamins. What vitamin is usually prescribed for treatment and
prevention of pellagra?
A. Vitamin C

B. * Vitamin PP
C. Vitamin A
D. Vitamin B1
E. Vitamin D
101. A woman noticed that a cut on her skin was still bleeding even after 20 minutes had passed. What
vitamin deficiency causes such condition?
A. Vitamin A
B. Vitamin D
C. * Vitamin K
D. Vitamin E
E. Vitamin B12
102. Pharmacy has received viricides. Choose the viricide used for influenza treatment from the list given
below:
A. * Rimantadine
B. Metisazone
C. Levamisole
D. Azidothimidine
E. Acyclovir
103. A patient with hyperproduction of thyroid hormones has been prescribed Merkazolilum. This drug
inhibits the following enzyme of iodothyronine synthesis:
A. Aromatase
B. * Iodide peroxidase
C. Reductase
D. Decarboxylase
E. Aminotransferase
104. Neuroleptanalgesia has been applied in the case of cardiac infarction. What neuroleptic is most often
applied along with fentanyl?
A. Perphenazine (Aethaperazinum)
B. Levomepromazine
C. Clozapine
D. Sulpiride
E. * Droperidol
105. Due to prolonged taking of phenobarbital the epileptic patient has developed tolerance for this drug.
What is this phenomenon based on?
A. Absorption process weakening
B. Increase of receptor sensitivity
C. * Biotransformation acceleration
D. Biotransformation suppression
E. Substance accumulation in body
106. During treatment of chronic cardiac failure with digitoxin a patient developed the drug-specific signs
of intoxication. A doctor prescribed Unithiol (Dimercaptopropansulfonate sodium). Explain its
mechanism of action of Unithiol in case of cardiac glycoside intoxication:
A. Binding of calcium ions
B. Increase of sodium concentration in cardiac hystiocytes
C. * Restoration of K+-Na+-adenosine triphosphatase activity
D. Increase of calciun permeability of cardiac hystiocytes
E. Binding of glycosides into complex compound

107. An elderly patient suffers from constipation caused by colon hypotonia. What drug should be
prescribed?
A. Sodium sulfate
B. Magnesium sulfate
C. * Bisacodyl
D. Atropine sulfate
E. Novocainamide (Procainamide)
108. In the course of bronchitis pharmacotherapy a patient has developed dyspeptic disorders,
photodermatitis and hepatic failure. What drug can cause such disorders?
A. Paracetamol
B. Ascorbic acid
C. Acetylcysteine
D. * Doxycycline
E. Codeine phosphate
109. The patient with rheumatoid arthritis and concomitant duodenal ulcer has to be prescribed a
nonsteroid antiinflammatory drug. Which one of the drugs listed below is the drug of choice in the
given case?
A. Acetylsalicylic acid
B. Paracetamol
C. Metamizole
D. * Celecoxib
E. Diclofenac sodium
110. The patient with bronchial asthma had been prescribed salbutamol, which led to disappearance of
bronchiospasm symptoms. It happened due to stimulation of:
A. Alpha1-adrenoreceptors
B. Muscarinic acetylcholine receptors
C. * Beta2-adrenoreceptors
D. Acetylcholine synthesis
E. Beta1-adrenoreceptors
111. A doctor has prescribed a nonsteroidal anti-inflammatory drug to relieve inflammation and pain
syndrome. Name this drug:
A. Glibenclamide
B. * Diclofenac sodium
C. Loratadine
D. Prednisolone
E. Calcium chloride
112. A 55-year-old patient had been diagnosed with angina pectoris. Calcium channel-blocking agent was
prescribed for treatment. Name this agent:
A. Atenolol
B. Guanethidine
C. * Amlodipine
D. Reserpine
E. Labetalol
113. A woman is to be prescribed a narcotic analgesic for labor pain relief. What drug is indicated in this
case?
A. Morphine
B. * Promedol (Trimeperidine)
C. Papaveretum (Omnopon)

D. Codeine
E. Fentanyl
114. A woman, who during the 5th-10th weeks of her pregnancy had been taking sodium valproate for
treatment of her epilepsy, gave birth to a child with pathology of the vertebral column (split spine).
What side effect of the drug caused such malformation?
A. Mutagenic
B. Embryotoxic
C. Fetotoxic
D. Sensitizing
E. * Teratogenic
115. A patient with acute bronchitis was prescribed an expectorant that caused bronchial spasm after the
patient had taken it. What drug of those listed below can cause such side effect?
A. Salbutamol
B. Validol (Menthyl isovalerate)
C. * Acetylcysteine
D. Platyphyllin
E. Prenoxdiazine (Libexin)
116. Prolonged application of broad spectrum antibacterial drugs resulted in the patient being hospitalised
with diagnosis of candidiasis. What side effect of antibiotic therapy has developed in the patient?
A. Endotoxic reaction
B. Toxic reaction
C. * Disbacteriosis
D. Allergic reaction
E. Formation of resistant microorganism strains
117. A 37-year-old patient with peptic gastric ulcer disease was prescribed a medicine as a part of his
multimodality therapy. The medicine lowers acidity of gastric juice, inhibits- Na+, K+ adenosine
triphosphatase, decreases the volume of gastric secretion and pepsinogen production. It is a prodrug.
Name this medicine:
A. Famotidine
B. Gastrozepin (Pirenzepine)
C. Ranitidine
D. * Omeprazole
E. Phosphalugel (Aluminium phosphate)
118. A pharmaceutical manufacture produces a drug, that is an animal antibiotic. Point out this drug
among those listed below:
A. * Lysozyme
B. Gramicidin
C. Novobiocin
D. Phaseolin
E. Chloramphenicol
119. A patient has been receiving Theophylline – inhibitor of cyclic adenosine monophosphate
phosphodiesterase – for a week. What hormone can increase its action and cause hyperglycemia?
A. * Glucagon
B. Testosterone
C. Aldosterone
D. Insulin
E. Estradiol

120. Adrenomimetic agents are differentiated into selective and non-selective. What drug is an agonist of
beta2-adrenergic receptors and can be used for treatment of bronchial asthma?
A. * Salbutamol
B. Metoprolol
C. Atenolol
D. Anaprilin (Propranolol)
E. Nebivolol
121. A local general practitioner recommends taking interferon for influenza prevention. What is the
mechanism of action of this drug?
A. Blocks virus stripping
B. * Blocks virus protein synthesis
C. Inhibits virion exit from cells
D. Prevents adsorption of virus in cell receptors
E. Disrupts the process of virus assembly
122. A doctor prescribed diazepam to a patient with anxiety disorders. What pharmacological effect is the
reason for such a prescription?
A. Anticonvulsant
B. Anti-inflammatory
C. * Anxiolytic
D. Antianginal
E. Hypotensive
123. Dobutamine has been administered to the 49-year-old-patient with acute cardiac failure and cardiac
glycoside intolerance. What is this drug’s mechanism of action?
A. Stimulation of M-cholinergic receptors
B. Stimulation of alpha1-adrenoreceptors
C. Blockade of K+-, Na+-adenosinetriphosphatase
D. Suppression of phosphodiesterase activity
E. * Stimulation of beta1-adrenoreceptors
124. The patient with hepatic colic has been prescribed spasmolytic of muscarinic receptor antagonists
group as a part of his complex therapy. What drug is it?
A. Proserin
B. Galantamine
C. * Atropine
D. Dithylin
E. Benzohexonium
125. The student with cold has been prescribed antipyretic medicine. Name this medicine.
A. Ascorbic acid
B. Oxytocin
C. Famotidine
D. * Paracetamol
E. Cyanocobalamin
126. Epinephrine is used to prolong the effect of novocaine during infiltration anesthesia. What
epinephrine action is this effect caused by?
A. Suppression of tissue esterases
B. Potentiation of novocaine action at CNS level
C. Suppression of nerve endings and conductors functioning
D. Vasodilatation

E. * Vasoconstriction
127. Oligomycin antibiotic is prescribed to the patient with tuberculosis. What mitochondrial process is
slowed down by this medicine?
A. Substrate-linked phosphorylation
B. * Oxidative phosphorylation
C. Microsomal oxidation
D. Lipid peroxidation
E. Oxidative decarboxylation
128. The patient has been prescribed drug with antibacterial effect on tuberculosis mycobacteria. What
drug is used in tuberculosis treatment and is pyridoxine antivitamin?
A. * Isoniazid
B. Heparin
C. Trimethoprim/sulfamethoxazole (Co-trimoxazole)
D. Streptomycin
E. Sulfanilamide
129. Certain drugs can stimulate liver to synthesize enzyme systems taking part in drugs and toxines
metabolism. What compound stimulates drug metabolism in liver microsomes?
A. Heparin
B. Menadione sodium bisulfite
C. * Phenobarbital
D. Sulfanilamide
E. Aspirin
130. The 32-year-old patient has been taking antituberculosis drugs. Later he noticed that his urine had
become red-orange in color. What drug is conductive to this phenomenon?
A. Isoniazid
B. Pyrazinamide
C. Ethambutol
D. * Rifampicin
E. Streptomycin sulphate
131. The 33-year-old female patient, who undergoes long-term treatment due to her chronic polyarthritis,
complains of increased arterial pressure, adipose tissue redistribution and menstrual irregularities.
What drug does the patient take?
A. Indometacin
B. * Prednisolone
C. Phenylbutazone
D. Fluocinolone acetonide
E. Diclofenac sodium
132. The patient with acute poisoning needs forced diuresis. What drug can be used for this purpose?
A. Caffeine and sodium benzoate
B. * Furosemide
C. Galantamine hydrobromide
D. Enalapril
E. Piracetam
133. The patient with parkinsonism has been prescribed a drug – dopamine precursor – to relieve muscular
rigidity. Name this drug.
A. Aminazine
B. Paracetamol

C. Scopolamine hydrobromide
D. * Levodopa
E. Atropine sulphate
134. Before tooth extraction a patient was advised to take a certain drug for hemorrhage prevention. What
drug was advised?
A. * Vikasol
B. Heparin
C. Asparcam
D. Magnesium sulphate
E. Dimedrol
135. On the 4th day of treatment with indometacin a 55-year-old patient developed gastrorrhagia as a
result of gastric mucosa ulcer. Ulcerogenic effect of this drug is explained by the lowered activity of
the following enzyme:
A. * Cyclooxygenase-1 (COX-1)
B. Cyclooxygenase-2 (COX-2)
C. Lipoxygenase (LOG)
D. Prostacyclin synthetase
E. All the above
136. A 45-year-old patient has been taking neodicumarinum for thrombophlebitis during two weeks. The
regular blood test revealed a decrease in prothrombin concentration, microhematuria. Which drug
should be used as neodicumarinum antagonist?
A. * Vikasol
B. Protamine sulphate
C. Sodium citrate
D. Heparin
E. Aminocaproic acid
137. A 33-year-old woman who has been treated for chronic polyarthritis for a long time complains about
arterial pressure rise, change of adipose tissue distribution, menstrual cycle disorder. What
preparation has she taken?
A. * Prednisolone
B. Indometacin
C. Butadion
D. Synaflan
E. Diclofenac sodium
138. A patient suffering from essential hypertension has increased concentration of rennin in blood
plasma. Which pharmacological group should be preferred for this patient treatment?
A. * ACE inhibitors
B. Alpha-adrenoblockers
C. Diuretics
D. Blockers of calcium channels
E. Sympatholytics
139. A patient suffering from stomach ulcer was prescribed almagel. Which of its pharmacological
properties is intended for treatment of this pathology?
A. * HCl neutralization
B. Local anaesthetization
C. Blocking the H2-histamine receptors
D. Blocking the muscarinic cholinoreceptors
E. Anti-inflammatory action

140. A patient suffering from hypertension consulted a doctor about dry cough that was presumably
provoked by antihypertensive therapy. What antihypertensive drug had she been taking?
A. * Lisinopril
B. Sodium chloride
C. Nifedipin
D. Furosemide
E. Hydrochlorthiazide
141. A patient suffering from ulcer disease was prescribed famotidine. What is the mechanism of its
action?
A. * Block of H2-histamine receptors
B. Block of H1-histamine receptors
C. Block of muscarinic cholinoreceptors
D. Inhibition of adenosine triphosphatase activity
E. Block of cholinoreceptors of sympathetic ganglions
142. A patient after bee stings has angioedema (Quincke's edema). Which drug should be immediately
injected to eliminate this condition?
A. * Epinephrine hydrochloride
B. Sodium chloride
C. Platyphyllini hydrotartras
D. Atropine sulphate
E. Furosemide
143. A patient with hypertension was administered metoprolol for the arterial pressure reduction. What is
its mechanism of action?
A. * Beta adrenoreceptor blockade
B. Alpha adrenoreceptor blockade
C. Indirect adrenomimetic action
D. All-cholinoreceptors blockade
E. Angiotensin receptors blockade
144. A 34-year-old patient suffering from bronchitis was prescribed an antitussive drug of central action.
What drug is it?
A. * Glaucine
B. Corglycon
C. Enalapril
D. Furosemide
E. Fercoven
145. What phenomenon does develop when we can see the decreasing of efficacy of pharmaceutical drug
due to its continuous administration:
A. * Habituation (Tolerance)
B. Sensibilization
C. Accumulation
D. Dependence
E. Tachyphylaxis
146. A patient was warned that taking the prescribed preparation might cause cough. What drug is it?
A. * Lisinopril
B. Clonidine
C. Phenihidine
D. Dichlothiazide

E. Metoprolol
147. A patient with acute cardiac insufficiency and cardiac glycoside intolerance got an injection of
dobutamine. What is the mechanism of its action?
A. * Stimulation of beta1-adrenoreceptors
B. Stimulation of alpha1-adrenoreceptors
C. Block of K+-, Na+-adenosine triphosphatase
D. Inhibition of phosphodiesterase activity
E. Stimulation of muscarinic cholinoreceptors
148. For thrombosis treatment a patient was prescribed a drug from the group of direct anticoagulants.
What drug is it?
A. * Heparin
B. Fenilin (phenylinum)
C. Syncumar
D. Neodicumarin
E. Vicasol
149. A patient suffering from chronic cardiac insufficiency was prescribed a cardiac glycoside from the
foxglove (Digitalis) group. What drug is it?
A. * Digoxin
B. Strophanthine
C. Corvalol
D. Corglycon
E. Cordiamin
150. A patient who has been suffering from gastric diseases for a long time is found to have hypochromic
anemia. Which of the following drugs is applied for this pathology treatment?
A. * Cyanocobalamin
B. Ascorbic acid
C. Unitiol
D. Ferrum Lek
E. Ferriscorbone
151. A man came to a pharmacy and asked to recommend him a drug for treatment of allergic rhinitis that
occurs in the period of linden flowering. What drug may be applied?
A. * Loratadine
B. Adrenaline
C. Anapriline
D. Ambroxol
E. Losartan
152. It is necessary to prescribe non-steroid anti-inflammatory drug for patient with rheumatoid arthritis
and concomitant duodenal ulcer. Which drug is the drug of choice in this case?
A. * Celecoxib
B. Acetylsalicylic acid
C. Paracetamol
D. Analgin
E. Diclofenac sodium
153. A patient has been taking isosorbide for stenocardia prevention for quite a long time. Now he notes a
significant decrease in the effect of the drug. What is this phenomenon called?
A. * Tolerance
B. Accumulation
C. Physical dependence

D. Allergy
E. Potentiation
154. In the complex medical treatment of hypertensive disease a diuretic was prescribed to the patient. In a
few days the BP went down, but the signs of hypokalemia arose up. What drug could cause such
complication?
A. Triamterene
B. Clofeline
C. Enalapril
D. Spironolactone
E. * Furosemide
155. In the patient with the considerable peripheral edema using of Dichlothiazide did not result in the
considerable diuretic effect. The analysis of blood indicated the considerable increasing of aldosteron
level. Prescribe drug for treatment in this situation.
A. Mannitol
B. * Spironolactone
C. Ethacrynic acid
D. Furosemide
E. Diacarb
156. The alternate usage of dichlotiazide, etacrin acid and lasex did not cause marked diuretic effect in the
patient with marked peripheral edema. The aldosterone level in the blood is increased. Indicate which
medicine should be prescribed:
A. Mannitol
B. Amiloride
C. Clopamide
D. Urea
E. * Spironolactone
157. The pregnant woman was delivered to the maternity hospital urgently with threat of premature births.
What drug is necessary to prescribe in this situation?
A. Ergometrine hydrotartrate
B. Kotarnini chloride
C. Pilocarpine hydrochloride
D. Oxytocin
E. * Partusisten
158. To the 55 years old man for the medical treatment of gout was prescribed ethanamide. What is the
mechanism of uric actions of the drug?
A. * Depression of reabsorbing of urinary acid
B. Increasing of secretion of urinary acid
C. Decreasing of production of urinary acid
D. Depression of activity of xantinoxydase
E. Production of easy soluble salts
159. In oncologic patients the cellular immunity decreased after the radial therapy. Prescribe the proper
drug.
A. Prodigiosan
B. Interferon
C. * T-aktivin
D. Betaferon
E. Reaferon

160. In the patient of 55 years old with stomach ulcer on the 4-th day of medical treatment by
indomethacin the gastric bleeding appeared. What is the mechanism of ulcer action of drug?
A. * Decreasing of production of prostaglandin Р•1
B. Decreasing of production of prostaglandin Р•2
C. Decreasing of production of leucotrienes
D. Decreasing of production of cyclic endoperexes
E. Decreasing of production of tromboxan
161. It is known, that the nonsteroidal anti-inflammatory drugs widely use for the medical treatment of
rheumatism. In what phase of inflammatory process do they influence?
A. * Exudative
B. Phase of alteration
C. Phase of proliferation
D. All phases
E. Autoimmune
162. Loratadine was prescribed to the patient with allergic dermatitis. What group of anti-allergic drugs
does this drug belong to?
A. Glucocorticosteroids
B. * Blockers of histamine receptors
C. Membranostabilisators
D. Antagonists of leukotriene receptors
E. Blockers of serotonin receptors
163. Man life environment ecologically dangerous. The radiation and others harmful factors damage DNA
and negatively affect a man state. Which vitamin drug does reparation of DNA assist and do
normalize development of man?
A. * Trio Vit
B. Nicotinic acid
C. Ergocalciferol
D. Riboflavin
E. Lipoic acid
164. Note the drug regulates functions of CNS, heart, GI tract, muscular system, blood coagulation. Forms
and saves structure of bone tissue. Decrease vessels permeability?
A. Sodium
B. Potassium
C. * Calcium
D. Magnesium
E. Trisamin
165. Patient with acute rheumatism treated oneself by salicylates. After some time the insomnia appeared
at him. When the doctor prescribed Phenobarbital, the anti-inflammatory action of salicylates became
weak. How to explain weakening an action of salicylates in presence of Phenobarbital?
A. * Induction of liver enzymes
B. Inhibition enzymes of liver
C. Chemical antagonism
D. Physiology antagonism
E. Acceleration of selection of salicylates from organism
166. The patient (60 years old) with diagnosis rheumatoid arthritis use nonsteroidal anti-inflammatory
drug indomethacin for a long time. What is the mechanism of action of this drug?
A. Blockade of enzyme of lipooxygenase
B. Blockade of enzyme of acetylcholinesterase

C. Blockade of enzyme of phospholipase


D. * Blockade of enzyme of cyclooxygenase
E. Blockade of enzyme of phosphodiesterase
167. Patient complaints on the pain in knee-joints. The doctor found slight swelling, hyperemia,
hyperthermia in the area of these joints. What drugs did the doctor prescribe for treating such patient?

A. * Nonsteroid anti-inflammatory drugs


B. Narcotic analgesics
C. Antidepressants
D. Antibiotics
E. Sulfanilamides
168. Patient suffers on allergy. Loratadine is prescribed as antiallergic agent. Loratadine (Claritin) has an
advantage over traditional antihistamines such as diphenhydramine (Benadryl) in that loratadine has:
A. Less risk of cardiac arrhythmias
B. * Less sedative effect
C. Increased bronchodilating effects
D. Less gastrointestinal upset
E. Increased gastrointestinal effects
169. A patient with rheumatoid arthritis who had been treated with indomethacin has got signs of
gastropathy. What activity of the drug can this complication be connected with?
A. * Anti Cyclooxygenase
B. Antikinine
C. Antiserotonin
D. Antihistamine
E. Locally irritating
170. A 23-year-old graduate student goes on a cruise to the Bahamas. She considers taking a drug for
sea-sickness as she is prone to developing motion sickness. Which compound and time of
administration would be most effective?
A. Transdermal scopolamine taken during the voyage
B. Fexofenadine taken 4 hours prior to sailing
C. * Transdermal scopolamine taken 4 hours prior to sailing
D. Cauterizing taken 1 hour prior to sailing
E. Metoclopramide taken during the voyage
171. Antivitamins are substances of various structure that limit utilization of vitamins in an organism and
have an opposite to them action. Name antivitamin of vitamin K:
A. Sulfapyridasine
B. Deoxypyridoxine
C. * Dicumarol
D. Aminopterin
E. Isoniazid
172. A patient with low immunity, frequent colds is recommended to take ascorutine as a more effective
drug than ascorbic acid. Which of the following is the active substance of this preparation which
intensifies action of vitamin C?
A. Vitamin A
B. Glucose
C. Lactose
D. * Vitamin P
E. Vitamin D

173. In order to prevent adipose liver degeneration after viral hepatitis it is necessary to prescribe the
patient lipotropic factors. Name one of them:
A. * Choline
B. Tryptophan
C. Allopurinol
D. Contrical
E. Vicasol
174. Antibiotics are classified by sources of production. Name an antibiotic of bacterial origin:
A. Penicillin
B. Tetracycline
C. Lysozyme
D. * Gramicidin
E. Gentamicin
175. A woman in labour was given a preparation that activates contractions of smooth uterine muscles.
What hormone is contained in this preparation?
A. Gastrin
B. Secretin
C. Angiotensin
D. * Oxytocin
E. Bradykinin
176. Patients ill with tuberculosis take a drug that is an antivitamin of nicotinic acid. Name this substance:
A. * Isoniazid
B. Sulfanilamide
C. Acrichine
D. Isoriboflavin
E. Oxythiamine
177. A patient was prescribed with an antitumoral antibiotic that inhibits synthesis of nucleic acids in the
cells. What of the following antibiotics has such a mechanism of action?
A. Tetracycline
B. Nystatin
C. * Actinomycin
D. Lincomycin
E. Erythromycin
178. A patient has neurasthenic syndrome, diarrhea, dermatitis. This is associated with deficiency of the
following vitamin:
A. * Nicotinic acid
B. Vitamin K
C. Vitamin D
D. Folic acid
E. Vitamin B12
179. Antibiotics can be classified according to various principles. According to the action mechanism
cephalosporins relate to the following group:
A. Inhibitors of protein synthesis
B. Inhibitors of respiratory processes
C. Inhibitors of oxidative phosphorilation
D. * Inhibitors of cell wall synthesis
E. Inhibitors of cytoplasmic membrane synthesis

180. It is required to diminish pump function of patients heart. This can be done by means of blockers of
the following membrane cytoreceptors:
A. Nicotinic cholinoreceptors
B. * Beta-adrenoreceptors
C. Muscarinic cholinoreceptors
D. Alpha-adrenoreceptors
E. Dopamine receptors
181. A few minutes afer repeated introduction of penicillin a patient got dyspnea, tongue numbness,
hyperemia and then skin pallor. The patient also lost consciousness. What is the cause of such a grave
condition?
A. Serum sickness
B. Hemolytic anemia
C. Acute glomerulonephritis
D. * Anaphylactic shock
E. Bronchial asthma
182. A patient complains about gingival haemorrhage, petechial haemorrhages. What vitamin preparation
should be recommended?
A. * Ascorutinum
B. Thiamine hydrochloride
C. Cyanocobalamin
D. Nicotinic acid
E. Pyridoxine hydrochloride
183. A female patient has been treated with antibiotics for a long time. Thereafter examination of smears
form vaginal secretion revealed oval cells with well-defined nucleus, some cells gemmate. What
preparations can help to confirm the diagnosis "candidosis"?
A. Antibacterial
B. Antichlamydial
C. * Antifungal
D. Antiviral
E. Antiprotozoal
184. A 30 year-old patient suffering from pulmonary tuberculosis, has been prescribed isoniazid.
Continuous taking of this drug may lead to the deficiency of the following vitamin:
A. * Pyridoxine
B. Tocopherol
C. Cobalamin
D. Ergocalciferol
E. Retinol
185. A patient has taken a large dose of a barbiturate hypnotic (amytal) that inhibits cardiac diseases are
treated with cocarboxylase preparation. This preparation is the coenzymatic form of the following
vitamin:
A. * B1
B. B6
C. B12
D. C
E. P
186. It is known that infectious type B hepatitis is a systemic disease caused by the type B hepatitis virus
and characterized by a predominant liver affection. Choose from the below given list the drugs for
the etiotropic therapy of this infection:

A. * Acyclovir
B. Penicillin
C. Tetracyclines
D. Sulfanilamides
E. Fluoroquinolones
187. Colloidal protection is used while manufacturing drug preparations. Name the preparation of
colloidal silver protected by proteins:
A. * Protargol
B. Festal
C. Enzymtal
D. Argentum
E. Collagen
188. It is required to diminish pump function of patients heart. This can be done by blocking the following
membrane cytoreceptors:
A. * Beta-adrenoreceptors
B. Nicotinic cholinoreceptors
C. Muscarinic cholinoreceptors
D. Alpha-adrenoreceptors
E. Dopamine receptors
189. Staphylococci were isolated in pure culture from a patient with sepsis. These were staphylococcal
producing beta lactamase. Such property should be taken into account when:
A. * Choosing an antibiotic for treatment
B. Determining biiochemical properties
C. Determining the strain pathogenicity
D. Differentiating specific types of staphylococci
E. Choosing optimal conditions for cultivation
190. A man tip of tongue was processed with an anesthetic solution. Therefore he will lose the sense of the
following taste:
A. * Sweet
B. Bitter
C. Sour
D. Salty
E. Bitter and salty
191. Anti-inflammatory effect of a number of drugs is caused by the inhibition of arachidonic acid release.
This acid is the precursor of:
A. * Prostaglandins
B. Uric acid
C. Urea
D. Haem
E. Cholesterol
192. To prevent fatty degeneration of liver after viral hepatitis, a patient should be administered lipotropic
factors. Indicate one of them:
A. * Choline
B. Tryptophan
C. Allopurinol
D. Contrical
E. Vicasol

193. A man got an injection of curare like substance causing the relaxation of all skeletal muscles. What is
its mechanism of action?
A. * Block of cholinergic receptors of postsynaptic membrane
B. Disturbance of acetylcholine synthesis
C. Block of Ca2+-channels of presynaptic membrane
D. Disturbance of cholinesterase synthesis
E. Disturbance of acetylcholine secretion
194. A 30 year-old patient suffering from pulmonary tuberculosis has been prescribed isoniazid.
Continuous taking of this drug may lead to the deficiency of the following vitamin:
A. * Pyridoxine
B. Tocopherol
C. Cobalamin
D. Ergocalciferol
E. Retinol
195. It is known that infectious type B hepatitis is a systemic disease caused by the type B hepatitis virus
and characterized by a predominant liver affection. Choose from the below given list the drugs for
the etiotropic therapy of this infection:
A. * Acyclovir
B. Penicillin
C. Tetracyclines
D. Sulfanilamides
E. Fluoroquinolones
196. In course of an experiment a dog has been injected a preparation that reduces secretory and motor
activity of stomach. What preparation is it?
A. * Atropine
B. Histamine
C. Secretin
D. Acetylcholine
E. Gastrin
197. In order to prevent adipose degeneration of liver after the viral hepatitis a patient should be
administered lipotropins. Name one of them:
A. * Choline
B. Tryptophan
C. Allopurinol
D. Contrykal
E. Vicasol
198. To relax skeletal muscles during complex surgeries, curare like substances are applied. These
substances block the following structure:
A. * Neuromuscular synapses
B. Basal ganglions
C. Red nuclei of the mesencephalon
D. Synaptic structures of the spinal cord
E. Vegetative ganglions
199. An elderly woman complains of twilight vision impairment. Which of the following vitamins should
be administered in this case?
A. * A
B. C
C. E

D. D
E. PP
200. A drugstore received a supply of a drug that is widely used for treatment of many virus diseases since
it is not virus specific. What drug is it?
A. Remantadin
B. Metisazone
C. Immunoglobulin
D. Vaccine
E. * Interferon
201. A patient was administered an antibiotic of animal origin for the corneal ulcer treatment. What is it
called?
A. * Lysozyme
B. Chlorophyllipt
C. Nystatin
D. Imanin
E. Gramicidin
202. Nonsteroid anti-inflammatory drugs are used in medical practice for treating the rheumatoid arthritis,
osteoporosis, inflammatory diseases of the connective tissue. These preparations inhibit the activity
of the following enzyme:
A. Hexokinase
B. Succinate dehydrogenase
C. Aminotransferase
D. * Cyclooxygenase
E. Xanthine oxidase
203. A man got an injection of curare like substance causing the relaxation of all skeletal muscles. What is
its mechanism of action?
A. Disturbance of acetylcholine synthesis
B. Block of Ca2+-channels of presynaptic membrane
C. * Block of cholinergic receptors of postsynaptic membrane
D. Disturbance of cholinesterase synthesis
E. Disturbance of acetylcholine secretion
204. Cardiac diseases are treated with cocarboxylase preparation. This preparation is the coenzymatic
form of the following vitamin:
A. * B1
B. B6
C. B12
D. C
E. P
205. Antibiotics can be classified according to various principles. According to the action mechanism
cephalosporins relate to the following group:
A. Inhibitors of protein synthesis
B. Inhibitors of respiratory processes
C. * Inhibitors of cell wall synthesis
D. Inhibitors of oxidative phosphorilation
E. Inhibitors of cytoplasmic membrane synthesis
206. A patient with low immunity, frequent colds is recommended to take ascoruti-ne as a more effective
drug than ascorbic acid. What constituent substance of this preparation intensifies action of vitamin
C?
A. Vitamin A
B. * Vitamin P
C. Glucose
D. Lactose
E. Vitamin D
207. A woman in labour has been given a drug that activates contractions of the smooth muscles of uterus.
Which hormone is a part of this drug?
A. Gastrin
B. Secretin
C. Angiotensin
D. * Oxytocin
E. Bradykinin
208. A patient fell ill the day before, the disease is acute with a predominance of general toxic symptoms.
With an account for the epidemic situation in the city, the doctor diagnosed the patient with influenza
A. What emergency etiotropic treatment must be administered to this patient?
A. Oxolinic ointment
B. Gentamicin
C. * Rimantadine
D. Inactivated influenza vaccine
E. Human gamma globulin
209. A 3,5-year-old child has been diagnosed with dysbacteriosis in form of critical reduction of
gram-positive anaerobic bacteria and increased number of staphylococci and yeast fungi. Which
preparation should be used for the correction of dysbacteriosis?
A. Colibacterin
B. Coli-Proteus bacteriophage
C. * Bifidumbacterin
D. Furazolidone
E. Lactoglobulin
210. Drugs that block certain channels can prevent the transmission of excitation from presynaptic
membrane to the postsynaptic membrane of synapse. What channels are blocked?
A. * Calcium
B. Sodium
C. Potassium potential-dependent
D. Potassium ATP-dependent
E. Chlorine
211. Depressive states can be treated by means of drugs inhibiting the enzyme that inactivates biogenic
amines. Specify this enzyme:
A. * MAO (monoamine oxidase)
B. LDH (lactate dehydrogenase)
C. CPK (creatine phosphokinase)
D. AST (aspartate aminotransferase)
E. ALT (alanine aminotransferase)
212. Sulfonamides are widely used as bacteriostatic agents. The mechanism of antimicrobial action of
sulfonamides is based on their structural similarity to:
A. * Para-aminobenzoic acid
B. Glutamic acid
C. Folic acid
D. Nucleic acid
E. Antibiotics
213. Bacteria eventually become resistant to antibacterial agents. Resistance of gram-positive bacteria to
penicillin antibiotics is caused by:
A. Permeability of the cell wall
B. Active synthesis of peptidoglycan
C. Active transport of antibiotic
D. * Beta-lactamase production
E. Protein synthesis
214. A patient with tuberculosis has been prescribed some anti-TB preparations. Which of the following
chemotherapeutic drugs has an effect on the tuberculosis pathogen?
A. Furacilinum
B. Methisazonum
C. * Ftivazide
D. Sulfadimezinum
E. Phthalylsulfathiazole
215. A 45-year-old patient with a gastric ulcer needs the reduction of HCl secretion. Which drug provides
this effect due to inhibition of the proton pump?
A. Atropine
B. Quamatel
C. Benzohexonium
D. Proglumide
E. * Omeprazole
216. Antidepressants can increase the concentration of catecholamine in the synaptic cleft. What is the
mechanism of action of these drugs?
A. Activation of monoamine oxidase
B. Inhibition of xanthine oxidase
C. * Inhibition of monoamine oxidase
D. Activation of acetylcholinesterase
E. Inhibition of acetylcholinesterase
217. On the 4th day of treatment with indomethacin a male 55-year-old patient developed gastric bleeding
due to the ulceration of the gastric mucosa. Ulcerogenic effect of the drug is associated with a
decrease in the activity of the following enzyme:
A. * Cyclooxygenase-1
B. Cyclooxygenase-2
C. Lipoxygenase
D. Thromboxane synthetase
E. Prostacyclin synthase
218. A patient with hypertensive crisis has been given an intravenous injection of clonidine. What
mechanism underlies the antihypertensive effect of clonidine?
A. Blockade of peripheral alpha1-adrenoceptors
B. Blockade of beta-adrenoceptors
C. * Stimulation of presynaptic central alpha2-adrenoceptors
D. Blockade of N-cholinergic receptors
E. Direct myotropic effect on blood vessels
219. A patient with hypertensive crisis should be administered a diuretic as a part of complex therapy.
What drug should be given the patient?
A. Diacarb
B. * Furosemide

C. Spironolactone
D. Triamterene
E. Amiloride
220. A patient with hypertension has been prescribed a drug that blocks angiotensin receptors. Specify this
drug:
A. Nifedipine
B. Prazosin
C. Captopril
D. * Losartan
E. Apressin
221. A patient with gastric ulcer has been administered omeprazole. What is the mechanism of its action?
A. Blockade of histamine H2-receptors
B. * Inhibition of H+K+-ATPase
C. M-cholinergic receptor blockade
D. Neutralization of HCl
E. Stimulation of mucus production
222. A patient with a heart rhythm disorder has been given lidocaine. Apart from the local anesthetic
effect, this drug has the following pharmacological effect:
A. Hypnotic
B. * Antiarrhythmic
C. Antipyretic
D. Antidepressant
E. Nootropic
223. A patient with chronic constipation has been prescribed bisacodyl. After 3 weeks of treatment, the
patient noticed a reduction of laxative effect. This is caused by the development of the following
side-effect:
A. Dependence
B. * Habituation (Tolerance)
C. Sensibilization
D. Cumulation
E. Dysbacteriosis
224. A patient with acute heart failure and cardiac glycosides intolerance was given an injection of
dobutamine. What is the mechanism of its action?
A. Stimulation of alpha1-adrenoceptors
B. Blockade of K+-, N a+-ATPase
C. * Stimulation of beta1-adrenoceptors
D. Inhibition of phosphodiesterase activity
E. Stimulation of M-cholinergic receptors
225. A female patient asked a pharmacist to recommend her a drug for headache with antiplatelet effect.
Specify this drug:
A. * Acetylsalicylic acid
B. Codeine phosphate
C. Promedol
D. Tramadol
E. Fentanyl
226. What are the indications for the use of naloxone?
A. Heavy metal intoxication

B. Intoxication with cardiac glycosides


C. * Acute intoxication with narcotic analgesics
D. Intoxication with ergot alkaloids
E. Atropine sulfate intoxication
227. A patient with renal colic has been administered a spasmolytic from the group of M-cholinergic
antagonists as a part of the complex therapy. Specify this drug:
A. * Atropine
B. Proserin
C. Galantamine
D. Dithylinum
E. Benzohexonium
228. A patient has been taking diclofenac sodium for a long time. The family physician withdraw this drug
and administered celecoxib. What disease was the reason for the drug substitution?
A. Bronchial asthma
B. Urolithiasis
C. Arterial hypertension
D. * Peptic ulcer
E. Chronic hepatitis
229. A patient has been diagnosed with bronchial asthma. Specify a medicament that can be administered
for asphyxiation:
A. Diclofenac sodium
B. Paracetamol
C. * Salbutamol
D. Anapriline
E. Acetylcysteine
230. Sulfanilamides inhibit the growth and development of bacteria. The mechanism of their action is
based on the impairment of the following acid synthesis:
A. * Folic
B. Lipoic
C. Nicotinic
D. Pantothenic
E. Pangamic
231. Sulfanilamides are widely used as bacteriostatic agents. The mechanism of antimicrobial action of
sulfanilamide is based on their structural similarity to:
A. Glutamic acid
B. Folic acid
C. Nucleic acid
D. * Para-aminobenzoic acid
E. Antibiotics
232. A patient with symptoms of chronic bronchitis has been administered acetylcysteine. What is the
mechanism of its expectorant action?
A. Stimulation of adrenergic receptors
B. Inhibition of cough center
C. Stimulation of respiratory center
D. Anesthesia of respiratory mucosa
E. * Depolymerization of sputum mucopolysaccharides
233. A 40-year-old patient has a history of bronchial asthma and bradyarrhythmia. In order to eliminate
bronchospasm, the drugs of the following pharmacological group should be administered

A. * M-anticholinergics
B. Beta-adrenergic blocking agents
C. M-cholinergic agents
D. Anticholinesterase agents
E. Muscle relaxants
234. Before a surgical operation, a surgeon treated his hands with an alcohol-containing solution. Which
group of drugs does this solution relate to?
A. Disinfectants
B. Sterilizing solutions
C. * Antiseptics
D. Detergents
E. Surface-active substances
235. The anti-tumor preparation Methotrexate is a structural analogue of folic acid. The mechanism of its
action is based on the inhibition of the following enzyme:
A. * Dihydrofolate reductase
B. Xanthine oxidase
C. Hexokinase
D. Creatine kinase
E. Lactate dehydrogenase
236. A patient with tuberculosis has been prescribed some anti-TB preparations. Which of the following
chemotherapeutic drugs has an effect on the tuberculosis pathogen?
A. Furacilinum
B. Methisazonum
C. * Ftivazide
D. Sulfadimezinum
E. Phtalazolum
237. P.Ehrlich is considered to be the founder of modern chemotherapy. What chemotherapy drug was
developed by this scientist?
A. Solusurminum
B. * Salvarsan
C. Calomel
D. Novarsenolum
E. Osarsolum
238. Antidepressants can increase the concentration of catecholamines in the synaptic cleft. What is the
mechanism of action of these drugs?
A. Activation of monoamine oxidase
B. Inhibition of xanthine oxidase
C. * Inhibition of monoamine oxidase
D. Activation of acetylcholinesterase
E. Inhibition of acetylcholinesterase
239. A 40-year-old female farmworker has been diagnosed with brucellosis and administered causal
chemotherapy. What group of drugs will be used for this purpose?
A. Donor immunoglobulin
B. * Antibiotic
C. Inactivated therapeutic vaccine
D. Polyvalent bacteriophage
E. Antitoxic serum

240. A parturient woman diagnosed with uterine inertia has been delivered to the maternity ward. The
doctor gave her an injection of the drug that activates the contraction of smooth muscles of the
uterus. What hormone is a component of this drug?
A. Gastrin
B. Secretin
C. * Oxytocin
D. Angiotensin
E. Bradykinin
241. A patient with chronic constipation had been prescribed bisacodyl. After 3 weeks of treatment, the
patient noticed a reduction of laxative effect. This is caused by the development of the following
side-effect:
A. Dependence
B. Sensibilization
C. Cumulation
D. * Tolerance
E. Dysbacteriosis
242. A patient has been taking diclofenac sodium for a long time. A family physician withdrew this drug
and prescribed celecoxib. What disease was the cause of drug substitution?
A. Bronchial asthma
B. Urolithiasis
C. Arterial hypertension
D. * Peptic ulcer
E. Chronic hepatitis
243. Diet of a human must contain vitamins. What vitamin is usually prescribed for treatment and
prevention of pellagra?
A. Vitamin C
B. Vitamin A
C. * Vitamin PP
D. Vitamin B1
E. Vitamin D
244. A 3,5-year-old child has been diagnosed with dysbacteriosis in the form of critical reduction of
gram-positive anaerobic bacteria and increased number of staphylococci and yeast fungi. What
preparation should be used for the correction of dysbacteriosis?
A. * Bifidumbacterin
B. Colibacterin
C. Coli-Proteus bacteriophage
D. Furazolidone
E. Lactoglobulin
245. Pharmacy has received virucides. Choose the virucide used for influenza treatment from the list given
below:
A. Metisazone
B. Levamisole
C. Azidothimidine
D. * Rimantadine
E. Acyclovir
246. A patient with hyperproduction of thyroid hormones has been prescribed Merkazolilum. This drug
inhibits the following enzyme of iodothyronine synthesis:
A. Aromatase

B. * Iodide peroxidase
C. Reductase
D. Decarboxylase
E. Aminotransferase
247. Neuroleptanalgesia has been applied in the case of cardiac infarction. What neuroleptic is most often
applied along with fentanyl?
A. Perphenazine (Aethaperazinum)
B. * Droperidol
C. Levomepromazine
D. Clozapine
E. Sulpiride
248. Due to prolonged taking of phenobarbital the epileptic patient has developed tolerance for this drug.
What is this phenomenon based on?
A. Absorption process weakening
B. Increase of receptor sensitivity
C. * Biotransformation acceleration
D. Biotransformation suppression
E. Substance accumulation in body
249. What side effect is characteristic of captopril?
A. Increase of arterial pressure
B. Hyperglycemia
C. Cardiac rate disorder
D. * Dry cough
E. Hypokaliemia
250. During treatment of chronic cardiac failure with digitoxin a patient developed the drug-specific signs
of intoxication. A doctor prescribed Unithiol (Dimercaptopropansulfonate sodium). Explain its
mechanism of action of Unithiol in case of cardiac glycoside intoxication:
A. * Restoration of -Na+-adenosine triphosphatase activity
B. Binding of calcium ions
C. Increase of sodium concentration in cardiac hystiocytes
D. Increase of calcium permeability of cardiac hystiocytes
E. Binding of glycosides into complex compound
251. An elderly patient suffers from constipation caused by colon hypotonia. What drug should be
prescribed?
A. Sodium sulfate
B. * Bisacodyl
C. Castor oil
D. Atropine sulfate
E. Novocainamide (Procainamide)
252. In the course of bronchitis pharmacotherapy a patient has developed dyspeptic disorders,
photodermatitis and hepatic failure. What drug can cause such disorders?
A. Paracetamol
B. Ascorbic acid
C. * Doxycycline
D. Acetylcysteine
E. Codeine phosphate

253. The patient with rheumatoid arthritis and concomitant duodenal ulcer has to be prescribed a
nonsteroid antiinflammatory drug. Which one of the drugs listed below is the drug of choice in the
given case?
A. Acetylsalicylic acid
B. * Celecoxib
C. Paracetamol
D. Metamizole
E. Diclofenac sodium
254. The patient with bronchial asthma had been prescribed salbutamol, which led to disappearance of
bronchospasm symptoms. It happened due to stimulation of:
A. * Beta2-adrenoreceptors
B. Alpha1-adrenoreceptors
C. Muscarinic acetylcholine receptors
D. Acetylcholine synthesis
E. Beta1-adrenoreceptors
255. A doctor has prescribed a nonsteroidal anti-inflammatory drug to relieve inflammation and pain
syndrome. Name this drug:
A. Glibenclamide
B. Loratadine
C. * Diclofenac sodium
D. Prednisolone
E. Calcium chloride
256. A 55-year-old patient had been diagnosed with angina pectoris. Calcium channel-blocking agent was
prescribed for treatment. Name this agent:
A. Atenolol
B. Guanethidine
C. Reserpine
D. * Amlodipine
E. Labetalol
257. A woman is to be prescribed a narcotic analgesic for labor pain relief. What drug is indicated in
this case?
A. Morphine
B. Papaveretum (Omnopon)
C. * Promedol (Trimeperidine)
D. Codeine
E. Fentanyl
258. A woman, who during the 5th-10th weeks of her pregnancy had been taking sodium valproate for
treatment of her epilepsy, gave birth to a child with pathology of the vertebral column (split spine).
What side effect of the drug caused such malformation?
A. * Teratogenic
B. Mutagenic
C. Embryotoxic
D. Fetotoxic
E. Sensitizing
259. A patient with acute bronchitis was prescribed an expectorant that caused bronchial spasm after the
patient had taken it. What drug of those listed below can cause such side effect?
A. Salbutamol
B. Validol (Menthyl isovalerate)

C. * Acetylcysteine
D. Platyphyllin
E. Prenoxdiazine (Libexin)
260. Prolonged application of broad spectrum antibacterial drugs resulted in the patient being hospitalised
with diagnosis of candidiasis. What side effect of antibiotic therapy has developed in the patient?
A. Endotoxic reaction
B. Toxic reaction
C. * Disbacteriosis
D. Allergic reaction
E. Formation of resistant microorganism strains
261. A 37-year-old patient with peptic gastric ulcer disease was prescribed a medicine as a part of his
multimodality therapy. The medicine lowers acidity of gastric juice, inhibits- H+, K+ adenosine
triphosphatase, decreases the volume of gastric secretion and pepsinogen production. It is a prodrug.
Name this medicine:
A. Famotidine
B. * Omeprazole
C. Gastrozepin (Pirenzepine)
D. Ranitidine
E. Phosphalugel (Aluminium phosphate)
262. A doctor prescribed a herbal drug with flavonoid complex of Silybum marianum to a patient
suffering from chronic hepatitis. This hepatic protector stimulates protein synthesis, normalizes
phospholipid metabolism, acts as an antioxidant. Name this drug:
A. * Silymarin
B. Essentiale
C. Galstena
D. Thiotriasoline
E. Ursodeoxycholic acid
263. Epinephrine is used to prolong the effect of novocaine during infiltrative anesthesia. What
epinephrine action can cause this effect?
A. * Vasoconstriction
B. Potentiation of novocaine action
C. Inhibition of the function of nerve endings and conductors
D. Vasodilatation
E. Suppression of tissue esterases
264. Bacteria become resistant to antibacterial agents. What is the resistance mechanism of gram-positive
bacteria’s to penicillin antibiotics treatment?
A. Cell wall permeability
B. * Beta-lactamases production
C. Active synthesis of peptidoglycane
D. Active transport of antibiotics
E. Protein synthesis
265. Pharmacy has received viricides. Choose the viricide used for influenza treatment from the list given
below.
A. Metisazone
B. Levamisole
C. Azidothimidine
D. * Rimantadine
E. Acyclovir

266. The patient has been prescribed a drug with antimicrobial action against mycobacterium tuberculosis.
What drug is used in the treatment of tuberculosis with antivitamin B6 activity?
A. Heparin
B. * Isoniazid
C. Trimethoprim/sulfamethoxazole (Co-trimoxazole)
D. Streptomycin
E. Sulfanilamide
267. Certain drugs can stimulate liver to synthesize enzyme systems taking part in drugs and toxines
metabolism. What agent induced synthesis of the microsomal drug-metabolizing enzyme system?
A. Heparin
B. * Phenobarbital
C. Menadione sodium bisulfite
D. Sulfanilamide
E. Aspirin
268. The 32-year-old patient has been taking antituberculosis drugs. Later he noticed that his urine had
become red-orange in color. Which of the following drugs is most likely responsible for this
phenomenon?
A. Isoniazid
B. Pyrazinamide
C. * Rifampicin
D. Ethambutol
E. Streptomycin sulphate
269. The 33-year-old female patient, who undergoes long-term treatment due to her chronic polyarthritis,
complains of increased arterial pressure, adipose tissue redistribution and menstrual irregularities.
Which of the following drugs is most likely responsible for these side effects?
A. Indometacin
B. * Prednisolone
C. Phenylbutazone
D. Acetaminophen
E. Diclofenac sodium
270. Due to prolonged use of phenobarbital in a patient with epilepsy developed tolerance to this drug.
What is the mechanism of the occurrence of this phenomenon?
A. * Acceleration of biotransformation
B. Impairment of absorption
C. Increase of receptor sensitivity
D. Suppression of biotransformation
E. Accumulation of substance
271. A patient with uncomplicated primary hypertension uses captopril as monotherapy. What is the most
common adverse effect of ACE inhibitors?
A. Increase of arterial pressure
B. * Dry cough
C. Hyperglycemia
D. Cardiac rate disorder
E. Hypokalemia
272. The patient with acute poisoning to enhance the rate of elimination of poison needs to start treatment
by forced diuresis. What drug can be used for this method for eliminating toxins?
A. Caffeine and sodium benzoate
B. * Furosemide

C. Galantamine hydrobromide
D. Enalapril
E. Piracetam
273. The patient with Parkinson disease has been prescribed a direct precursor of dopamine to relieve
muscular rigidity. Name this drug:
A. Aminazine
B. Paracetamol
C. * Levodopa
D. Scopolamine hydrobromide
E. Atropine sulphate
274. The patient with neurosis has been prescribed anxiolytic drug that is a benzodiazepine derivative.
Name this drug.
A. * Diazepam
B. Atropine sulphate
C. Piroxicam
D. Nandrolone
E. Trihexyphenidyl
275. The elderly patient suffers from constipation as a result of decreased bowel movements frequency.
Which of the following drugs should prescribe to relieve the symptoms?
A. * Bisacodyl
B. Sodium sulfate
C. Castor oil
D. Atropine sulphate
E. Procainamide
276. Inhibitors of one of the enzyme of amide metabolism are used in the treatment of depression. Which
of the following enzymes inhibition has such effect?
A. * Flavin adenine dinucleotide (FAD)-containing monoamine oxidase (MAO)
B. Acetylcholinesterase
C. Formylkynurenine (Arylformamidase)
D. Kynurenine 3-hydroxylase
E. Lactate dehydrogenase
277. Clonidine hydrochloride (Clopheline) lowers blood pressure by stimulating alpha-2 receptors in the
brain stem. It has a sympatholytic effect, suppresses release of norepinephrine. What is the potential
serious adverse reaction associated with this medication?
A. myelosuppression
B. anemia
C. irreversible nephrotoxicity
D. * withdrawal syndrome manifested by hypertensive crisis
E. systemic lupus erythematosus
278. Propranolol produces most of its important effects by antagonizing the beta-adrenergic effects of
catecholamines released in the stimulation of the sympathetic nerve or adrenal medulla. This leads to
all the following effects, EXCEPT:
A. negative inotropic effect on the heart
B. bronchoconstriction
C. lowering plasma renin activity
D. * strong diuretic effect
E. negative chronotropic effect on the heart

279. Antihypertensive agents must be used with caution. Find out medical condition for which propranolol
is most contraindicated:
A. renal insufficiency
B. coronary insufficiency
C. mental depression
D. liver disease
E. * asthma
280. Antihypertensive agents must be used with caution. The medical condition for which reserpine is
most contraindicated is:
A. renal insufficiency
B. coronary insufficiency
C. * mental depression
D. liver disease
E. asthma
281. Propranolol is a potent blocker of beta-adrenergic sympathetic activity. It has been found to be
detrimental for patients with:
A. * bronchial asthma
B. hypertension
C. hyperthyroidism
D. atrial fibrillation
E. ventricular arrhythmias
282. A patient with the clinical diagnosis of deep venous thrombosis has started heparin infusion of 1000
U/HR after initial intravenous bolus of 5000 units. Duration of action of heparin is:
A. * 4 hours
B. 10 hours
C. 12 hours
D. 24 hours
E. None of the above
283. Clonidine hydrochloride (Clopheline) lowers blood pressure by stimulating alpha-2 receptors in the
brain stem. It has a sympatholytic effect, suppresses release of norepinephrine. What is the potential
serious adverse reaction associated with this medication?
A. myelosuppression
B. anemia
C. irreversible nephrotoxicity
D. * withdrawal syndrome manifested by hypertensive crisis
E. systemic lupus erythematosus
284. Propranolol produces most of its important effects by antagonizing the beta-adrenergic effects of
catecholamines released in the stimulation of the sympathetic nerve or adrenal medulla. This leads to
all the following effects, EXCEPT:
A. negative inotropic effect on the heart
B. bronchoconstriction
C. lowering plasma renin activity
D. * strong diuretic effect
E. negative chronotropic effect on the heart
285. Antihypertensive agents are a class of drugs that are used to treat high blood pressure, with different
mechanism of action. The expected onset of action of nitroprusside is:
A. * seconds
B. 45 minutes

C. 15 to 30 minutes
D. several hours
E. 12 to 24 hours
286. Antihypertensive agents must be used with caution. Find out medical condition for which propranolol
is most contraindicated:
A. renal insufficiency
B. coronary insufficiency
C. mental depression
D. liver disease
E. * asthma
287. It is important to understand the mechanism of action of the various group of drugs used in the
treatment of hypertension. The antihypertensive action of propranolol involves all of the following
EXCEPT:
A. decreased stroke volume
B. inhibition of renin release
C. block adrenergic receptors
D. * depletion (decrease of syntesis) of catecholamines
E. decreased heart rate
288. A drug from group of cardiac glycosides is indicated in the management of congestive heart failure.
What is the mechanism of their positive ionotropic effect?
A. * Strengthening and shorten of systole
B. Shorten of diastole
C. Increase diastolic excitability
D. Decrease impulse generation
E. Weakening and lengthening of systole
289. A 67-year-old man presents to the emergency department with symptoms of acute heart failure.
Which of the following is the most appropriate immediate treatment for this patient?
A. * Strophanthine
B. Amiodarone
C. Adrenalin
D. Digitoxine
E. Digoxine
290. Patients with heart failure and edema presents to the emergency department. The physician decides to
prescribe strophantine. What is the mechanism of cardiotonic effect of cardiac glycosides?
A. * Blockade of Na/K ATPase
B. Stimulation of Na/K ATPase
C. Reflex influences on heart
D. Suppressed conduction of the myocardium
E. Indirect activation of adrenoreceptors
291. In patient, that suffers from chronic heart failure and takes cardiac glycosides every day, started to
complain of headache, fatigue, nausea, loss of appetite, diarrhea, anxiety, change in taste, confused
vision, objects appearing green and yellow. Sinus bradycardia occurred on an ECG and signs of
violation of atriovenrical conduction. Which of the following drugs is the most appropriate treatment
for reduction symptoms of intoxication?
A. * Unithiol
B. Naloxone
C. Bemegride
D. Dipiroxime

E. Atropine sulfate
292. Unithiol is prescribed to the patient with acute intoxication with cardiac glycosides. What is the
mechanism of action of the medicine in this situation?
A. * Remove the sulfhydryl groups deficiency
B. Increase concentration of Ca
C. Inhibit Na/K ATPase
D. Increase concentration of Na
E. Activate Na/K ATPase
293. After long-term therapy of Digitoxine in patient with chronic heart failure appeared such symptoms
nausea, vomiting, abdominal pain, headache, dizziness, confusion, delirium, vision disturbance,
irregular heartbeat. What should be used to restore the Na+-K+-ATPase activity?
A. * Unitiole
B. Dipiroxime
C. Naloxone
D. Atropine sulfate
E. Flumazenil
294. A patient with congestive heart failure to treat symptoms of atrial fibrillation uses digoxin. However,
after one week of repeated administration of this drug he starts to complain about arrhythmia, nausea,
anorexia, diarrhea, confused vision, objects appearing green and yellow. Which of the following
conditions most likely caused the appearance of these signs and symptoms in this patient?
A. * Material accumulation
B. Tolerance
C. Functional accumulation
D. Tachyphilaxis
E. Idiosyncrasy
295. The patient with congestive heart failure was admitted to the cardiology department. Digitoxine was
prescribed to him at the dose of 0.0001 g, but patient noted improvement and reduction of symptoms
only after a week of regular usage of a drug. The doctor explained the fact that this medication has
slow-onset of effect:
A. * Because of stable binding of digitoxine to plasma proteins
B. Because of an insufficient dose of digitoxine
C. Because of an insufficient absorption of the drug in the intestine
D. Because of increased diuresis
E. Because of very low-carbohydrate diet
296. Water-soluble cardiac glycoside was prescribed to the patient. Which concomitant disease can
contribute to accumulation of cardiac glycosides?
A. * Renal insufficiency
B. Hepatic insufficiency
C. Hyperacid gastritis
D. Hypoacid gastritis
E. High blood pressure
297. The patient with chronic heart failure complains of weakness, shortness of breath, swelling of the
lower extremities. Which of the following drugs would be the most efficacious in reducing
symptoms?
A. Adrenaline
B. * Digitoxine
C. Caffeine
D. Phenylephrine

E. Atropine
298. Patient with hypertensive disease and bradyarrhythmia. Which of the following drug is the most
appropriate treatment for this patient?
A. * Platiphyllline
B. Clophelin
C. Papaverine
D. Reserpine
E. Methyldopa
299. A 55-year-old man with a history of ischemic heart disease and hypertension presents to his
cardiologist, complains of chest pain and tachycardia Which of the following drugs would be most
appropriate for treating this patient?
A. * Propranolol
B. Sustac-forte
C. Potassium chloride
D. Strophantine
E. Lidocaine
300. It is necessary to performe partial hepatectomy for the patient. This operation can leads to severe
bleeding. The surgeon prescribed intravenously droply drug to prevent blood loss by decreasing
blood pressure. Which of the following drugs was most likely administered?
A. * Hygronium
B. Lobeline
C. Methacin
D. Pilocarpine
E. Anapriline
301. The patient presents to the emergency department with hypertensive crises. The physician decides to
prescribe intravenous drug which temporarily increased blood pressure and only then decreased.
Which of the following drugs was most likely administered?
A. * Clopheline
B. Hygronium
C. Magnesium sulfate
D. Dibazolum
E. Captopril
302. The 60 years old patient with diabetes mellitus for longer than 9 years, takes for correction of
hyperglycemia insulin. About 10 days ago his doctor has administered to him hypotensive agent to
treat arterial hypertension. Within an hour after last introduction of this drug hypoglycemic coma has
developed. Which of the following drugs could cause this complication?
A. * Propranolol
B. Prasosin
C. Verapamil
D. Captopril
E. Niphedepin
303. In treating patient with a hypertensive crisis was injected hypotensive drug, after using it developed
such side effects as tachycardia, dry mouth, dilatation pupils, orthostatic hypotension, urinary
retention. Which of the following drugs most likely caused the appearance of these signs and
symptoms in this patient?
A. * Benzohexonium
B. Clopheline
C. Magnesium sulfate

D. Dibazolum
E. Verapamil
304. Captopril is prescribed to the patient with hypertensive disease. Indicate the mechanism of the
hypotensive effect of this drug?
A. * Blockade of angiotensin converting enzyme activity
B. Blockade of beta-adrenoreceptors
C. Stimulation of beta2-adrenoceptors
D. Blockade of angiotensin 2 receptors
E. Stimulation of beta1 and beta2-adnenoceptors
305. Clopheline was injected intravenously to the patient with hypertensive crisis. Indicate the mechanism
of the hypotensive effect of this drug
A. * Stimulate central postsynaptic alpha-adrenoreceptors
B. Blockade of peripheral alpha- adrenoreceptors
C. Blockade of beta-adrenoreceptors
D. Blockade of N-cholinoreceptors of sympathic ganglions
E. Direct myotropic action on blood vessels
306. A patient who had been suffering from hypertensive disease was prescribed a drug from the
adrenergic group. However after using it blood pressure is normalized, but developed such side
effects as bradycardia (50 beats per minute) and second-degree atrioventricular block. Indicate the
drug which caused these complications.
A. * Anaprilinum
B. Prasosinum
C. Clophelinum
D. Mesatonum
E. Verapamilum
307. The 45 years old patient with the hypertensive disease has been taking antihypertensive drug during 4
days. The blood pressure normalized but patient noticed such symptoms as anxiety, drowsiness and
dry mouth. Which of the following drugs most likely caused these complications?
A. Prasosinum
B. * Clophelinum
C. Captopril
D. Enalapril
E. Hydralazine
308. The 56 years old woman with hypertensive disease presents to her physician complaining of
worsening her general condition. Methyldopa is prescribed to her. Indicate the mechanism of action
of this drug.
A. Stimulation of acetylcholine synthesis
B. Blockade of beta-adrenoreceptors
C. * Stimulation of alpha2-adrenoceptors
D. Stimulation of beta2-adrenoceptors
E. Blockade of angiotensin converting enzyme activity
309. The patient with hypertensive disease began to complain of heartburn, pain in the epigastrial area,
diarrhea. Which of the following drugs most likely caused these complications?
A. * Octadine
B. Pentamine
C. Propranolol
D. Captopril
E. Furosemid

310. A 45-year-old man presents to the emergency department with hypertensive crisis caused by
pheochromocytoma-related hypercatecholaminemia. Which of the following groups of drug is the
most appropriate treatment for this patient?
A. * Alpha-adrenoblockers
B. Beta-adrenoblockers
C. Gangliooblockers
D. Sympatolitics
E. Calcium channels blockers
311. The patient with ischemic heart disease to prevent heart attack received combined therapy. Which of
the following antianginal drugs can dilate coronary arteries and has antiplatelet effect?
A. * Dipiridamol
B. Acetylsalicylic acid
C. Validol
D. Nitroglycerine
E. Molsidomin
312. A patient is admitted to the emergency department with acute attack of angina pectoris. After
intravenous injection of drugs the patient noted such symptoms as severe headache and nausea.
Which of the following group of antianginal drugs is most likely responsible for this side effect?
A. * Organic nitrates
B. Calcium channels blockers
C. Beta-adrenoblockers
D. ACE-inhibitors
E. Beta-adrenostimulants
313. The patient with hypertensive disease is treated with hypotensive drug, arterial pressure has
decreased but the patient started complaining of dry cough. Which of the following drugs most likely
caused this complication?
A. propanolol
B. indomethacine
C. * captopril
D. nitroglycerin
E. acetaminophen
314. The patient with ischemic heart disease and chronic stable angina pectoris presents to his physician
complaining of high blood pressure. He prescribes drug that activates baroreceptors and stimulates
the sympathetic nervous system and can cause reflex tachycardia. Which of the following drugs is
most likely responsible for this side effect?
A. diltiazem
B. * nifedipine
C. clopheline
D. nitroglycerine
E. lidocaine
315. A 57-year-old patient with ischemic heart disease presents to the physician. On physical examination,
his blood pressure is 150/90 mm Hg and pulse is 90/min. Physician prescribes combine therapy
verapamil (Calan) and atenolol (Tenormin). It is important to consider which side effect is based on
the combination of these two drugs before use?
A. * Bradycardia
B. Muscle pain
C. Shortness of breath
D. Thrombocytopenia

E. Tachycardia
316. Beta blockers such as pindolol (Visken), penbutolol sulfate (Levatol), and acebutolol hydrochloride
(Sectral) differ from other beta blockers as they possess intrinsic sympathomimetic activity, which
means:
A. * increase heart rate
B. cause headache
C. shortness of breath
D. liver toxicity
E. decrease heart rate
317. To treat urinary retention physician prescribed agent that indirectly stimulates cholinergic receptors
in the smooth muscle of the urinary bladder and gastrointestinal tract, resulting in increased
peristalsis, causing hypersecretion of the gastric glands and common side effects include nausea,
crampy abdominal pain and diarrhea:
A. Atropine sulfate
B. Scopolamine
C. * Neostigmine
D. Hygronium
E. Platyphyllinum
318. The patient with hypertensive disease is treated with hypotensive drug, after a while began to
complain about a dry cough. Which of the following drugs most likely caused this complication?
A. Propanolol
B. Indomethacine
C. Nitroglycerine
D. * Enalapril
E. Acetaminophen
319. A patient who had been suffering from stenocardia accompanied by cardiac tachyarrhythmia is
treated with a drug, that has the following side effects: inhibition of the myocardium contraction,
bradycardia, hypotension, bronchospasm. Determine the agent:
A. Nitroglycerin
B. Dipyridamol
C. Verapamil
D. * Propranolol
E. Neostigmine
320. The patient suffered from rheumocarditis with frequent exacerbations of the disease. Which of the
following antibiotics should be used for prevention and treatment this disease?
A. Cloramfenicol
B. Erythromycin
C. * Bicillin-5
D. Cefazolin
E. Tetracycline
321. The patient used to treat infections antibiotic from the group of tetracycline. Doctor advised him not
to drink or eat milk or dairy products within one to two hours of taking tetracyclines. Why did he
recommend it?
A. The risk of superinfection increases
B. Milk products do not absorb
C. * The absorption of antibiotics decreases
D. The toxic of antibiotics increases
E. The process of digestion decreases

322. The patient with acute head injury, due to a breakdown of the blood-brain barrier appeared brain
edema. Which of the following drugs is the most appropriate treatment for this complication?
A. Spironolactone
B. Papaverini hydrochloride
C. * Furosemide
D. Dichlothiazide
E. Diazoline
323. A 42-year-old man was admitted to the hospital with acute pain in his big toe. Laboratory
examination reveals a high level of uric acid. After few days allopurinol was prescribed as a part of
combine therapy. What is the mechanism of action of allopurinol?
A. * Inhibits the activity of xanthine oxidase
B. Inhibits the activity of Na+-K+-ATPase
C. Inhibits the activity of cholinesterase
D. Inhibits the activity of MAO
E. Inhibits the activity of COMT
324. A 75-year-old man presents to the emergency department with symptoms of acute heart failure such
as shortness of breath, fatigue, weakness, edema, irregular heartbeat. Which of the following drugs is
the most appropriate treatment for this patient?
A. * Strophanthine
B. Amiodarone
C. Adrenaline
D. Dimedrole
E. Hydrochlorothiazide
325. A patient who had been suffering from congestive heart failure was treated with a drug from group of
cardiac glycoside. What is the mechanism of their positive ionotropic effect?
A. * Strengthening and shorten of systole
B. Shorten of diastole
C. Increase diastolic excitability
D. Decrease impulse generation
E. Weakening and lengthening of systole
326. A 41 -year-old woman presents to her physician complaining of weakness, weight loss, myalgia,
arthralgia, gingival hemorrhages, loss of teeth, alopecia, poorly healing wounds, painful bruises as a
result of dietary deficiency. Laboratory examination reveals an anemia. Which vitamin deficiency
most likely caused the appearance of these signs and symptoms in this patient?
A. * Ascorbic acid
B. Alpha-tocopherol acetate
C. Thiamine hydrochloride
D. Cyanolocobalamin
E. Riboflavin
327. A 69-year-old man with a history of hypertensive disease accompanied by chronic bronchitis presents
to his physician complaining of shortness of breath. On examination, his heart rate is 98/min and
respiratory rate is 20/min. Indicate the drug which is contraindicated for this patient:
A. * Propranolol
B. Captopril
C. Papaverine
D. Nifedipine
E. Hydrochlorothiazide

328. A 65-year-old man with a history of ischemic heart disease and hypertension presents to his
physician, complains of chest pain and tachycardia Which of the following drugs would be most
appropriate for treating this patient?
A. * Propranolol
B. Suprastine
C. Potassium chloride
D. Strophantine
E. Lidocaine
329. It is necessary to performe partial hepatectomy for the patient. This operation can leads to severe
bleeding. The surgeon prescribed intravenously droply drug to prevent blood loss by decreasing
blood pressure. Which of the following drugs was most likely administered?
A. * Hygronium
B. Aceclidinum
C. Pyracetam
D. Paracetamol
E. Anaprilinum
330. The patient presents to the emergency department with hypertensive crises. The physician decides to
prescribe intravenous drug which temporarily increased blood pressure and only then decreased.
Which of the following drugs was most likely administered?
A. * Clopheline
B. Sustac-forte
C. Magnesium sulfate
D. Dimedrole
E. Strophantine
331. Captopril is prescribed to the patient with hypertensive disease. Indicate the mechanism of the
hypotensive effect of this drug?
A. * Blockade of angiotensin converting enzyme activity
B. Blockade of beta-adrenoreceptors
C. Stimulation of beta2-adrenoceptors
D. Blockade of angiotensin 2 receptors
E. Stimulation of beta1 and beta2-adnenoceptors
332. The patient presents to the emergency department with hypertensive crises, after intravenous
injection of the drug, appeared such side effects as tachycardia and orthostatic hypotension. Which of
the following drugs most likely caused this complication?
A. Reserpine
B. * Clopheline (Clomidine)
C. Magnesium sulfate
D. Dibazol
E. Verapamil
333. Clopheline was injected intravenously to the patient with hypertensive crisis. What is the mechanism
of antihypertensive action of this drug?
A. * Stimulate central postsynaptic alpha-adrenoreceptors
B. Blockade of peripheral alpha- adrenoreceptors
C. Blockade of beta-adrenoreceptors
D. Blockade of N-cholinoreceptors of sympathic ganglions
E. Direct myotropic action on blood vessels

334. A patient who had been suffering from hypertensive disease was prescribed a drug from the
adrenergic group. However after using it blood pressure is normalized, but developed such side
effects as bradycardia (50 beats per minute) and second-degree atrioventricular block. Indicate the
drug which caused these complications.
A. * Propranolol
B. Papaverine
C. Clopheline
D. Phenylephrine
E. Verapamil
335. The 47 years old patient with the hypertensive disease has been taking antihypertensive drug. The
blood pressure normalized but patient noticed such symptoms as drowsiness, dizziness and dry
cough. Which of the following drugs most likely caused these complications?
A. Clopheline
B. Prazosin
C. * Captopril
D. Aspirine
E. Adrenaline
336. The 56 years old woman with hypertensive disease presents to her physician complaining of
worsening her general condition. Methyldopa is prescribed to her. Indicate the mechanism of action
of this drug.
A. Stimulation of acetylcholine synthesis
B. Blockade of beta-adrenoreceptors
C. * Stimulation of alpha2-adrenoceptors
D. Stimulation of beta2-adrenoceptors
E. Blockade of angiotensin converting enzyme activity
337. A 49-year-old man with diagnosis of pheochromocytoma presents to the emergency department
with high blood pressure, headache, excessive sweating and palpitations . Which of the following
groups of drug is the most appropriate treatment for this patient?
A. * Alpha-adrenoblockers
B. Beta-adrenoblockers
C. Gangliooblockers
D. Sympatholytics
E. Calcium channels blockers
338. The patient with acute head injury, due to a breakdown of the blood-brain barrier appeared brain
edema. Which of the following drugs is the most appropriate treatment for this complication?
A. Spironolactone
B. Papaverini hydrochloride
C. * Furosemide
D. Dichlothiazide
E. Diazoline
339. A 55 years old man who had been suffering from gout, was treated with aethamid in order to reduce
the level of uric acid in the blood. What is the mechanism of action of this drug?
A. * Inhibits reabsorption of uric acid
B. Increases secretion of uric acid
C. Reduces the production of uric acid
D. Inhibits xanthine oxidase activity
E. Produces easily soluble salts

340. A 66 years old patient with pulmonary edema caused by congestive heart failure, acute left
ventricular insufficiency was admitted to the hospital. Which of the following drugs is the most
appropriate treatment for this patient?
A. * Furosemide
B. Triamterene
C. Selegiline
D. Amlodipine
E. Aspirine
341. A 62 years old men with pulmonary edema caused by congestive heart failure, acute left ventricular
insufficiency was treated with diuretics. Indicate the diuretic which is contraindicated in this
situation?
A. Hypothiazide
B. Spironolactone
C. Furosemide
D. * Mannitol
E. Diacarb
342. A 42 years old man who had been suffering from gout, was treated with allopurinol in order to reduce
the level of uric acid. What is the mechanism of action of this drug?
A. * Competitive inhibition of xantinoxydase
B. Increased excretion of nitrogen substances
C. Acceleration of carbohydrates catabolism
D. Decreased the reuse of nucleotides
E. Increased synthesis of norepinephrine
343. A 62 years old patient with peripheral edemas was treated with dichlotiazide and furosemide but the
use of these drugs did not cause marked diuretic effect. Laboratory examination reveals a high level
of aldosterone in the blood. Indicate which medicine should be prescribed:
A. Mannitol
B. * Spironolactone
C. Ethacrynic acid
D. Furosemide
E. Diacarb
344. The 59 years old patient with the hypertensive disease has been taking complex antihypertensive
therapy including diuretics. The blood pressure normalized but the signs of hypokalemia appeared.
Which of the following drugs most likely caused these complications?
A. Triamterene
B. Clopheline
C. Enalapril
D. Spironolactone
E. * Furosemide
345. The 39 years old patient suffers from chronic hyperacid gastritis presents to her physician
complaining of abdominal pain, heartburn, particularly after meals and constipation. Which of the
following drugs should her physician prescribe to relieve her symptoms?
A. * Magnesium oxyde
B. Sodium hydrocarbonate
C. Atropine sulfate
D. Potassium chloride
E. Sodium sulphate

346. Choose the group of drug that decreases smooth muscle tone activity of gastrointestinal tract, reduces
secretion of the gastric glands and bile acids:
A. Ganglion-blockers
B. * M-cholinoblockers
C. Myotropic spasmolytics
D. Anticholinesterase drug
E. M-cholinomimetics
347. A 6-year-old boy with no previous medical history presents to physician. Physical examination
demonstrates numerous subcutaneous haemorrhages, gingivitis, dental caries. What combination of
vitamins should be prescribed in this case?
A. Folic acid and cyanocobalamin
B. Thiamine and pyridoxine
C. Riboflavin and nicotinamide
D. * Ascorbic acid and rutin
E. Calciferol and ascorbic acid
348. Patient undergoing surgery on the organs of abdominal cavity, in the early postoperative period was
developed paresis of the intestine. Which of the following is the most appropriate pharmacotherapy
for stimulation of GIT motility?
A. * Proserin
B. Aspirin
C. Nifedipine
D. Metoclopramide
E. Papaverine
349. A 66-year-old patient with acute myocardial infarction left main coronary artery thrombosis presents
to the emergency department. Which of the following group of drugs should be used to reestablish
blood flow?
A. Opioid analgesics
B. Angiotensin-converting enzyme inhibitors
C. Glucocorticoids
D. Beta-adrenergic blockers
E. * Fibrinolytic
350. A patient with essential hypertension takes enalapril. Indicate the mechanism of the hypotensive
effect of this drug?
A. Stimulation of beta1 and beta2-adnenoceptors
B. Blockade of beta-adrenoreceptors
C. Stimulation of beta2-adrenoceptors
D. Blockade of angiotensin 2 receptors
E. * Blockade of angiotensin converting enzyme activity
351. A woman with ischemic heart disease has been taking an antianginal medication that has the
following properties: dilates coronary arteries, peripheral vessels (arterial and venous), reduces the
need of myocardium in oxygen, improves coronary circulation. Name this drug:
A. * Nitroglycerine
B. Validole
C. Aminophylline
D. Papaverine
E. Dibasol

352. A patient who had been suffering from congestive heart failure was treated with digitoxin and
furocemide. She arrives at her physicians office complaining of weakness, fatigue, muscle cramps,
heart palpitations and constipation. What electrolyte imbalance may appear in her blood?
A. Hyperkalemia
B. * Hypokalemia
C. Hypocalcemia
D. Hypernatremia
E. Hypercalcemia
353. A 50 years old patient with ischemic heart disease was treated with antiplatelet drug. The patient
started complaining of nausea, vomiting, stomach pain, bleeding gums, cuts that don't stop bleeding.
Which of the following drugs most likely caused these complications?
A. Ticlide
B. * Acetylsalicylic acid
C. Paracetamol
D. Dipiridamol
E. Pentoxyphilline
354. After tooth extraction the blood pressure of the patient decreased rapidly, the patient lost
consciousness. Collaptoid state was diagnosed. Which of the following drug is the most appropriate
treatment for this patient?
A. Nitroglycerine
B. Isadrin
C. Sustac
D. Cordiamin
E. * Epinephrine
355. A patient has been taking glucocorticoids for a long time. The use of steroids was stopped abruptly
which caused such symptoms of withdrawal syndrome as acute attack of the main disease, low blood
pressure and weakness. Which of the following phenomena is most likely responsible for this side
effect?
A. Cumulation
B. Hyperproduction of corticotropin hormone
C. * Adrenal glands insufficiency
D. Sensibilisation
E. Drug abuse
356. Bile duct obstruction in patient caused the inhibition of coagulation, developed bleeding due to low
absorption of the vitamin. Which of the following vitamins deficiency is most likely responsible for
this side effect?
A. * Vitamin K
B. Vitamin D
C. Vitamin B
D. Vitamin A
E. Vitamin E
357. A 68 years old patient who had been suffering from congestive heart failure with peripheral edemas
was treated with dichlotiazide and furosemide but the use of these drugs did not cause marked
diuretic effect. Laboratory examination reveals a high level of aldosterone in the blood. Which of the
following drugs is the most appropriate treatment for this patient?
A. Mannit
B. Amilorid
C. Clopamid
D. Urea

E. * Spironolactone
358. A 48 years old patient has been taking glucocorticoids for a long time. The use of steroids was
stopped abruptly which caused such symptoms of withdrawal syndrome as acute attack of the main
disease, low blood pressure and weakness. Which of the following phenomena is most likely
responsible for this side effect?
A. Sensibilization
B. Habituation
C. Hyperproduction of ACTH
D. * Appearance of adrenal insufficiency
E. Cumulation
359. A 37-year-old man was admitted to the surgical department with the symptoms of acute pancreatitis:
vomiting, nausea, abdominal pain, diarrhea, bradycardia, hypotension, weakness, dehydration. What
medicine should be used first of all?
A. Ethaperazine
B. No-spa
C. Plathyphylline
D. Ephedrine
E. * Contrycal
360. A 36-year-old man presents to his family physician several days after acute attack of gout, the
physician decides to prescribe allopurinol. What pharmacological action of allopurinol provides
therapeutic effect?
A. By increasing uric acid excretion
B. By inhibiting leucocyte migration into the joint
C. * By inhibiting uric acid synthesis
D. By general analgetic effect
E. By general anti-inflammatory effect
361. A 50-year-old male farm worker has been admitted to the emergency department. He was found
confused in the orchard and since then has remained unconscious. On physical examination,his heart
rate is 45/min and blood pressure is 80/40 mm Hg. He is sweating and salivating profusely. Which of
the following drugs is the most appropriate treatment for this patient?
A. Physostigmine
B. * Atropine
C. Proserine
D. Pentamine
E. Norepinephrine
362. A 13-year-old girl with a childhoodhistory of asthma presents to the emergency department
complained of cough, shortness of breath, chest pain and wheezing. Physical examination revealed
diaphoresis, dyspnea, tachycardia and tachypnea. Her respiratory rate was 42/min, heart rate was
110/min, and blood pressure was 130/70 mm Hg. Which of the following is the most appropriate
drug to rapidly reverse her bronchoconstriction?
A. Inhaled fluticasone
B. * Inhaled salbutamol
C. Inhaled beclomethasone
D. Inhaled cromolyn sodium
E. Oral theophylline
363. A 41-year-old man was admitted to the surgical department with the symptoms of acute pancreatitis:
vomiting, nausea, abdominal pain, diarrhea, bradycardia, hypotension, weakness, dehydration. What
medicine should be used first of all?
A. Etaperazine

B. No-spa
C. Platyphylline
D. Ephedrine
E. * Contrycal
364. A 32 years old man who had been suffering from gout, presents to the physician. He decides to
prescribe allopurinol in order to reduce the level of uric acid:
A. By increasing uric acid excretion
B. By inhibiting leukocyte migration into the joint
C. * By inhibiting uric acid synthesis
D. By general analgetic effect
E. By general anti-inflammatory effect
365. A 62 years old woman with ischemic heart disease has been taking an antianginal medication that has
the following properties: dilates coronary arteries, peripheral vessels (arterial and venous), reduces
the need of myocardium in oxygen, improves coronary circulation. Name this drug:
A. * Nitroglycerine
B. Validole
C. Papaverine
D. Dibasol
E. Aminophylline
366. A 51 years old patient has been taking glucocorticoids for a long time. Drug withdrawal caused such
symptoms as acute attack of the main disease, low blood pressure and weakness. Which of the
following phenomena is most likely responsible for this side effect?
A. * Adrenal glands insufficiency
B. Drug habituation
C. Sensibilisation
D. Hyperproduction of adrecorticotropic hormone
E. Cumulation
367. A 26-year-old man with chronic, poorly controlled asthma presents to the emergency department
complained of cough, shortness of breath, chest pain and wheezing. Physical examination revealed
dyspnea, tachypnea respiratory rate was 30/min, tachycardia heart rate was 102/min, and blood
pressure was 140/90 mm Hg. The physician decides to prescribe salbutamol. What is the mechanism
of action of this drug?
A. * Stimulates beta2-adrenoreceptors
B. Stimulates alpha-adrenoreceptors
C. Blocks H1-histamine receptors
D. Blocks phosphodiesterase
E. Blocks M-cholinergic receptors
368. A 69-year-old patient with acute myocardial infarction due to left main coronary artery thrombosis
presents to the emergency department. Which of the following group of drugs should be used to
reestablish blood flow?
A. * Fibrinolysis activators
B. Narcotic analgesics
C. Beta-adrenergic blockers
D. Angiotensin-converting enzyme inhibitors
E. Glucocorticoids
369. A 42-year-old male patient with generalized severe chronic periodontitis presents to physician. He
decides to prescribe a topical corticosteroid therapy in ointment. Which of the following drug should
be used in this situation?

A. * Triamcinolone
B. Tetracycline
C. Decamine
D. Ampicillin
E. Erythromycin
370. A 62-year-old patient with congestive heart failure was treated with digitoxin and furocemide
complains of weakness, fatigue, muscle cramps, heart palpitations and constipation. What electrolyte
imbalance may appear in his blood?
A. * Hypokalemia
B. Hyperkalemia
C. Hypocalcemia
D. Hypernatremia
E. Hypercalcemia
371. What is nessesary to do after tooth extraction to limit bleeding while clotting takes place?
A. * Epinephrine locally
B. Thrombin injection
C. Fibrinogen injection
D. Vikasol orally
E. Neodicumarine orally
372. After tooth extraction the blood pressure of the patient decreased rapidly, and he lost consciousness.
Collaptoid state was diagnosed. Which of the following drug is the most appropriate treatment for
this patient?
A. * Epinephrine
B. Nitroglycerine
C. Isadrin
D. Sustac
E. Cordiamin
373. A 66 years old man with ischemic heart disease accompanied by cardiac arrhythmia has been taking
an antianginal medication. An electrocardiogram shows lengthening of P-Q interval. Which of the
following drugs most likely caused these complications?
A. * Atenolol
B. Prazosin
C. Atropine
D. Lidocaine
E. Plathyphyllin
374. A 56 years old patient who had been suffering from ischemic heart disease, was treated with
antiplatelet drug. The patient started complaining of nausea, vomiting, stomach pain, bleeding gums,
cuts that don't stop bleeding. Which of the following drugs most likely caused these complications?
A. * Acetylsalicylic acid
B. Parmidine
C. Ticlide
D. Dipiridamol
E. Pentoxyphilline
375. A patient with Quincke's edema after being stung by a bee has been admitted to the emergency
department. What antihistamine without sedation should be prescribed to the patient?
A. * Diasoline
B. Suprastine
C. Phencarol

D. Tavegil
E. Diphenhydramine hydrochloride
376. A 35 years old patient with hyperthyroidism has been taking a drug that inhibits the enzyme
thyroperoxidase, and the synthesis of thyroid hormones. Indicate this drug:
A. * Mercazolil
B. Diiodotyrosine
C. Potassium iodide
D. Levothyroxine
E. Radioactive iodine
377. A 45-year-old man with chronic alcoholism was admitted to the hospital with symptoms of
peripheral neuropathy such as muscle weakness, numbness and pain, in his hands and feet. Which
vitamin preparation should be prescribed for the treatment of this patient?
A. * Thiamine
B. Ergocalciferol
C. Retinol
D. Rutin
E. Phylloquinone
378. A patient has a slowly healing fracture. Which of the following drugs stimulates the process of
regeneration and promotes the activation of metabolic processes in tissues?
A. * Methyluracil
B. Prednisolone
C. Cyclophosphan
D. Methotrexate
E. Cyclosporine
379. A patient with atopic dermatitis to reduce inflammation and to decrease skin irritation applied the
topical glucocorticoid. Which of the following drugs has the minimal resorptive effect?
A. * Flumethasoni pivalas
B. Prednisolone
C. Hydrocortisone
D. Triamcinolone
E. Dexamethasone
380. A patient with ischemic heart disease chest pain, shortness of breath and palpitations, physician
decides to prescribe verapamil. What is the mechanism of its action?
A. * It blocks calcium channels
B. It blocks alpha-adrenoreceptors
C. It blocks beta-adrenoreceptors
D. It blocks potassium channels
E. It blocks sodium channels
381. A student with seasonal allergic rhinitis complains of rhinorrhoea, sneezing and nasal itching. Which
of the following drugs is the most effective in relieving these symptoms?
A. * Loratadine
B. Noradrenaline hydrotartate
C. Propranolol
D. Ambroxol
E. Losartan
382. A 70-year-old patient suffering from congestive heart failure has been uncontrolled taking digoxin
complains of anorexia, nausea, vomiting, fatigue, headache, weakness, arrhythmia, blurred vision and
diarrhea. Which of the following group of drugs will be pathogenetically reasonable in this case:
A. * Donators of sulfhydric groups
B. Stimulants of beta1-adrenoreceptors
C. Angiotensin II receptor blockers
D. Potassium channel blockers
E. Donators of nitrogen oxide
383. A 42-year-old man with rheumatoid arthritis was treated with COX-2 selective nonsteroidal
anti-inflammatory drug which reduces the risk of gastrointestinal ulcers and ulcer complications.
Indicate this drug:
A. * Celecoxib
B. Analgine
C. Acetylsalicinic acid
D. Butadion
E. Ibuprofen
384. А patient who has been suffering from chronic bronchitis, with excessive mucus production is treated
with a synthetic mucolytic drug that decreases viscosity of the sputum. Indicate this drug:
A. * Acetylcysteine
B. Diazepam
C. Heparin
D. Furosemide
E. Enalapril
385. A 62-year-old woman with type 2 diabetes mellitus, metabolic syndrome and atherosclerosis,
associated with insulin resistance. Which of the following drug is the most appropriate treatment for
this patient?
A. * Glibenclamid
B. Insulin
C. Retabolil
D. Lovastatin
E. Amlodipine
386. A 39-year-old patient with a history of chronic gastroduodenitis presents to physician complaining of
morning stiffness, joint pain and swelling. Which of the following nonsteroidal anti-inflammatory
drugs is the most appropriate treatment for this patient?
A. * Celecoxib
B. Diclofenac sodium
C. Ibuprofen
D. Acetylsalicylic acid
E. Indomethacin
387. A 69-year-old patient with sudden worsening of the symptoms of heart failure, such as dyspnea, chest
pain, palpitations, feet swelling and fatigue presents to the emergency department. Which of the
following agents would be the most efficacious in reducing his symptoms?
A. * Corglycon
B. Digitoxin
C. Cordiamine
D. Nitroglycerine
E. Adrenalin hydrochloride
388. A 41 -year-old woman presents to her physician complaining of sudden attacks of severe sharp
shooting facial pain that last from a few seconds to about two minutes. Trigeminal neuralgia was
diagnosed. The physician decides to prescribe nonsteroidal anti-inflammatory drug. Which of the
following drugs is the most appropriate treatment for this patient?
A. * Diclofenac sodium
B. Codeine phosphate
C. Ketamine hydrochloride
D. Lidocaine hydrochloride
E. Droperidol
389. A patient was prescribed with bile preparation for the purpose of improvement of rich food digestion.
What components of this preparation take part in fat emulsification?
A. * Bile acids
B. Cholesterol and its ethers
C. Diglyceride
D. Bilirubin-glucuronids
E. Higher fatty acids
390. Antivitamins are substances of various structure that limit utilization of vitamins in an organism and
have an opposite to them action. Name antivitamin of vitamin K:
A. Sulfapyridasine
B. * Dicumarol
C. Deoxypyridoxine
D. Aminopterin
E. Isoniazid
391. A patient with low immunity, frequent colds is recommended to take ascorutine as a more effective
drug than ascorbic acid. What constituent substance of this preparation intensifies action of vitamin
C?
A. Vitamin A
B. Glucose
C. Lactose
D. * Vitamin P
E. Vitamin D
392. In order to prevent adipose liver degeneration after viral hepatitis it is necessary to prescribe the
patient lipotropic factors. Name one of them:
A. Tryptophane
B. * Choline
C. Allopurinol
D. Contrical
E. Vicasol
393. Antibiotics are classified by sources of production. Name an antibiotic of bacterial origin:
A. Penicillin
B. Tetracycline
C. Lysozyme
D. * Gramicidin
E. Gentamycin
394. A woman in labour was given a preparation that activates contractions of smooth uterine muscles.
What hormone is contained in this preparation?
A. * Oxytocin
B. Gastrin
C. Secretin
D. Angiotensin
E. Bradykinin

395. Patients ill with tuberculosis take a drug that is an antivitamin of nicotinic acid. Name this substance:
A. Sulfanilamide
B. * Isoniazid
C. Acrichine
D. Isoriboflavin
E. Oxythiamine
396. A patient was prescribed with an antitumoral antibiotic that inhibits synthesis of nucleic acids in the
cells. What of the following antibiotics has such a mechanism of action?
A. Tetracycline
B. Nystatin
C. * Actinomycin
D. Lincomycin
E. Erythromycin
397. Depressive states can be treated by means of drugs inhibiting the enzyme that inactivates biogenic
amines. Specify this enzyme:
A. LDH (lactate dehydrogenase)
B. CPK (creatine phosphokinase)
C. * MAO (monoamine oxidase)
D. AST (aspartate aminotransferase)
E. ALT (alanine aminotransferase)
398. A parturient woman diagnosed with uterine inertia has been delivered to the maternity ward. The
doctor gave her an injection of the drug that activates the contraction of smooth muscles of the
uterus. What hormone is a component of this drug?
A. Gastrin
B. Secretin
C. Angiotensin
D. * Oxytocin
E. Bradykinin
399. The elderly patient suffers from constipation caused by large intestine hypotonia. What drug should
be prescribed?
A. Sodium sulfate
B. * Bisacodyl
C. Castor oil
D. Atropine sulphate
E. Procainamide
400. Inhibitors of one of the amides metabolism enzymes are used to treat depression. What enzyme
inhibition has such an effect?
A. * Flavin adenine dinucleotide (FAD)-containing monoamine oxidase (MAO)
B. Acetylcholinesterase
C. Formylkynureninase (Arylformamidase)
D. Kynurenine 3-hydroxylase
E. Lactate dehydrogenase
Назва наукового напрямку (модуля): Семестр: 6
Pharmacology KROK 1 2018-2019
Опис:
Факультет іноземних студентів (медицина), 3 курс
Перелік питань:
1. Despite the administration of cardiotonics and a thiazide diuretic a patient with chronic heart failure
has persistent edemata, there is a risk of ascites. What medication should be administered in order to
enhance the diuretic effect of the drugs used?
A. Furosemide
B. * Spironolactone
C. Amiloride
D. Clopamide
E. Manithol
2. A 66-year-old patient with Parkinson’s disease shows an improvement in locomotor activity after
prolonged use of a certain drug which is converted to dopamine by the decarboxylation. What drug
has the patient taken?
A. Naloxone
B. Celecoxib
C. * Levodopa
D. Droperidol
E. Chlorpromazine
3. A 46-year-old female is scheduled for a maxillofacial surgery. It is known that the patient is prone to
high blood coagulation. What natural anticoagulant can be used to prevent blood clotting?
A. Trombine
B. * Heparin
C. Sodium citrate
D. Fibrinolysin
E. None of the above-listed substances
4. A patient has been administered an anti-inflammatory drug that blocks the action of cyclooxygenase.
Specify this anti-inflammatory agent:
A. Metamirde sodium
B. Allopurinol
C. * Aspirin
D. Thiamin
E. Creatine
5. A 26-year-old female patient with bronchitis has been administered a broad spectrum antibiotic as a
causal treatment drug. Specify this drug:
A. * Doxycycline
B. Interferon
C. BCG vaccine
D. Ambroxol
E. Dexamethasone
6. A 12-year-old child has a viral infection complicated by obstructive bronchitis. Bronchospasm can be
eliminated by inhalations of a drug from the following pharmacological group:
A. M-anticholinergics
B. N-cholinomimetics
C. * Beta-2-agonists
D. Beta-adrenergic blockers
E. Analeptics
7. A 26-year-old woman at 40 weeks pregnant has been delivered to the maternity ward. Objectively:
the uterine cervix is opened, but the contractions are absent. The doctor has administered her a
hormonal drug to stimulate the labor. Name this drug:
A. * Oxytocin
B. Hydrocortisone
C. Estrone
D. Testosterone
E. ACTH
8. A patient has recurrent attacks of epileptic seizures and stays unconscious between them. In order to
stop convulsions the drugs of the following group should be used in the first place:
A. * Tranquilizers
B. Neuroleptics
C. Muscle relaxants
D. Sedatives
E. Analeptics
9. A patient with arthritis and varicose veins has been taking a non-steroidal anti-inflammatory drug for
a long time, which caused thrombosis of skin veins. Which of the following drugs might have caused
this complication?
A. Indomethacin
B. Aspirin
C. Phenylbutazone
D. * Celecoxib
E. Ibuprofen
10. A patient has arterial hypertension. What long-acting drug from the group of calcium channel
blockers should be prescribed?
A. * Amlodipine
B. Octadine
C. Pyrroxanum
D. Atenolol
E. Reserpine
11. A 65-year-old female patient has chronic constipations due to the colon hypotonia. What drug should
be chosen in this case?
A. * Bisacodyl
B. Sodium sulfate
C. Magnesium sulfate
D. Neostigmine methylsulfate
E. Metoclopramide
12. Administration of doxycycline hydrochloride caused an imbalance of the symbiotic intestinal
microflora. Specify the kind of imbalance caused by the antibiotic therapy:
A. * Dysbacteriosis
B. Sensibilization
C. Idiosyncrasy
D. Superimposed infection
E. Bacteriosis
13. Ascarid eggs have been detected during stool analysis. What drug should be prescribed?
A. Nystatin
B. Chloramphenicol
C. * Mebendazole

D. Tetracycline
E. Furazolidone
14. A patient with chronic heart failure with edema has increased level of blood aldosterone. What
diuretic would be most effective in this case?
A. Triamterene
B. Acetazolamide
C. Hydrochlorothiazide
D. Furosemide
E. * Spironolactone
15. What drug will be most appropriate for the patient who has chronic gastritis with increased secretion?

A. Pancreatine
B. * Pirenzepine
C. Pepsin
D. Aprotinin
E. Chlorphentermine
16. A child was born asphyxiated. What drug must be administered to the newborn to stimulate
breathing?
A. Lobeline
B. * Aethimizolum
C. Prazosin
D. Atropine
E. Proserine
17. A patient complaining of rapid pulse, dyspnea and bluish color of mucosa has been admitted to the
cardiological department. The objective symptoms are as follows: edema of lower extremities,
ascites. Which of the given medicines should be prescribed for intravenous administration to improve
the patient’s general state?
A. Cordiamin
B. Adrenalin hydrochloride
C. Digitoxin
D. * Corglyconum
E. Drotaverine
18. A child patient has dry cough. What non-narcotic antitussive drug will relieve the patient’s
condition?
A. Codeine phosphate
B. Morphine hydrochloride
C. Potassium iodide
D. Althaea officinalis root extract
E. * Glaucine hydrochloride
19. An patient with insomnia induced by allergic rash and itch has been prescribed the drug that has
antihistamine and hypnotic effect. Specify this drug:
A. Loratadine
B. Prednisolone
C. * Benadryl
D. Acetylsalicylic acid
E. Analgin
20. During local anesthetization the patient has gone into anaphylactic shock. What drug must be
administered to the patient?

A. Diazepam
B. Atropine sulfate
C. * Epinephrine hydrochloride
D. Propranolol
E. Nitroglycerin
21. A patient who had been taking diclofenac sodium for arthritis of mandibular joint developed an acute
condition of gastric ulcer. Such side effect of this medicine is caused by inhibition of the following
enzyme:
A. Cyclooxygenase-2 (COX-2)
B. Lipoxygenase
C. Phosphodiesterase
D. * Cyclooxygenase-1 (COX-1)
E. Monoamine oxidase
22. A 55-year-old male had been delivered to the resuscitation unit unconscious. Relatives reported him
to have mistakenly drunk an alcoholic solution of unknown origin. On examination the patient was
diagnosed with methanol intoxication. What antidote should be used in this case?
A. Teturamum
B. * Ethanol
C. Naloxone
D. Protamine sulfate
E. Acetylcysteine
23. An 8-week-pregnant woman with acute respiratory disease and temperature rise up to 39oC has called
in a doctor. The doctor insisted on her avoiding taking paracetamol, because in this period of
pregnancy there is a risk of its:
A. * Teratogenicity
B. Embryotoxicity
C. Fetotoxicity
D. Hepatotoxicity
E. Allergenicity
24. A pneumonia patient has been administered acetylcysteine as a part of complex therapy. What
principle of therapy has been taken into consideration when applying this drug?
A. Symptomatic
B. Etiotropic
C. Antimicrobial
D. Immunomodulatory
E. * Pathogenetic
25. A 68-year-old patient consults a cardiologist, complaining of high arterial blood pressure, pain in the
heart egion, intermittent pulse. Prescribe the beta-1-adrenoreceptor blocker for the treatment of the
described pathology:
A. Morphine hydrochloride
B. Nootropil
C. * Metoprolol
D. Fenoterol
E. Benzylpenicillin
26. A patient is diagnosed with acute morphine hydrochloride intoxication. Prescribe an oxidizing agent
for gastric lavage:
A. * Potassium permanganate
B. Chloramine

C. Sulfocamphocainum (Procaine + Sulfocamphoric acid)


D. Cerigel
E. Chlorhexidine (bi)gluconate
27. A 35-year-old female patient has been hospitalised with acute intoxication caused by salts of
high-density metals (lead,most probably).As a part of complex therapy the antidote that contains two
active sulfhydric groups has been prescribed. Specify this antidote:
A. Metamizole
B. Mannitol
C. * Dimercaprol
D. Nalorphine hydrochloride
E. Calcium chloride
28. A 60-year-old male patient has type II diabetes. A doctor has prescribed him synthetic hypoglycemic
long-acting drug that is sulfonylurea derivative. What drug is it?
A. Butamide
B. Metformin
C. Actrapid (soluble insulin)
D. * Glibenclamide
E. Acarbose
29. A patient has been given atropine sulfate for rapid relief of spastic colon symptoms. The use of this
drug is contraindicated during the following disease:
A. Bronchial asthma
B. Bradycardia
C. Hypotension
D. Gastric ulcer
E. * Glaucoma
30. A 63-year-old male patient with bladder atony had been prescribed a medication, which he had been
arbitrarily taking at a higher dose. The patient developed hypehydration, salivation, diarrhea, muscle
spasms. The prescribed drug relates to the following group:
A. Cholinesterase reactivators
B. Adrenergc blockers
C. Tocolytics
D. * Cholinomimetics
E. Ganglionic blockers
31. A patient complains of photoreception disorder and frequent acute viral diseases. He has been
prescribed a vitamin that affects photoreception processes by producing rhodopsin, the photosensitive
pigment. What vitamin is it?
A. Tocopherol acetate
B. Pyridoxine hydrochloride
C. Cyanocobalamin
D. * Retinol acetate
E. Thiamine
32. A patient with acne has been prescribed doxycycline hydrochloride. What recommendations should
be given to the patient, while he is taking this drug?
A. Take with large quantity of liquid, preferably milk
B. Take before meal
C. * Avoid long stay in the sun
D. The course of treatment should not exceed 1 day
E. Do not take with vitamins

33. A 30-year-old patient with a past history of virus B hepatitis complains of prolonged nosebleeds.
What drug will be most efficient in remedying this condition?
A. * Vicasolum
B. Fraxiparine
C. Folic acid
D. Dipiridamol
E. Asparcam
34. In cancer patients who have been continuously receiving methotrexate, the target cells of tumor with
time become insensitive to this drug. In this case, gene amplification of the following enzyme is
observed:
A. * Dihydrofolate reductase
B. Thiaminase
C. Deaminase
D. Thioredoxin reductase
E. Octadine
35. A patient with signs of emotional lability that result in troubled sleep has been prescribed nitrazepam.
Specify the sleep-inducing mechanism of this drug:
A. Blockade of opiate receptors
B. Inhibition of stimulating amino acids
C. H1-histamine receptors stimulation
D. Supression of serotonergic neurotransmission
E. * GABA-ergic system activation
36. After implantation of a cardiac valve a young man systematically takes indirect anticoagulants. His
state was complicated by hemorrhage.What substance content has decreased in blood?
A. Haptoglobin
B. Heparin
C. Creatin
D. * Prothrombin
E. Ceruloplasmin
37. A 4-year-old child was admitted to an orthopaedic department with displaced shin fracture. Bone
fragments reposition requires analgesia. What drug should be chosen?
A. Analgin
B. * Promedol
C. Morphine hydrochloride
D. Panadol
E. Gentamycin
38. Monoamine oxidase inhibitors are widely used as psychopharmacological drugs. They change the
level of nearly all neurotransmitters in synapses, with the following neurotransmitter being the
exception:
A. Noradrenaline
B. Adrenaline
C. * Acetylcholine
D. Dopamine
E. Serotonin
39. A patient with urolithiasis has developed severe pain attacks. For pain shock prevention he was
administered an antispasmodic narcotic analgesic along with atropine. Name this drug:
A. * Promedol
B. Nalorphine

C. Tramadol
D. Ethylmorphine hydrochloride
E. Morphine hydrochloride
40. Despite the administration of cardiotonics and thiazide diuretic a patient with chronic heart failure
has persistent edemas and the risk of ascites arose.What medication should be administered to
enhance the diuretic effect of the administered drugs and save K+ions?
A. Furosemide
B. Amiloride
C. * Spironolactone
D. Clopamide
E. Manithol
41. A 66-year-old woman had intravenous injection of magnesium sulfate solution to stop hypertensive
crisis. However her arterial pressure did not decrease and after repeated introduction of the same
preparation she developed sluggishness, slow response to stimuli; the patient is unconsciousness and
her respiration is inhibited. What preparation is antagonist of magnesium sulfate and can remove the
symptoms of its overdose?
A. * Calcium chloride
B. Potassium chloride
C. Sodium chloride
D. Activated charcoal
E. Potassium permanganate
42. A patient was prescribed loratadine to treat allergic cheilitis. What is the mechanism of action of this
drug?
A. * Blockade of H1-histamine receptors
B. Blockade of adrenergic receptors
C. Increases activty of monoamine oxidase
D. Suppresses activity of Na+/K+-ATPase
E. Suppresses activity of choline esterase
43. During introduction of local anesthesia a patient has gone into anaphylactic shock. What drug must
be administered to the patient?
A. * Epinephrine hydrochloride
B. Diazepam
C. Atropine sulfate
D. Propranolol
E. Nitroglycerin
44. When treating a patient with chronic cardiac failure a doctor detected bradycardia and deterioration
of the patient’s general state. Such condition is caused by cumulative effect of a drug. Which drug of
those listed below has cumulative action?
A. * Digoxin
B. Diphenhydramine (Dimedrol)
C. Hydrochlorothiazide
D. Isosorbide
E. Retinol acetate
45. A patient has developed paroxysmal ventricular tachycardia against the background of cardiac
infarction.What antiarrhythmic drug should be chosen to avoid lowering cardiac output?
A. Procainamide
B. Verapamil
C. Propranolol

D. Potassium chloride
E. * Lidocaine hydrochloride
46. Prolonged treatment of hypothyroidism has caused general dystrophy, dental caries, tachycardia,
tremor of extremities. What drug is the cause of these side effects?
A. Humulin (Human insulin)
B. * L-thyroxin
C. Parathyreoidinum
D. Thyrocalcitonin
E. Prednisolone
47. Name the halogen-containing antiseptic with fungicidal properties, which is used to treat
dermatomycosis:
A. Formalin solution
B. Methylene blue
C. * Iodine solution
D. Brilliant green
E. Boric acid solution
48. Due to severe pain syndrome a patient was prescribed a narcotic analgesic. Name this drug:
A. Metamizole (Analgin)
B. * Morphine
C. Nimesulide
D. Dimethyl sulfoxide
E. Indometacin
49. To treat rheumatoid arthritis a 65-year-old woman was prescribed an immunosuppressive hormonal
drug as a part of her complex therapy. Name this drug:
A. Thymus cytomedins (Thymalin)
B. * Prednisolone
C. Chloropyramine (Suprastin)
D. Riboflavin
E. Fercovenum
50. During narcosis a patient developed a risk of cerebral edema.What drug should be administered in
this case?
A. Dopamine
B. Phenazepam
C. * Furosemide
D. Triamterene
E. Sodium bromide
51. Name the drug that inhibits excretory function of pancreas during treatment of acute pancreatitis:
A. Allochol
B. Раnzynorm
C. * Contrykal (Aprotinin)
D. Pancreatin (Mezym forte)
E. Festal
52. An 18-year-old patient has developed candidiasis after the case of pneumonia treated with
beta-lactam antibiotic. What antimycotic agent should be prescribed?
A. Streptomycin
B. Ampicillin
C. * Fluconazole

D. Phthalylsulfathiazole
E. Trimethoprim/sulfamethoxazole (Biseptol)
53. A woman poisoned with unknown substance was hospitalised in a toxicological department. What
group of drugs can be administered to decrease absorption and introduction of the poison to her
body?
A. Neuroleptics
B. Antioxidants
C. * Adsorbents
D. Organic nitrates
E. Cholinesterase inhibitors
54. A patient after disrupted cerebral circulation has developed paralysis. Choose the anticholinesterase
drug to be prescribed in this case:
A. Cordiamin
B. Aceclidine
C. * Proserin
D. Methacin
E. Hexamethonium (Benzohexonium)
55. A patient is diagnosed with diabetic coma. Blood sugar is 18,44 mmol/l. What glucose-regulating
drug should be prescribed in the given case?
A. Intermediate-acting insulin
B. Long-acting insulin
C. * Rapid-acting insulin
D. Biguanide
E. Sulfonylurea derivative
56. A patient complaining of dizziness, thirst, difficult swallowing, and impaired vision of close objects
has addressed a doctor. Objectively: respiratory rate is increased, pupils are dilated, general agitation,
talkativeness, though the speech is indistinct. BP is 110/70 mm Hg, heart rate is 110/min. Given
symptoms can indicate overdosage of the following drug:
A. Morphine
B. * Atropine
C. Ephedrine
D. Aminazine
E. Caffeine
57. A 1-year-old child has been admitted to an orthopaedic department with displaced shin fracture. Bone
fragments reposition requires analgesia. What drug should be chosen?
A. Analgin
B. * Promedol
C. Morphine hydrochloride
D. Panadol
E. Caffeine
58. What drug will be the most appropriate for a patient suffering from chronic gastritis with increased
secretion?
A. Pancreatine
B. Pepsin
C. * Pirenzepine
D. Aprotinin
E. Chlorphentermine

59. A patient has been diagnosed with gonorrhea. As fluoroquinolones are the drugs of choce for
treatment of gonorrhea the patient should be prescribed:
A. Furazolidone
B. Fluorouracil
C. Sulfacarbamide (Urosulfanum)
D. * Ciprofloxacin
E. Cefazolin
60. A 40-year-old patient suffers from bronchial asthma and prolonged tachycardia. Choose the optimal
drug for rapid relief of bronchial spasm in the given case:
A. Adrenalin hydrochloride
B. Ephedrine hydrochloride
C. * Salbutamol
D. Orciprenaline
E. Isoprenaline (Isadrinum)
61. A patient suffers from acute cardiopulmonary failure with pulmonary edema. What diuretic should be
prescribed in the given case?
A. Triamterene
B. * Furosemide
C. Spironolactone
D. Hydrochlorothiazide (Dichlothiazidum)
E. Acetazolamide (Diacarb)
62. A 45-year-old woman suffers from arterial hypertension with high blood concentration of angiotensin
II. What antihypertensive drug is the most recommended in the given case?
A. * Lisinopril
B. Prazosin
C. Metoprolol
D. Reserpine
E. Verapamil
63. An alcoholic suffers from alcoholic psychosis with evident psychomotor agitation. What neuroleptic
drug should be administered for emergency aid?
A. Diazepam
B. Sodium bromide
C. * Chloropromarine
D. Reserpine
E. Interferon
64. A 26-year-old woman with bronchitis has been administered a broad spectrum antibiotic as a causal
treatment drug. Specify this drug:
A. Interferon
B. * Doxycycline
C. BCG vaccine
D. Ambroxol
E. Dexamethasone
65. A 16-year-old young man suffering from seasonal allergic rhinitis has been prescribed a highly active
secondgeneration H1 blocker, which can be characterized by absence of marked sedative action.
Name this drug:
A. Pipolphen
B. Chloropyramine (Suprastin)
C. * Loratadine

D. Indometacin
E. Erythromycin
66. A 36-year-old patient has been administered a depolarizing muscle relaxant during a surgery. Name
this drug:
A. Proserin
B. Pipecuronium bromide (Arduan)
C. Diazepam
D. Aminazine
E. * Succinilcholine
67. UN volunteers have arrived in Nigeria to assist the locals in after math of earth quakes. What drug
should they prescribe for individual chemoprophylaxis of malaria?
A. Pyrantel
B. Pyrimethamine (Chloridinum)
C. Primaquine
D. * Chingamin
E. Interferon (Laferon)
68. After an extended treatment with sulfanamides a patient has developed macrocytic anemia.
Production of active forms of the following vitamin is disrupted in such a condition:
A. Thiamine
B. Riboflavin
C. * Folic acid
D. Pyridoxine
E. Cyanocobalamin
69. Local anesthetics (novocaine, lidocaine and others) decreases pain sensitivity of tissues by blocking
Na+ and K+ ions from permeating membranes of nerve fibers and endings. Such mechanism of drug
action is called:
A. Receptor
B. Enzyme
C. * Membrane ionic
D. Antienzyme
E. Direct chemical
70. A patient presents with dry peeling skin, frequent cases of acute respiratory diseases, xerophthalmia.
What vitamin preparation should be prescribed in this case?
A. Thiamine
B. * Retinol acetate
C. Cyanocobalamin
D. Menadione (Vikasolum)
E. Ergocalciferol
71. Prescription of penicillin G sodium salt has caused development of neurotoxic effects (hallucinations,
convulsions). Such reaction is the result of antagonism with the following neurotransmitter:

A. Dopamine
B. Serotonin
C. * GABA
D. Adenosine
E. Acetylcholine
72. A patient with arthritis has been prescribed an anti-inflammatory selective COX2 inhibitor. Select this
drug among those given below:

A. Phenylbutazone (Butadion)
B. Dimethylsulfoxide (Dimexid)
C. Indometacin
D. Metamizole (Analgin)
E. * Celecoxib
73. The first-aid center has received a victim of a trafic accident diagnosed with closed displaced fracture
of the middle third of the thigh. For repositioning of bone fragments the patient received 10 ml of
2 % dithylinum solution intravenously, which resulted in prolonged period of apnoea and muscle
relaxation. What enzyme is deficient, resulting in such pharmacogenetic enzymopathy?
A. * Pseudocholinesterase
B. Uridine diphosphate glucuronyltransferase
C. Glucose 6-phosphate dehydrogenase
D. Methemoglobin reductase
E. N-acetyltransferase
74. A 42-year-old man with gout presents with high content of uric acid in blood. The patient was
prescribed allopurinol to lower the concentration of uric acid. Allopurinol is a competitive inhibitor
of the following enzyme:
A. * Xanthine oxidase
B. Adenosine deaminase
C. Adenine phosphoribosyl transferase
D. Hypoxanthine phosphoribosyl transferase
E. Guanine deaminase
75. A patient complains of dizziness, thirst, difficult swallowing, and impaired vision of close objects.
Objectively: respiratory rate is increased, pupils are dilated, general agitation, talkativeness, though
the speech is indistinct. BP is 110/70 mm Hg, heart rate is 110/min. Given symptoms can indicate
overdosage of the following drug:
A. * Atropine
B. Morphine
C. Ephedrine
D. Aminazine
E. Caffeine
76. A patient used an indirect-acting adrenergic agonist to treat rhinitis. After the patient has been putting
in the nose drops for several days, the vasoconstrictive effect of the drug gradually diminished. Name
this phenomenon:
A. * Tachyphylaxis
B. Idiosyncrasy
C. Teratogenicity
D. Allergy
E. Cumulation
77. A 56-year-old man complains of thirst and frequent urination. The endocrinologist diagnosed this
patient with diabetes mellitus and prescribed him glibenclamide. What mechanism of action does this
drug have?
A. * Stimulation of beta-cells of islets of Langerhans
B. Facilitates glucose uptake by the tissues
C. Facilitates glucose transport through cell membranes
D. Suppression of alpha-cells of islets of Langerhans
E. Inhibits glucose absorption in the intestine

78. Patients with bile duct obstruction typically present with inhibited blood clotting and develop
hemorrhages due to insufficient assimilation of vitamin:
A. * K
B. A
C. D
D. E
E. C
79. Condition of a patient with diabetes mellitus sharply deteriorated after a regular injection of insulin.
The patient became anxious and broke out in cold sweat; tremor of the extremities, general weakness,
and dizziness appeared. What medicine can remove these symptoms?
A. * Adrenaline
B. Tolbutamide
C. Caffeine
D. Noradrenaline
E. Glibutid (Buformin)
80. A schizophrenia patient was prescribed aminazine. What pharmacodynamic action of this drug is the
reason its prescription in this case?
A. * Antipsychotic
B. Antiemetic
C. Hypothermic
D. Muscle relaxant
E. Hypotensive
81. A man with ischemic heart disease has been taking his medicine too often throughout a day, which
resulted in poisoning. Examination detects cyanosis of skin and mucosa, sharp drop of blood
pressure, tachycardia, and respiratory depression. Blood methemoglobin is high. What type of
medicine did the patient overdose on?
A. * Organic nitrates
B. Alpha-adrenergic blockers
C. Calcium channel blockers
D. Adenosine-based drugs
E. Myotropic antispasmodics
82. Preoperative examination revealed prothrombin deficiency in the blood of the patient. What drug
should be preliminarily prescribed tomitigate blood lossin the patient during the surgery?
A. * Vicasol (Menadione)
B. Thrombin
C. Aminocapronic acid
D. Phenylin (Phenindione)
E. Contrykal (Aprotinin)
83. A patient with streptococcal infection of the gingiva was prescribed a drug with beta-lactam ring in
its structure. What drug of those listed below belongs to this pharmacological group?
A. * Benzylpenicillin
B. Rifampicin
C. Erythromycin
D. Streptomycin sulfate
E. Levomycetin (Chloramphenicol)
84. A 46-year-old patient suffering from ulcer disease of the stomach is diagnosed with rheumatoid
arthritis. What antiin?ammatory drug would be the most advisable in this case?
A. * Celecoxib

B. Prednisolone
C. Analgin (Metamizole)
D. Promedol (Trimeperidine)
E. Paracetamol
85. A patient with myocardial infarction has acute heart failure. Among the drugs that increase the force
of heart contractions the least dangerous in this case will be:
A. * Dobutamine
B. Adrenaline
C. Isadrinum (Isoprenaline)
D. Euphyllin (Aminophylline)
E. Caffeine
86. Monoamine oxidase inhibitors are widely used as psychopharmacological drugs. They change the
level of nearly all the following neurotransmitters in synapses EXEPT:
A. * Acetylcholine
B. Noradrenaline
C. Adrenaline
D. Dopamine
E. Serotonin
87. A patient suffers from acute cardiopulmonary failure with pulmonary edema. What diuretic should be
prescribed in the given case?
A. * Furosemide
B. Triamterene
C. Spironolactone
D. Hydrochlorothiazide (Dichlothiazidum)
E. Acetazolamide (Diacarb)
88. A 60-year-old man diagnosed with chronic heart failure was brought to the hospital. After a long
course of treatment the patient developed signs of intoxication: dyspnea, extrasystole, nausea, and
disturbed perception of colors. What medicine has caused such side-effects?
A. * Digoxin
B. Anaprilin (Propranolol)
C. Nitroglycerine
D. Drotaverine
E. Furosemide
89. A patient with exacerbated peptic ulcer disease of the stomach has made an appointment with the
doctor. What type of drugs should be included in the complex therapy of this patient?
A. * H2 antagonists
B. H1 antagonists
C. Alpha-adrenergicantagonists
D. Beta-adrenergicantagonists
E. Alpha-adrenergicagonists
90. A patient with parkinsonism was prescribed levodopa, which led to rapid improvement of the
patient’s condition. What is the mechanism of action of this drug?
A. * Stimulation of dopamine synthesis
B. Muscarinic acetylcholine receptor blockade
C. Stimulation of dopamine receptors
D. Anticholinesterase action
E. Muscarinic acetylcholine receptor stimulation

91. A patient with contactdermatitis needs to be prescribed an antihistamine drug without somnolescent
effect. Select this drug from the list:
A. * Loratadine
B. Dimedrol (Benadryl)
C. Suprastin (Chloropyramine)
D. Diprazine (Promethazine)
E. Ranitidine
92. Tostimulatethelaboractivityaparturient woman was prescribed a drug – a posterior pituitary hormone
that does not affect the blood pressure. As the pregnancy progresses, the sensitivity to this hormone
increases. Name the prescribed drug:
A. * Oxytocin
B. Dinoprostone
C. Dinoprost
D. Pituitrin
E. Ergotal
93. A patient, who has been taking beta-adrenergic blockers, developed a bronchial spasm. What group
of bronchodilators should be chosen to stop the bronchial spasm?
A. * Myotropic antispasmodics
B. Beta-adrenergicblockers
C. Indirect adrenergic agonists
D. Muscarinic cholinomimetic agents
E. Cholinesterase inhibitors
94. After a nephrectomy the patient developed enteroparesis. What cholinergic agent with
anticholinesterase action should be prescribed in this case?
A. * Proserin
B. Carbacholine
C. Aceclidine
D. Pilocarpine
E. Acetylcholine
95. A 25-year-old young man came to the doctor complaining of general weakness, rapid fatigability,
irritability, reduced working ability, and bleeding gums. What vitamin is likely to be de?cient in this
case?
A. * Ascorbic acid
B. Ribo?avin
C. Thiamine
D. Retinol
E. Folic acid
96. A patient with pulmonary tuberculosis is prescribed the most effective antituberculous antibiotic.
Name this drug:
A. * Rifampicin
B. Tetracycline
C. Streptocide
D. Furasolidone
E. Bactrim (Co-trimoxazole)
97. An 18-year-old patient has developed candidiasis after the case of pneumonia treated with
beta-lactam antibiotic. What antimycotic agent should be prescribed?
A. * Fluconazole
B. Streptomycin

C. Ampicillin
D. Phthalylsulfathiazole
E. Trimethoprim/sulfamethoxazole (Biseptol)
98. A patient was prescribed atropine sulfate to treat intestinal colic. What concomitant disease can be a
contraindication for this drug?
A. * Glaucoma
B. Bronchial asthma
C. Sinus bradycardia
D. Hypotension
E. Vertigo
99. A patient has developed status epilepticus. What medicine should be used in this case to stop the
seizures?
A. * Diazepam
B. Cyclodol (Trihexyphenidyl)
C. Diprazine (Promethazine)
D. Sodium bromide
E. Valerian extract
100. A 4-year-old child presents with numerous carious cavities and yellowcolored teeth. The mother has
a history of antibiotic treatment during her pregnancy. What antibiotic was the most likely taken by
the child’s mother?
A. * Doxycycline
B. Streptomycin sulfate
C. Ampicillin
D. Erythromycin
E. Cefazolin

You might also like